X



[English] -- 英文法・語法に関する質問 Part 22
レス数が1000を超えています。これ以上書き込みはできません。
0002名無しさん@英語勉強中 (ワッチョイ 5993-6Yq+)
垢版 |
2022/08/30(火) 21:52:24.57ID:WDtF3HE50
前スレまとめ
仮定法現在のthat節(that S V ...)はshouldを補うことが可能(that S should V ...)。
イギリス英語ではshouldを補った形がよく使われます。
よって仮定法現在に不慣れな方が文意を取る際はshouldを補って考えると良いでしょう。
0004名無しさん@英語勉強中 (ワッチョイ 3d96-+deR)
垢版 |
2022/08/30(火) 22:33:27.13ID:in3aAKUu0
「英語の動名詞・現在分詞・形容詞の区別について」
--- 日韓TOEIC公式問題集の文法解説は正しいか ---

https://cir.nii.ac.jp/crid/1390855190129763840
0005前スレ981 (ワッチョイ fee3-7jXZ)
垢版 |
2022/08/30(火) 23:08:47.12ID:NGKWw0XB0
Seeker先生にお尋ねします。
My hobby is の後に続くのが to不定詞か動名詞かという件に関して、

> ネイティブだけの結果が出るように検索すると、20万件中、15万件が動名詞を使い、5万件が不定詞を使うという割合 

という検索結果が得られたとのことですが、どうやって検索したのかもう少し具体的に教えていただけませんか?

前スレ1000を引用すると、
> >>***
> eduは例え(ひとつの例)であって、それだけで見てないから。
> >>***
> ネイティブの書いたページだけを選んでみればいいんだよ。そんなこともできないの?

ということですが、 site:edu というコマンドオプション以外にいくつかのオプションで検索し、
その中から「ネイティブの書いたページだけを選んで」、上記20万件の検索結果を得たのですか?

「ネイティブだけの結果が出るように検索する」方法をどうしても知りたいので質問しています。
しつこくてすいませんが、ご教示いただけますでしょうか? よろしくお願いします。
0006名無しさん@英語勉強中 (ワッチョイ a522-yNcK)
垢版 |
2022/08/30(火) 23:15:50.55ID:wcFiuN320
>>2

その説明を下の英文に適用すると、誤解を招くのでダメ。なぜ、誤解を招くかを説明してあげるので、次の質問に答えてね。

A necessary and sufficient condition for A to be invertible is that A should be square and of maximal rank.
----------------------------------
このshouldが、辞書のshould欄のどの項目で扱われているか、その項目名を明らかにせよ。
----------------------------------

この質問に答えることができる人は、英文法を熟知している人であり、上記英文の「should beの用法」と「仮定法現在の用法」が文法分類上別のものだと分かる人です。
0007名無しさん@英語勉強中 (ワッチョイ a522-yNcK)
垢版 |
2022/08/30(火) 23:24:55.47ID:wcFiuN320
>>5
たとえば、

site:uk "My hobby is to collect"
site:uk "My hobby is collecting"

上記2つを検索かけて、数を比較してごらん。イギリスでの結果がでるから。
こういう作業を地道に続けて統計を取るのさ。簡単に算出できるものではない。
0008名無しさん@英語勉強中 (ワッチョイ 5993-6Yq+)
垢版 |
2022/08/30(火) 23:29:27.58ID:WDtF3HE50
(1)A necessary and sufficient condition for A to be invertible is that A be square and of maximal rank.
(2)A necessary and sufficient condition for A to be invertible is that A should be square and of maximal rank.
(1)、(2)が表現している内容は同じです。
よって(1)の意味が取り辛い場合、(2)に置換して考えればよいのです。
0012名無しさん@英語勉強中 (ワッチョイ 5993-6Yq+)
垢版 |
2022/08/30(火) 23:35:43.61ID:WDtF3HE50
ネイティブが書いたものか否かを特定するには全件調査が必要です
口から出まかせでしょうね
いつもの悪い癖です
0013名無しさん@英語勉強中 (ワッチョイW c1fb-I0r4)
垢版 |
2022/08/30(火) 23:37:43.26ID:AIR0JHdj0
>>9
イギリス在住のパキスタン人は、二つのうちの両方にほぼ均等に分かれているだろうと想定して、
site::ukでイギリス英語でどちらが多いかを出せるんではないかと考えたと思われ
0015名無しさん@英語勉強中 (ワッチョイ fee3-7jXZ)
垢版 |
2022/08/30(火) 23:39:06.05ID:NGKWw0XB0
あ、まちがえた
site:uk "My hobby is to collect" 約933件
site:uk "My hobby is collectinging" 約338件
でした。動名詞の方が少ない!?

ちなみに、約2,650,000件というのは、
site:uk "My hobby is to collectinging" の、引用符無しの検索結果でした m(_ _)m
0017名無しさん@英語勉強中 (ワッチョイ a522-yNcK)
垢版 |
2022/08/31(水) 00:04:13.49ID:/JTFUXUO0
>>11
デマ書くなよ。

site:uk "My hobby is to collect" 約 15,200 件 (0.22 秒)
site:uk "My hobby is collecting" 約 338 件 (0.24 秒)

この比率は異常値なので、信頼しないほうがいいが、少なくとも、私の「ネイティブがto collectを使うケースをマレだと言うのは言い過ぎ」が証明されたね。

>>12
日本人やその他の外国ドメインを除けば、ネィティブだけに近い近似値はでるんだよ。だから、edu, us, mil gov, ukでクロスチェックすれば相当真実に近づける。
一人残らず非ネイティブを排除するなんてできるわけないだろ。
0018名無しさん@英語勉強中 (ワッチョイ a522-yNcK)
垢版 |
2022/08/31(水) 00:07:29.58ID:/JTFUXUO0
>>14
間違ったデータに踊らされる可哀そうなお人 ^^

>>17がとりあえず、実際の検索結果。 異常値だけどね。
0019名無しさん@英語勉強中 (ワッチョイ a522-yNcK)
垢版 |
2022/08/31(水) 00:10:58.49ID:/JTFUXUO0
>>8
それだと、質問者の「この仮定法現在の用法は何ですか?」に答えてないだろ。

さ、はやく、次の質問に答えてね。
A necessary and sufficient condition for A to be invertible is that A should be square and of maximal rank.
----------------------------------
このshouldが、辞書のshould欄のどの項目で扱われているか、その項目名を明らかにせよ。
----------------------------------
0023名無しさん@英語勉強中 (ワッチョイ 5993-6Yq+)
垢版 |
2022/08/31(水) 01:07:42.70ID:PqlJz/Vq0
should にはいろんな用法がありますが、その多くに共通してるのは「想定・仮定」のニュアンス。
だから同じく「想定・仮定」を表現する仮定法現在とは相性抜群なんですね〜
願望も想定の一種だけど、願望だけ特別扱いするなんて不自然だと思いませんか?
もし思ったならその感覚正しいです。デマに惑わされないようにしましょう。
0024名無しさん@英語勉強中 (ワッチョイ 5993-6Yq+)
垢版 |
2022/08/31(水) 02:25:27.52ID:PqlJz/Vq0
(ex.1)To obtain an interference pattern, the necessary condition is that the two sources should be coherent.
(ex.2)For distinct interference pattern to be observed, necessary condition is that ratio of intensity of light emission by both the sources should be:(1)2:1 (2)1:2 (3)1:1 (4)1:4
(ex.3)The first condition is that actions should be devoted to Allah Alone.
(ex.4)Other material indicates that the condition is that spectral radius of matrix H should be less than 1.
(ex.5)Then we have a “while” loop whose iterating condition is that the value of the variable “number” should be less than 10.
(ex.6)The third condition is that the therapist should be, within the confines of this relationship, a congruent, genuine, integrated
person.
(ex.7)The necessary condition is that the coefficients of the characteristic polynomial should be positive.

shouldを補えない? またまたご冗談を
0027Seeker2022 ◆iymFBv30wPKz (ワッチョイ a522-yNcK)
垢版 |
2022/08/31(水) 05:23:57.23ID:/JTFUXUO0
今、「My hobby is to」と「My hobby is -ing」について再検証してみた。

そしたら、
site:uk "My hobby is to collect" 約 15,200 件 (0.22 秒)
site:uk "My hobby is collecting" 約 338 件 (0.24 秒)
のような、異常値が含まれていることが判明。

使用率「15:5」は、信用できない数字だと判明したので、撤回したい。ネイティブでも不定詞を使う人がいることは事実だが、不定詞を使う人はごく少数派であると考えて差支えないと思う。

↓これで結果数を比較すると、まあ、使うネイティブはいることはいるのだが。
site:com "My hobby is to communicate" 
site:com "My hobby is communicating" 
0028Seeker2022 ◆iymFBv30wPKz (ワッチョイ a522-yNcK)
垢版 |
2022/08/31(水) 05:28:49.94ID:/JTFUXUO0
>>23
さ、はやく、次の質問に答えてね。
A necessary and sufficient condition for A to be invertible is that A should be square and of maximal rank.
----------------------------------
このshouldが、辞書のshould欄のどの項目で扱われているか、その項目名を明らかにせよ。
----------------------------------

上記英文は確かに「A be square」と置き換えらえる。しかし、このshouldの用法は、常に仮定法現在と置き換えられる用法じゃないの。
たまたま、置き換えらるだけ。両者は別用法なの。だから、文法上同一用法であると、読者に錯覚させてはダメなのだ。
0033名無しさん@英語勉強中 (ワッチョイW 799d-fJpa)
垢版 |
2022/08/31(水) 06:58:21.19ID:d25R5Yrw0
夏休み終わるから先生は忙しくなるのか
0034Seeker2022 ◆iymFBv30wPKz (ワッチョイ a522-yNcK)
垢版 |
2022/08/31(水) 10:42:02.20ID:/JTFUXUO0
訂正
↓これで結果数を比較すると、まあ、使うネイティブはいることはいるのだが。
site:com "My hobby is to communicate" 
site:com "My hobby is communicating" 

この結果は、説明するのに悪い例だった。communicateの例は、無視してくれ。
0035Seeker2022 ◆iymFBv30wPKz (ワッチョイ a522-yNcK)
垢版 |
2022/08/31(水) 10:45:41.15ID:/JTFUXUO0
次の実例から、My hobby is toをネイティブが使う例がごく少数派であるが存在することがわかる。

My hobby is to collect sports cards
https://www.unm.edu/~rbose/abose/favorites.htm

My hobby is to use education for sane living.
https://www.ferris.edu/HTMLS/news/founder/woodbridge/25habits.htm

His main hobby is to travel and learn about other cultures.
https://rockdalecountyga.gov/cpo_team/james-jamie-f-cabe-mpa/

ネイティブの中に不定詞で書くことをおかしいとみなさない人たちがいる証拠↓
https://ja.hinative.com/questions/178218
0038Seeker2022 ◆iymFBv30wPKz (ワッチョイ a522-yNcK)
垢版 |
2022/08/31(水) 11:03:15.35ID:/JTFUXUO0
37は間違ってるね。ほんと自動翻訳は妄信できないなあ。Sie(あなた)じゃなく、ich(私)だろ。
Dies ist das letzte Mal, dass ich eine Nachricht in diesem Nachrichten-Thread veroffentlicht habe. ^^

これで正しい文になったかな? 余計なことしなきゃよかった\(^o^)/

では、これで本当にこの板とはお別れだ。
0043名無しさん@英語勉強中 (ワッチョイ 025d-SvMG)
垢版 |
2022/08/31(水) 17:32:07.92ID:exPmI+m50
ネイティブが使ってた文章でわからないのがあったので質問させて下さい。

We need to know what their token is we assigned them

上記の2個目のwe以降なんですが、 (that) we assigned them (their token) の様に省略されている関係代名詞の文かと思ったんですが、何故tokenの直後に来ないのでしょうか、、?
0044名無しさん@英語勉強中 (ワッチョイ fee3-7jXZ)
垢版 |
2022/08/31(水) 19:12:48.45ID:PcSIE7pd0
We need to know what their token we assigned them is,.
とすると、
what[C] [their token we assigned them(S)] [is(V)],
この部分のSが長すぎてwhatSVの関係が分かりにくくなってしまいます。
これを避けるためにwe assigned them が後ろに回された、と考えるの良いかと思います。
0046名無しさん@英語勉強中 (ワッチョイ fee3-7jXZ)
垢版 |
2022/08/31(水) 20:09:26.08ID:PcSIE7pd0
こういうのは「外置 (Extraposition)」と言うらしいです。
https://en.wikipedia.org/wiki/Extraposition

通常の語順
Someone whom we don't know left a message.

外置 (whom we don't know)が後ろに回される
Someone left a message whom we don't know

この点について記述されている文法書をご存知の方はご教示ください。
よろしくお願いします。
0048名無しさん@英語勉強中 (ワッチョイ 5993-6Yq+)
垢版 |
2022/08/31(水) 21:09:44.53ID:PqlJz/Vq0
>>28
>上記英文は確かに「A be square」と置き換えらえる。
じゃあ「should を補えない」は間違いですね。

>たまたま、置き換えらるだけ。
should を補えない仮定法現在の文の例を挙げてみて。
0051😉三年英太郎🌈 ◆3CZBjOt3.Y (JPW 0He6-ZQFb)
垢版 |
2022/09/01(木) 01:53:05.78ID:kyoun3dwH
>my friend は会話の参加者が誰なのかわかっている場合や, 名前の前に置いてその友人を紹介したり, その友人について話したりする場合に用いる:
My friend Yuki is a dentist.
友達の由紀は歯科医なんです;
0052名無しさん@英語勉強中 (ワッチョイW c1fb-I0r4)
垢版 |
2022/09/01(木) 05:41:21.50ID:YrhCZRrG0
一番最後の使い方とその例文がとてもわかりやすかった。
教えてくれてありがとう。

myという人称代名詞の所有格はtheにさらに私のという意味が加わったもの
だから、と書いたら、それは違うと思いますか?
知らないから尋ねているのですが?
0065名無しさん@英語勉強中 (ワッチョイW c7fb-7yAK)
垢版 |
2022/09/03(土) 00:19:39.89ID:MMfh8UOO0
a fan of part of your works
でいいのでは?
0066名無しさん@英語勉強中 (ワッチョイW c7fb-7yAK)
垢版 |
2022/09/03(土) 00:20:35.77ID:MMfh8UOO0
a fan of some part of your works
に訂正
0068名無しさん@英語勉強中 (ワッチョイW c7e3-/6ZG)
垢版 |
2022/09/03(土) 19:26:09.73ID:2QY33JCF0
カナダで生まれ育ったカナダ人がメールでこう書きました。
My cousin has been staying with me for the last month, so I've been spending more money than usual on groceries.
それで私が“from” the last month だよね?とツッコんだら、その友人が、いや違うfor the last month またはsince the last monthだ。 from the last monthだとvery unnaturalだ。って言うんです。

for の用法を私なりに調べても、「〜以来」っていうのは見つからないです。だいぶ納得いきません。
どなたか解説をお願い致します。
0069名無しさん@英語勉強中 (ワッチョイ 5fe2-InTp)
垢版 |
2022/09/03(土) 19:29:11.54ID:/xbP9cCa0
>>68
for the last ten years
この 10 年間. - 研究社 新英和中辞典

for the last week
ここ 1 週間. - 研究社 新英和中辞典
0071名無しさん@英語勉強中 (ワッチョイW 079d-y/4f)
垢版 |
2022/09/04(日) 07:45:12.13ID:gDwjy2eQ0
すいません。アメリカ人に
アニメキャラの似顔絵 ラフ画を出したら
Yours face looks off
と返事がありました。
似てないてことでしょうか?
007268 (ワッチョイW c7e3-/6ZG)
垢版 |
2022/09/04(日) 10:18:28.11ID:KEuEXeOH0
>>69
>>70
ありがとうございます。
この場合にforを使うって、結構ショックです。
やっぱり英語は難しいですね。
0074名無しさん@英語勉強中 (ワッチョイ c7f0-6nWD)
垢版 |
2022/09/04(日) 12:41:46.83ID:8js0LJuD0
例えば今日が9月4日だとして、
for the last monthと言えば、だいたい8月の上旬くらいからというニュアンスになりますよね。

since last monthと言うと、8月のいつからでも構わないことになるので、
だいぶ日数に幅が出ますね。

「ここひと月」 と 「先月から」の違いみたいなもんでしょうか。
0075名無しさん@英語勉強中 (ワッチョイW c7e3-/6ZG)
垢版 |
2022/09/04(日) 13:35:38.42ID:KEuEXeOH0
>>73
Are you satisfied, huh?
0076名無しさん@英語勉強中 (ワッチョイW df8a-Iqu5)
垢版 |
2022/09/04(日) 19:47:47.19ID:qZHJ8GwV0
>>68
確かに俺もそれ間違ってると思っちゃったわ
普通に「先月」って意味だと思っちゃった
ちなみにfrom は普通に学校で習う文法的にも普通に間違い
0078名無しさん@英語勉強中 (ワッチョイ 5f33-bBdM)
垢版 |
2022/09/06(火) 10:18:06.53ID:PJckHKn10
テスト
0079名無しさん@英語勉強中 (ワッチョイ 5f33-bBdM)
垢版 |
2022/09/06(火) 10:21:02.71ID:PJckHKn10
以下の定理のステートメントで,「at the point a of A」と書いてありますが,なぜ,「at a point a of A」
ではないのでしょうか?非常に不自然に感じます.

Theorem 8.3 (The inverse function theorem).

Let A be open in R^n; let f : A → R^n be of class C^r. If Df(x) is non-singular
at the point a of A, there is a neighborhood U of the point a such that f carries U
in a one-to-one fashion onto an open set V of R^n and the inverse function is of class
C^r.
0081名無しさん@英語勉強中 (ワッチョイ 5f33-bBdM)
垢版 |
2022/09/06(火) 10:44:37.12ID:PJckHKn10
問題の文は,「Df(x)がAの点aで非特異ならば,…」です.

点aというのは初めて登場するので「the」ではなく「a」が適切じゃないですか?
0082名無しさん@英語勉強中 (ワッチョイ 5f33-bBdM)
垢版 |
2022/09/06(火) 10:46:34.07ID:PJckHKn10
「at the point a of A」と突然言われても「え?どこの点のこと?」ってなりませんか?
0084名無しさん@英語勉強中 (ワッチョイ 5f33-bBdM)
垢版 |
2022/09/06(火) 11:50:35.38ID:PJckHKn10
「Df(x)がAのある点aで非特異ならば,」

なのでやはり「the」ではなく「a」が正しいと思います.
0086名無しさん@英語勉強中 (ワッチョイ 0793-HH83)
垢版 |
2022/09/06(火) 12:12:37.13ID:6awZKSy00
aは固有の点だから定冠詞になる
aなどと限定せずに或るひとつの点を言う場合は不定冠詞になる
0087名無しさん@英語勉強中 (ワッチョイ 5f33-bBdM)
垢版 |
2022/09/06(火) 12:33:21.55ID:PJckHKn10
関数fを開区間(c, d)で定義された実数値関数とする.
fが点aで微分可能ならば点aで連続である.

以下の2つのうち,どちらが正しいですか?

Let f be a real-valued function defined on an open set (c, d).
If f is differentiable at a point a of (c, d), then f is continous at the point a.

Let f be a real-valued function defined on an open set (c, d).
If f is differentiable at the point a of (c, d), then f is continous at the point a.
0088名無しさん@英語勉強中 (ワッチョイ 5f29-9TNW)
垢版 |
2022/09/06(火) 13:02:05.13ID:syALXHCD0
自然さを聞くならともかく、正しさを聞くならどっちでもいいよそんなもんどっちが正しくないとかはない完全に通じる
しいて言えば上はなんか含みを感じる人もいるかもくらい んな細かいこと気にするならまず誤字をなくせよ
0089名無しさん@英語勉強中 (ワッチョイ 0793-HH83)
垢版 |
2022/09/06(火) 13:03:45.99ID:6awZKSy00
どちらも間違い
0091名無しさん@英語勉強中 (ワッチョイ bf2c-BZR6)
垢版 |
2022/09/06(火) 13:22:11.36ID:1A+wtVE70
>>88 のいう通り誤字がなければどちらでも良いのだが、上の方が自然かな

英語はあんたの文法に従っているのではない
まずは書かれているものを受け入れなさい
そのうちに >>79>>87 の違いがわかってくる
0094名無しさん@英語勉強中 (ワッチョイ 6632-25Bf)
垢版 |
2022/09/13(火) 14:51:14.61ID:guqEZWxR0
・What I want to do most is to find new practical methods from various approaches.
・What I want to do most is finding new practical methods from various approaches.

この文はどちらが文法的に正しいでしょうか?
0095名無しさん@英語勉強中 (ワッチョイ 1593-aQ9k)
垢版 |
2022/09/13(火) 16:37:07.03ID:1qAQQa630
to find
0099😉三年英太郎🌈 ◆3CZBjOt3.Y (TWW 0He1-01TO)
垢版 |
2022/09/14(水) 21:02:56.47ID:a+wkAoXgH
コーパスは、自分が望む文字列の検索じゃなくて、むしろこーゆーことに真価を発揮するのですよ

ググるのと変わりなしとか言ってた愚かな人がいました😥
0101名無しさん@英語勉強中 (ワッチョイW 97e3-KXRs)
垢版 |
2022/09/17(土) 15:24:14.96ID:0ia4tC7i0
The bugs came out in droves and they wouldn’t leave us alone.
“wouldn’t leave”という部分について教えてください。
これはどういうニュアンスなのですか?
they didn’t leave とはどう違うのですか?
0102名無しさん@英語勉強中 (ワッチョイ f732-cETI)
垢版 |
2022/09/17(土) 15:30:22.60ID:IqQuGsKw0
>>101
虫が群れを成してやってきてまとわりついて離れなかった
って感じかな

wouldn't は (手で追っ払うとか 何をやっても) ~しようとしない という強い否定
辞書を引いたら The door wouldn't open 「何をやってもドアは開かなかった」のような文が載ってると思う
0103名無しさん@英語勉強中 (ワッチョイ 5793-bG2j)
垢版 |
2022/09/17(土) 18:50:49.32ID:q7HiBYBc0
wouldは時制の一致
強い意志を表すwill
主語は人に限らない 物でも使える 恐らく擬人化
0104名無しさん@英語勉強中 (オッペケ Srcb-dBhJ)
垢版 |
2022/09/17(土) 19:20:21.76ID:pHMjb+gAr
powers that be のbeは仮定法現在じゃなくて
まんま直接法らしい
0105名無しさん@英語勉強中 (ワッチョイW 97e3-KXRs)
垢版 |
2022/09/17(土) 19:32:51.77ID:0ia4tC7i0
>>102
ありがとうございます!
これ、覚えておいた方がいい表現ですね!
0106名無しさん@英語勉強中 (ワッチョイW 97e3-KXRs)
垢版 |
2022/09/17(土) 19:46:58.77ID:0ia4tC7i0
倒置について教えてくださいませ。
カナダ人が、
It would’ve been nice had it not rained. とレスしました。
これ、
It would’ve been nice if it had not rained. とは言わないのでしょうか?
0113The OED Loves Me Not (ワッチョイ ff89-W3aP)
垢版 |
2022/09/18(日) 05:05:39.63ID:ewBIjUek0
>>104
なるほど。そのことについては、StackExchange に次のような解説があるね。

68

To expand on Henry's answer: "The powers that be" is a set phrase quoted from Romans 13:1.

Let every soul be subject unto the higher powers. For there is no power but of God:
★the powers that be★ are ordained of God.

In that context, it means "the temporal powers that indisputably do exist in the world,"
so we can rule out explanations that call on the subjunctive mood — here Paul isn't talking
about "the powers that [may or may not] be", ●he's definitely talking about what we today would
call "the powers that are."★
0114名無しさん@英語勉強中 (ワッチョイ 975d-ffU0)
垢版 |
2022/09/18(日) 12:06:10.69ID:ibEm3+iv0
「和文英訳の修業」(p45)に次の例文があります.
Don't lose hope and get desperate when everything seems to go against you.

このようにdon'tが後ろの二つの動詞(hopeとget)にかかっていると思いますが,正しいでしょうか?
また,このような書き方はよくあり,そして文法的に問題ないのでしょうか?
0115The OED Loves Me Not (ワッチョイ ff89-W3aP)
垢版 |
2022/09/18(日) 12:49:25.14ID:ewBIjUek0
Don't lose hope and get desperate.
のみならず、一般的に
"not AAA and BBB"
は、本来(厳密な意味で論理的な文章の中においてだったら)
「AAAして、しかも BBB すること」を行うな、という意味だけど、だったら
AAA と BBB とを同時に行うことだけを避ければよいのであって、AAA だけを
行い、その代わりに BBB を行わないのは許されるのだな、と言いたくなる。しかし
このような日常的な表現の場合、しかも AAA と BBB とがほぼ同義語の場合には、
Don't AAA and BBB.
とは書いてあっても、実用上は
Don't AAA or BBB.
と大して意味は変わらないものだと人々は認識しているようだ。

今回の例文と似たようなものとして、AAA と BBB とがほぼ同義語の場合として、
次のようなものがネット上で見つかる。

Don't lose hope and give up.
Don't lose hope and stop praying now!
Don't lose hope and stop trying.

他にも、こういう用例はいくらでも見つかると思う。
0116名無しさん@英語勉強中 (JP 0Hab-wzEt)
垢版 |
2022/09/18(日) 12:52:21.87ID:4ziYJaVgH
>>114
日本語では少ないけどn(a+b), (a+b)nみたいな構造は結構柔軟に使われると思う
日本語の古文でも連用形+連用形否定で両方否定するのあったけどいつ消えていったんやろな
0118114 (ワッチョイ 975d-ffU0)
垢版 |
2022/09/18(日) 17:21:27.46ID:ibEm3+iv0
>>115,116,117
レスありがとうございます.

"not AAA and BBB"
と書いたとき,
"(not AAA) and BBB"
と読まれることがないかが心配なのですが, そういうことは英語的にはありえないのでしょうか?
元の例だと「希望を失う」と「自暴自棄になる」なので,意味的にあり得ないのでその心配はなさそうんですが,意味的には,"not (AAA and BBB)"と"(not AAA) and BBB"が成立する場合だと,心配になります.どうでしょうか?
プログラミング言語だと明確なルールがあったり,()が使えたりして,悩まないですが...

コメントいただければ助かります.
0119The OED Loves Me Not (ワッチョイ ff89-W3aP)
垢版 |
2022/09/18(日) 17:48:38.29ID:ewBIjUek0
>>118
>>"(not AAA) and BBB"

まともな英語ネイティブなら、それを意味したいときには
Don't AAA, but do BBB.
または
BBB, but don't AAA.
というふうに書くはずだ。
0120The OED Loves Me Not (ワッチョイ ff89-W3aP)
垢版 |
2022/09/18(日) 18:17:51.40ID:ewBIjUek0
>>118
Don't lose hope and get desperate.
ここでもしも
lose hope してはいけないけど、get desperate はしないといけない、と言いたいのなら、
(1) Don't lose hope, but do get desperate.
とか
(2) Get desperate, but don't lose hope.
と言えばいいし、少しまともなネイティブならそのように言うだろう。ただし
いい加減なネイティブは、変な英文を書くだろうから、そのときには文脈から
ああだこうだと考えざるを得ない。

もしも get desperate もしてはいけないし、lose hope もしてはいけない(両方ともダメ)
という趣旨を、絶対に誤解されたくないなら、次のように書けばいい。
(1) Don't get desperate or lose hope.
(2) Don't get desperate. Don't lose hope, either.

というふうに、まともなネイティブなら誤解されにくい英文を書くと思う。
少なくともジャーナリストや作家など、著述家はそうする。
0121The OED Loves Me Not (ワッチョイ ff89-W3aP)
垢版 |
2022/09/18(日) 18:19:36.78ID:ewBIjUek0
もちろん今回の場合は、質問者もわかっている通り、動詞が
lose hope
get desperate
というものだから、最初の文でも誤解されることがないけどね。
0123名無しさん@英語勉強中 (ワッチョイ 5793-bG2j)
垢版 |
2022/09/18(日) 20:07:33.78ID:lTzcuSNC0
>A XOR B -> either A or B
え?
0125名無しさん@英語勉強中 (ワッチョイW 9ff0-wx9k)
垢版 |
2022/09/18(日) 21:49:47.24ID:iRbFb2+O0
英会話通いたいと思ったんだけど
単語も文法も全て無いのに通っても「??」となって無駄ですかね

受験のように英文法学ぶか、リスニング用の文法ってのを学んでからの方がいいですか
0128名無しさん@英語勉強中 (ワッチョイW 1f8a-JmM/)
垢版 |
2022/09/18(日) 22:23:33.81ID:iHxEbsan0
>>125
それはマジでやめろ
本当に無駄
0130名無しさん@英語勉強中 (ワッチョイ 5793-bG2j)
垢版 |
2022/09/18(日) 23:39:41.23ID:lTzcuSNC0
>>124
OR と XOR の違いを説明してみて
0131114 (ワッチョイW 975d-v9er)
垢版 |
2022/09/18(日) 23:46:24.88ID:ibEm3+iv0
>>120

すごく参考になりました.
大変ありがとうございます.
0132The OED Loves Me Not (ワッチョイ ff89-W3aP)
垢版 |
2022/09/19(月) 06:10:27.62ID:Y8hRFC4A0
>>131
>>もしも get desperate もしてはいけないし、lose hope もしてはいけない(両方ともダメ)
>>という趣旨を、絶対に誤解されたくないなら、

もう一つ、誤解されないための言い方を思いついた。

(1) Don't lose hope and get desperate at the same time.
(つまり、2つのことを同時にやってはいけないよ〜ん)

(2) Don't lose hpe while getting desperate.

ネット上で見つけた類似表現:
(a) Please don't text and walk at the same time.
(b) Most dogs don't pee and poop at the same time, ....
(c) She captioned it, “don't watch bachelor in paradise and crave chicken tenders & fries at the same time…. you'll cry.
(d) This is because electrons simply don't have a definite position, and direction of motion, at the same time!
(e) Don't Try to Be a Publisher and a Platform at the Same Time ...

この他にも、
"don't * anfdd * at the same time"
をそのまま検索したら、似たような例文がずらずら並んで出てくると思う。
0133The OED Loves Me Not (ワッチョイ ff89-W3aP)
垢版 |
2022/09/19(月) 06:12:49.92ID:Y8hRFC4A0
あっと、ちょっと勘違いしてた。>>132 で並べた表現はすべて、「2つのことを
同時にやってはいけない」つまり「一方だけやるのは OK」という意味だな。
「どっちもダメ」という意味ではないよね。申し訳ない。
0134The OED Loves Me Not (ワッチョイ ff89-W3aP)
垢版 |
2022/09/19(月) 06:20:51.29ID:Y8hRFC4A0
>>131
Don't lose hope while getting desperate.
これも「一方をやりながら他方をやってはいけない」つまり「一方だけなら OK」と
いう意味になるけど、それについても
"don't * while"
をそのまま検索したら、類似表現が並ぶと思う。
例えば、次のようなもの。

(1) DON'T LISTEN TO THIS WHILE DRIVING.
(2) Make sure you don't lose the moon while counting the stars.

というわけで、誤解されない言い回しは他に考えられるはずなのに、それでも
(あとで推敲したあとでも)なおかつ誤解されてしまう言い回しになってしまう
ような英文は、きちんとした新聞雑誌や小説やノンフィクションでは少ないからこそ、
そういうまともな文章ばかりなるべく読むようにしたらいいよね。そして Twitter とか YouTube 上の
視聴者の気楽なコメントとか文章の下手そうな人のブログを読むときには、
いい加減な書き方をして両義的になってしまっているかもしれないと
覚悟するしかないと思う。
0135😉三年英太郎🌈 ◆3CZBjOt3.Y (ワッチョイW 9f86-Pbkh)
垢版 |
2022/09/19(月) 07:20:58.95ID:eqT4f5ZW0
仮に Don't lose hope and get desperate. を Don't lose hope while getting desperate. と言い換えたとして、希望失うのだけならOK♪と思う人いるの?

Don't take cocaine while you are high with heroin. どっちか一方ならOK❓🤓 薬中かな❓

> Make sure you don't lose the moon while counting the stars.

「counting the stars だけならOK😝」なんて取ったら、言った人ズッコケるだろうね🤦🏻‍♂
0137😉三年英太郎🌈 ◆3CZBjOt3.Y (ワッチョイW 9f86-dzFe)
垢版 |
2022/09/19(月) 09:54:07.33ID:eqT4f5ZW0
What does this quote mean "Never ignore a person who loves you, cares for you, misses you. Because one day, you might wake up from your sleep and realize that you lost the moon while counting the stars."?

https://www.quora.com/What-does-this-quote-mean-Never-ignore-a-person-who-loves-you-cares-for-you-misses-you-Because-one-day-you-might-wake-up-from-your-sleep-and-realize-that-you-lost-the-moon-while-counting-the-stars

一方だけならOK、月なら月、星なら星を追いかけなよ、という意味なんだ…❓🤔 

英語に命かけてないから分かんないや😥
0139名無しさん@英語勉強中 (アークセー Sxcb-TkCD)
垢版 |
2022/09/19(月) 12:01:11.84ID:prUNb1zpx
https://i.imgur.com/HgesGr7.jpg
https://i.imgur.com/qR0rWfh.jpg
https://i.imgur.com/zfdS4FI.jpg
https://i.imgur.com/ov6XAeM.jpg
https://i.imgur.com/Pi5Tl2e.jpg
https://i.imgur.com/BKud6xL.jpg
https://i.imgur.com/lMsW6Sn.jpg
https://i.imgur.com/0d9sSW0.jpg
https://i.imgur.com/DwPoxSC.jpg
https://i.imgur.com/6zvS2Ps.jpg
https://i.imgur.com/F3UvEZY.jpg
https://i.imgur.com/EXI6F2s.jpg
0141名無しさん@英語勉強中 (ワッチョイ 5793-ret5)
垢版 |
2022/09/19(月) 16:03:34.01ID:8q4MzCSc0
高校文法からやって分からないところだけ中学文法に戻った方が効率的かもね
0143名無しさん@英語勉強中 (テテンテンテン MM8f-W3aP)
垢版 |
2022/09/20(火) 00:38:16.46ID:/yxsKNjoM
名詞 and 名詞 の場合、通常、andは論理積(かつ)という意味だろうが

動詞 and 動詞 の場合は、単に「そして」みたいな意味だろ。
「〜してから〜する」みたいな。
それを否定してるだけじゃん。
0146名無しさん@英語勉強中 (テテンテンテン MM8f-W3aP)
垢版 |
2022/09/20(火) 01:38:27.75ID:/yxsKNjoM
>>144
名詞 and 名詞 が論理積とはドンでもないことを
言ってしまった。
和集合というべきだった。
論理積は、命題 and 命題の場合だな。

とにかく
2つの動詞が「そして」みたいな意味のandで結ばれている場合は
ド・モルガンの法則を当てはめるような対象ではないだろうということを
言いたかった。
0148名無しさん@英語勉強中 (テテンテンテン MM8f-W3aP)
垢版 |
2022/09/20(火) 02:02:45.84ID:/yxsKNjoM
>>145
動詞 and 動詞 の場合のandが
常に「そして」みたいな意味であるという
ような書き方をしてしまいましたが
それも訂正します。

動詞 and 動詞 のandが
「…して(から), それから」という意味で使われている場合
それに否定辞がかかっている場合
ド・モルガンの法則は関係ありません。

という主張に変更します。
0149名無しさん@英語勉強中 (テテンテンテン MM8f-W3aP)
垢版 |
2022/09/20(火) 02:04:07.41ID:/yxsKNjoM
>>147
↓これに反論できる?中卒馬鹿

動詞 and 動詞 のandが
「…して(から), それから」という意味で使われている場合
それに否定辞がかかっている場合
ド・モルガンの法則は関係ありません。
0151名無しさん@英語勉強中 (テテンテンテン MM8f-W3aP)
垢版 |
2022/09/20(火) 02:17:41.70ID:/yxsKNjoM
少し英文を変えてみる。

Never lose hope and commit suicide.

このandは論理積を表しているというより、時間的連続、因果関係を表していると考える方が普通であろう。
ド・モルガンの法則は関係ないとわかるだろう。
0152名無しさん@英語勉強中 (テテンテンテン MM8f-W3aP)
垢版 |
2022/09/20(火) 02:18:43.12ID:/yxsKNjoM
>>150
じゃあ、どっか行ってろよ、中卒
0153名無しさん@英語勉強中 (テテンテンテン MM8f-W3aP)
垢版 |
2022/09/20(火) 02:20:30.75ID:/yxsKNjoM
>>150
日本語が意味不明とだけ言えばいいのに
「議論と日本語が意味不明」とか言うところが
アホっぽいw
中卒だからしゃーないか。
0154名無しさん@英語勉強中 (テテンテンテン MM8f-W3aP)
垢版 |
2022/09/20(火) 02:30:09.48ID:/yxsKNjoM
(スッップ Sdbf-fO9i)って、スレ立てるまでもない質問スレでも
なんか、俺のことを誰かの別垢とか妄想して荒らしてる基地外だったw
以後スルー
0156名無しさん@英語勉強中 (ワッチョイW d7b9-g/36)
垢版 |
2022/09/20(火) 09:09:25.88ID:KwcMjuH70
{(お前の議論)と(日本語)}が意味不明
->(お前の議論が意味不明) and (日本語が意味不明)

は意味をなさない気がする。日本語が意味不明なら日本語での議論が意味不明になるから
前者が後者を包含する。
スップは、日本語が意味不明とだけいえばいいんじゃないか?
0157名無しさん@英語勉強中 (ワッチョイ 1f8a-W3aP)
垢版 |
2022/09/20(火) 09:13:47.79ID:+VDRbKE90
The sandwich was the only good thing going on in my life.

The only good thingがCだとして、その後の
going on in my lifeは何になりますか?副詞句ですか?
句や品詞がわかりません。
0158名無しさん@英語勉強中 (ワッチョイW d7b9-g/36)
垢版 |
2022/09/20(火) 09:15:44.03ID:KwcMjuH70
スップは高卒を馬鹿にしてる。
そうなるとうちの市の市議会議員さんの中には、大学中退の人や、
高卒の人や、中卒の人がいる。それどころか市議会議員さんで大卒の人はほんの
数人だ。どうすればいいんだろ?
0162名無しさん@英語勉強中 (テテンテンテン MM8f-W3aP)
垢版 |
2022/09/20(火) 09:40:05.57ID:/yxsKNjoM
' going on in my life' って副詞句?
'the only good thing'を修飾している形容詞句ではないの?
0163名無しさん@英語勉強中 (テテンテンテン MM8f-W3aP)
垢版 |
2022/09/20(火) 09:41:57.33ID:/yxsKNjoM
>>161
現在分詞であることは間違いない。
形容詞的用法か副詞的用法かのどちらかと
問われれば
形容詞的用法でしょう。
0165名無しさん@英語勉強中 (テテンテンテン MM8f-W3aP)
垢版 |
2022/09/20(火) 09:50:59.93ID:/yxsKNjoM
>>132
(a) Please don't text and walk at the same time.
↑この英文はとても自然だけど

(1) Don't lose hope and get desperate at the same time.
(つまり、2つのことを同時にやってはいけないよ〜ん)

こんな英文あるかな?w
別に同時性を強調しなくてもいいでしょ。
ちょっと時間ずらしてやるのは良いことになってしまうじゃんw
0166名無しさん@英語勉強中 (ワッチョイ 1744-qB2I)
垢版 |
2022/09/20(火) 10:02:49.17ID:5Ymbrstz0
スプーキーがまた朝から暴れててワロタw
0167The OED Loves Me Not (ワッチョイ ff89-W3aP)
垢版 |
2022/09/20(火) 10:13:25.26ID:GGfE4qoZ0
>>165
もちろん、そんな英文があるわけないことくらい、超初心者でもわかるからこそ、
俺はいちいちそんなことを断ったりはしなかった。

>>元の例だと「希望を失う」と「自暴自棄になる」なので,意味的にあり得ないのでその心配はなさそうんですが,

質問者は最初から上のように理解しているから、そんなことをいちいち説明は
しなかったことくらい、馬鹿でもわかるだろう?

Don't AAA and BBB.
なんていう伏字だらけの英文だったらわかりにくいから、あえて今回は
たまたま lose hope と get desperate が出て来たから、仮にもしこの二つを
AAA と BBB に入れたらどうなるかと言っていただけだ。

>>121 でもちゃんと
>>もちろん今回の場合は、質問者もわかっている通り、動詞が
>>lose hope
>>get desperate
>>というものだから、最初の文でも誤解されることがないけどね。

というふうに断り書きをしてるだろ?

ほんともう、このスレは超初心者だらけで、あまりにもレベルが低いから、
中学生レベルにまで戻って説明しないといけないのかよ。
0168The OED Loves Me Not (ワッチョイ ff89-W3aP)
垢版 |
2022/09/20(火) 10:14:56.27ID:GGfE4qoZ0
ほんとレベルが低すぎる。まだ「スレッドを立てるまでもない」のスレの方がましだな。
この文法スレでは、ろくに洋書を読まずに文法書だけを読んで威張りかえってる奴ばかりが
集まるから、レベルが落ちる。
0169名無しさん@英語勉強中 (テテンテンテン MM8f-W3aP)
垢版 |
2022/09/20(火) 10:18:08.75ID:/yxsKNjoM
>>167-168
逆上してて草
お前は「もう一つ、誤解されないための言い方を思いついた。」と書いている。
あんな英文は逆に誤解される言い方だろ、アホ。
0170名無しさん@英語勉強中 (テテンテンテン MM8f-W3aP)
垢版 |
2022/09/20(火) 10:23:03.43ID:/yxsKNjoM
あ、>>133 で気づいて訂正してんのか。それは失礼。

>>167 みたいなレスしなくて
単に、「>>133」で訂正済みって書けばいいのに。
0171名無しさん@英語勉強中 (ワッチョイ 1f8a-W3aP)
垢版 |
2022/09/20(火) 14:07:20.21ID:+VDRbKE90
>>159

有難うございます。
0172名無しさん@英語勉強中 (ワッチョイ 1f8a-W3aP)
垢版 |
2022/09/20(火) 14:08:17.46ID:+VDRbKE90
>>161

有難うございます。
0173名無しさん@英語勉強中 (ワッチョイ 1f8a-W3aP)
垢版 |
2022/09/20(火) 14:11:08.68ID:+VDRbKE90
>>160

訳を有難うございます。
0174名無しさん@英語勉強中 (ワッチョイ 1f8a-W3aP)
垢版 |
2022/09/20(火) 14:12:50.16ID:+VDRbKE90
>>163

前の事を説明しているから、
形容詞的用法なのですね。
どっちかよくわからなかったです。
有難うございます。
0176名無しさん@英語勉強中 (ワッチョイW 579d-dBhJ)
垢版 |
2022/09/20(火) 19:40:28.85ID:PAnZVZ3z0
動詞の文型で challenge 人 to V
があるんだが、ざっくりいうと意味は「誘う、させる」っていうのと逆に「反対する」ってのがある

で前者はたくさん見つかるが後者がなかなか見つけられない

ウィズダムには We challenged the police to arrest him [on his arrest].
があるがこれだけで、まあこれは言えるとして
あまり英英では取り上げてないし意味が曖昧だから
後者はあまり使われないのかなあ
0179名無しさん@英語勉強中 (ワッチョイW 579d-dBhJ)
垢版 |
2022/09/20(火) 20:57:45.89ID:PAnZVZ3z0
おおありがとう
じゃあ根気よく探せば見つけられそうだな
全然見つからん
0180名無しさん@英語勉強中 (ワッチョイW 1f8a-JmM/)
垢版 |
2022/09/20(火) 20:58:40.18ID:K2CLMnfL0
まずなんでそんなの探してんだよ
0181名無しさん@英語勉強中 (ワッチョイ 5793-ret5)
垢版 |
2022/09/20(火) 21:44:29.19ID:/mELXOvT0
てゆーか S V O to不定詞 の文型じゃん
challenge に「異議を唱える」という意味があることさえ知ってればよいだけ
テニスのチャレンジシステムって知ってるよね?
0182名無しさん@英語勉強中 (ワッチョイW 579d-dBhJ)
垢版 |
2022/09/20(火) 21:54:48.33ID:PAnZVZ3z0
いや「させる」とは対照的な意味で
かつ意外と用例が見つからないということが気になったんで
類推は言う通り簡単だと思う
0183名無しさん@英語勉強中 (ワッチョイ ff89-ret5)
垢版 |
2022/09/22(木) 12:45:42.38ID:FdkriJCs0
>>177の「オーレックス」に載っているというその英文の訳がどうしても納得
いかなかったので、WordReference. com で質問してみた。 (´·ω·`)

(リンクを貼ると規制されそうなので、各自で WordReference. com の English
Only の質問の challenged the school to prohibit を参照して下さい)

すると、やはり We challenged the school to prohibit the use of bikes. の意味は、
>>177に示されている
「私たちは学校がオートバイの使用を禁止するのに異議を唱えた」
ではなくて、
「私たちは学校にオートバイの使用を禁止するよう求めた」
の意であった。

やはり自分の解釈が正しかったのでホッとしている。

ちなみに、>>176の We challenged the police to arrest him の意味は
「我々は警察に『彼を逮捕してみろ』と言った・『彼を逮捕してみろ』と言って挑発
した」
ということです。
0185名無しさん@英語勉強中 (ワッチョイW 579d-dBhJ)
垢版 |
2022/09/22(木) 13:46:27.17ID:lZGqmghK0
GWO英和の記述はLDOCEを参照して書かれた
ような気がする

policeの用例は最初検索した時にインドニュースが
上位に来てたからローカルっぽい語法かなと
思ったりしたんだけど謎が深まったね
0187名無しさん@英語勉強中 (ワッチョイW 579d-dBhJ)
垢版 |
2022/09/22(木) 14:04:08.15ID:lZGqmghK0
If you challenge ideas or people, you question their truth, value, or authority.
▶ Democratic leaders have challenged the president to sign the bill.

cobuildのこれは異議を唱える型だなあ
でもこれは単文ならサインすることを
促すと読んじゃう気がする
0189183 (ワッチョイ ff89-ret5)
垢版 |
2022/09/22(木) 14:17:00.99ID:FdkriJCs0
自分はあくまで>>177>>176の英文の意味だけを問題にしていて、
challenge にいろいろな使い方があるのは最初からわかってます。

ここで問題になっているのは「challenge + 人の目的語 + to不定詞」の
パターンの時の解釈。

自分としては、>>184のウィズダムの例文だけがひっかかる。 (´·ω·`)

>>186
The opposition leader challenged the prime minister to call an election.

「野党のトップは首相に対し選挙の実施を要求した・選挙をやってみろと
言った」の意。
0192名無しさん@英語勉強中 (ワッチョイ ff89-ret5)
垢版 |
2022/09/22(木) 14:51:30.05ID:FdkriJCs0
>>190
リンクをありがとう。

(「リンク貼って規制対象になるのは、…特定のサイトだけ」ということだけど、
その特定のサイトが一覧表になって明示されてなければ、やはりいろんなリンクを
貼るのをためらわざるを得ない) (´·ω·`)

>>189を書き込んだ時は入れ違いで>>187を見てなかった。ふーむ。

それにしても、この問題は思ったよりも厄介だった。
WordReference.com の回答者も一人だけで済んでいるということは、異論がない
ように思える。
が、もう少し時間をおいて他に回答が書き込まれるかどうか待ってみよう。
ということで、今日はここまで。
(もっとも、また規制を受けるかもしれない)

>>191
なるほど、その可能性はありますね。
0193😉三年英太郎🌈 ◆3CZBjOt3.Y (ワッチョイW 9f86-dzFe)
垢版 |
2022/09/22(木) 15:09:24.65ID:mlFWiElT0
ある妄想気味の方がURLを貼ると規制されると吹聴してて、それで萎縮してる人がいるみたいね…

アフィ系やステマ系のサイトの、それから1レス内にURLがいくつもあればNGになるけど、24時間でとける。しつこくやったら段々伸びてくけど

何ヶ月も規制されたのは、嫌儲とかのスクリプト荒らし規制に巻き込まれただけ
0194名無しさん@英語勉強中 (ワッチョイW 1f8a-JmM/)
垢版 |
2022/09/22(木) 16:19:03.38ID:xe89n4f10
>>190
この回答要領を得ないな
おい川上、「こいつ役に立たないんで他のネイティブ誰か答えて下さい」と追加で書き込め
0196名無しさん@英語勉強中 (ワッチョイW 9ff0-1eLz)
垢版 |
2022/09/22(木) 16:41:11.81ID:CbrIgmLz0
どっちにも取れる話で状況で判断できることでしょ
選挙実施しているから抗議するわけで、してないから求めるわけでしょ
学校が禁止してるなら抗議で、してないなら求める
簡単な話じゃね?
0197名無しさん@英語勉強中 (ワッチョイW 579d-dBhJ)
垢版 |
2022/09/22(木) 18:03:05.53ID:lZGqmghK0
じゃあ辞書が採用した例文はそれ以上の文脈を欠いているからどちらの意味でもとれるってことかな
50:50?
0198名無しさん@英語勉強中 (ワッチョイW 97fb-g/36)
垢版 |
2022/09/22(木) 19:37:44.43ID:jv2Sx9ZS0
辞書に載っているものは信じよう、というお約束を作ったらダメなの?
辞書にその例文を載せた人は、文脈の中から抜き出してきたわけでしょ。
それを、この文だけではいろんな意味がある。だから例文としては不適切とやるか、
あるいは、文脈の中から取ってこられたので、これこれの意味の例文としては適切と
その項目を書いた人のした抜き出しを考えるか、というと後者の方が生産的では
ないのか?
0200名無しさん@英語勉強中 (ワッチョイW 579d-dBhJ)
垢版 |
2022/09/22(木) 20:04:16.40ID:lZGqmghK0
もっと簡単な話で信じるかどうかじゃないんですよ

動詞の意味解釈に大きな違いが出るような単文を
文型の記述のなかで掲載するのは、
もしそうだとすれば辞書として不適切なんです
0201名無しさん@英語勉強中 (ワッチョイW 9ff0-1eLz)
垢版 |
2022/09/22(木) 20:13:36.25ID:CbrIgmLz0
実際似に起こってることに抗議
起こりそうにないことをもとn
0203The OED Loves Me Not (ワッチョイ ff89-W3aP)
垢版 |
2022/09/23(金) 04:45:37.95ID:KlSHDXkt0
"We challenged the school to prohibit the use of bikes."
この文について Kawakami さんが wordreference.com で質問したスレッドで、
新たにたくさん回答がついたけど、そこにはっきり書いているよ。

例の2冊のイギリスの辞書(そしてそれに追随しただけの日本の辞書2冊)は、編集のときに
削除したり追加したりしているうちに、うっかりと変なところに別の例文を
挿入してしまっただけなんだ。

簡単に言えば辞書が間違っていたということになるが、
そこまで言うと辞書編纂者がかわいそうだ。単なるマイナーな編集ミスに過ぎない。
0204名無しさん@英語勉強中 (ワッチョイW 579d-dBhJ)
垢版 |
2022/09/23(金) 07:08:37.95ID:Li3juclC0
オックスフォードコロケ日本語版にもあったわ
He challenges teachers to rethink their approach.彼は教員たちにやり方を再考するよう求めている

ルポ、ノンフィクション系でもこっちしか見ないから
だいたいこっちなんだろう
to suggest strongly that somebody should do something
0205名無しさん@英語勉強中 (ワッチョイW 7796-i+qs)
垢版 |
2022/09/23(金) 08:47:50.12ID:bdIHYelx0
単にbikeと言ったらオートバイではなく自転車の方を指すのが一般的でしょ
だから通学にbikeを禁止するというのはネイティブにとっては奇妙に聞こえるんじゃないか
英語圏においての一般的な通学ルールや校則については全く分からないけど
0206名無しさん@英語勉強中 (ワッチョイW 7796-i+qs)
垢版 |
2022/09/23(金) 09:02:00.39ID:bdIHYelx0
↓の例文の日本語訳がコリンズ日本語版のページに載ってた
しっかり「異議を唱えた」って書いてあるな

Democratic leaders have challenged the president to sign the bill.
民主党の指導者達は大統領がその法案に署名するのに異議を唱えた.,

https://www.collinsdictionary.com/jp/dictionary/japanese-english/%E7%95%B0%E8%AD%B0%E3%82%92%E5%94%B1%E3%81%88%E3%82%8B
0207The OED Loves Me Not (ワッチョイ ff89-W3aP)
垢版 |
2022/09/23(金) 09:51:04.39ID:KlSHDXkt0
>>almost 70%って70%未満なんですか?

"教えて goo" は、間違い回答に片っ端からベストアンサーがつくので、むしろ
"教えて bad" と名前を変えるべきだと思うが、ともかくこの上の質問でも、
間違い回答にベストアンサーがついている。

まあ、質問者には本当の答えがどれなのか自分で判断できないのが普通だが、
それにしても、僕が何度も口を酸っぱくして言ってあげてることが、回答者にも
質問者にも、まるでわかっていない。質問者にわからなくても仕方ないが、
なぜ僕が何度も言ってあげてることを、完全に無視して僕のあとにまたもや
間違い回答をするのか?しかも何人もの回答者がそんなことをする。

Roderic Ai Camp · 2012 · ‎Political Science
elections the rate of turnout has reached alarming proportions:
from 64 percent in 1991, electoral participation went down to almost 58 percent in 1997 and ...

印刷された書籍の中にも、はっきりと上のように書いている。つまり、
1991 年に 64% だった投票率が、1997年には "almost 58%" にまで落ち込んだと言っている。
この almost 58% はこの場合、58% を少し上回っているに決まっている。

もしも 58% を下回ったのであれば、
(1) went down to a level even below 58%
(2) went to a level even lower than 58%
(3) went down to a level a little lower than 58%
とかいうふうに書くだろう。
0208名無しさん@英語勉強中 (ワッチョイ 579d-ffU0)
垢版 |
2022/09/23(金) 10:19:01.42ID:Li3juclC0
最近きづいたのはcomplicatedlyは全然使われてないってことだ
日本人にとってはいかにも使いやすい言い方に感じるが、実際は違う

ちなBurchfieldは使ってる。
0209名無しさん@英語勉強中 (ワッチョイ b722-E+l9)
垢版 |
2022/09/23(金) 10:26:18.17ID:kQhCxAo+0
challenge+目的語+to不定詞は、次のように考えるとよい

基本は「できるものならやってみなよ」という意味。

① He challenged me to run the Honolulu Marathon.
  彼は、私に「ホノルルマラソンで走れるものなら走ってみなよ」と挑んできた
  ※この場合、「目的語に~にするよう挑む」と意訳するとうまくいく
   
② Democratic leaders have challenged the president to sign the bill.
  民主党の指導者達は大統領に「あなたがその法案に署名できるものならやってみなよ」と挑んだ。
  ※この文は、皮肉的ニュアンスがあり、「目的語が~することに異議を唱える」と意訳するとうまくいく。
  
両者は根本は同じなのだ。両者は、「できるものならやってみなよ」という意味を持ち、あとは文脈からどう意訳するかが変わってくるに過ぎない。
"We challenged the school to prohibit the use of bikes."は、「私たちは、学校に対し『学校が自転車の使用を禁止できるものならやってみてよ』と挑んだ」という意味でしかない。「私たち」が自転車通学に賛成かどうかは、これだけではわからない。
0210183 (ワッチョイ ff89-ret5)
垢版 |
2022/09/23(金) 11:14:02.46ID:I/L4L2Fa0
やれやれ、面倒なことになってしまった。 (´·ω·`)

自分はただ>>177に示されている訳にどうしても納得がいかなかったから、その点を
はっきりさせたかったにすぎない。
WordReference.com でネイティブに質問して、自分と同じ解釈だったから、基本的
に自分の問題は解消したのである。

challenge に意味的に2つの訳し方ができる可能性があるとしても、実際の文章を
読んでいく際には、そこに至るまでの文の流れ、意味の流れがあるので、判断に迷う
ことはまずないであろう。ごく自然に、即座にわかるはずである。
書く場合は、迷ったら別の表現を使えばいいだけである。

要するに実際の読み書きで悩むことはない。自分にとっては何も問題がない。
だから、これ以上 challenge について論じようという気にはならない。
後は、議論したい人におまかせします。 (´·ω·`)

WordReference.com の方も先ほどケリをつけておいた。ネイティブに質問したい
人は各自で、それぞれお好きな流儀でやって下さい。
0211183 (ワッチョイ ff89-ret5)
垢版 |
2022/09/23(金) 11:16:36.72ID:I/L4L2Fa0
>>193
ふーん、そうですか。

でも、結局、5ちゃんねるの運営者、管理者、ひろゆき?が、今回の規制はこれこれ
こういう理由ですとはっきり説明しないのが問題。

大規模規制のたびに運用情報臨時板では、ああだこうだと規制の理由を憶測している。
馬鹿げた話である。 (´·ω·`)
0212名無しさん@英語勉強中 (ワッチョイ 579d-ffU0)
垢版 |
2022/09/23(金) 11:28:50.74ID:Li3juclC0
そのタイプの説明もあると思うけど、捻り出した感がある
invite, dare / question, disputeに語義が大きく別れる際に、someone to Vでは
そのふたつが連続体になるってことだよね。でもその場合、やはり皮肉型反語型の
ほうはあくまでも副次的な存在で実際にも少ない印象がある。

これだけで問題ない気がする。相手に行為を促し、勧めてる。これをquestion, disagree with,
object to, だと普通に読んで良い、とはちょっと思えないなあ。
• [with object and infinitive] invite (someone) to do or say something that one thinks will be
difficult or impossible: I challenge the Minister to deny these accusations. (ODE)
0214三年英太郎 ◆3CZBjOt3.Y (ワッチョイ 9f86-ldZd)
垢版 |
2022/09/23(金) 14:47:01.00ID:8WDo4qMV0
どちらもコアイメージは「挑む」であるが、文型論でいうところの、SVOかSVOCかの違いだな。

1. We challenged [the police] [to arrest him.]
[警察ニ][逮捕することヲ]を挑む→求める

2. We challenged [the police to arrest him.]
[警察が逮捕することヲ]挑む→疑問視する

CGEL&講義でわchallengeは1のSVOOを取るとゆーことなんで、1は意味は保証されてる(と思う)。
2の意味を取る英英が複数あるので、おれ的には揺れがあると見てる。
0217名無しさん@英語勉強中 (ワッチョイW 9fe3-D29d)
垢版 |
2022/09/24(土) 00:22:21.63ID:d9shD/r20
When I watch movies or anime, I often encounter scenes in which the main characters are not able to swim.
このin which のin が理解出来ません。
どなたか解説をお願い致します!
Thank you in advance.
0218名無しさん@英語勉強中 (テテンテンテン MMde-JEMU)
垢版 |
2022/09/24(土) 00:47:28.02ID:Enesl5rtM
>>217
文法書で、関係代名詞と関係副詞のところを見直しましょう
中学生用でもよいので
0219名無しさん@英語勉強中 (テテンテンテン MMde-JEMU)
垢版 |
2022/09/24(土) 01:41:59.06ID:Enesl5rtM
>>214
SVOでも「求める」という意味に
SVOOでも「疑問視する」という意味になり得そうだけど・・・
0220😉三年英太郎🌈 ◆3CZBjOt3.Y (ワッチョイW 1286-jMj8)
垢版 |
2022/09/24(土) 02:17:50.93ID:iy4141rM0
もうちょっと分かりやすく🧐

1. We challenged [the police] [to arrest him.]

We ⚔ the police
SVOOの直接目的語は、広い意味で「贈るもの」である。to arrest him という要求を警察に贈ったのである。

2. We challenged [the police to arrest him.]

We ⚔ the police arrest him
the police は to arrest の主語である
0221😉三年英太郎🌈 ◆3CZBjOt3.Y (ワッチョイW 1286-jMj8)
垢版 |
2022/09/24(土) 02:28:07.34ID:iy4141rM0
つまり、もともとSVOOだったものが、SVOに解釈され始めた、とゆーことではなかろうか🧐

1. I want [you.]
2. I want [you to come here.]

1, 2ともにSVOである。2の目的語は、非定形節 to come here に主語 you が付いている。
0222The OED Loves Me Not (ワッチョイ 1689-JEMU)
垢版 |
2022/09/24(土) 04:24:01.00ID:5esuA3q30
challenge someone to do something について

(1) Collins COBUILD の次のような記述

challenge
[4] [Verb] If you challenge ideas or people, you question their truth, value, or authority

[verb + noun + to-infinitive] Democratic leaders have challenged the president to sign the bill.

(2) Longman の LDOCE の記述

challenge
(1) [Question something]
to refuse to accept that something is right, fair, or legal:
[Examples] a boy with a reputation for challenging the authority of his teachers
challenge a view, an idea, an assumption, etc.

challenge someone to do something:
[Example] I challenge Dr. Carver to deny his involvement!

この二つについては、それぞれの会社に昨日、俺が質問メールを送った。
うまく行けば1か月くらいで返事をくれるだろう。
0223The OED Loves Me Not (ワッチョイ 1689-JEMU)
垢版 |
2022/09/24(土) 04:32:45.43ID:5esuA3q30
なぜ1か月も待とうと考えているかというと、5年前に OALD の記述に編集ミスがあり、
そのせいでその当時に存在していた Alcom という英語質問サイトで日本人たち数人が
その間違い記述を真実だと思い込んでとんでもないデタラメ解釈をどんどん推し進める
一方で、俺がいくら「それはただの編集ミスだ」と言っても聞かないから、仕方なく
俺が Oxford University Press に質問したら、いくら待っても返事が来ず、諦めていた。
しかし1か月も経ってからきちんとした丁寧なメールが来て、「編集ミスでした。
済みません」と言っていたからだ。彼らは世界中からおびただしい問い合わせを
受けていて、それに天手古舞なんだろう。だから1か月くらい待たされても仕方ない。

俺が見るところ、The Oxford English Dictionary の専属スタッフたちは、この22年の
あいだに物凄く忙しくなったみたいで、22年前なら OED Online についての問い合わせに
1時間以内に何度でも返事をくれたのに、数年前からは問い合わせに対する応答は
2週間くらい待たされるようになった。

気を悪くして少し文句を言うと、丁寧に謝ってくれて、どうやらそのスタッフは、
OED Online を格納しているサーバーなのかサイトなのかしらんけど、ともかく
システムを別の場所に移転するにあたって、天手古舞らしいのだ。

というわけで、英英辞典の記載ミスについて問い合わせても、応答までは1か月どころか
数か月も待たされるかもしれないと思っている。
0224名無しさん@英語勉強中 (ワッチョイW 9fe3-D29d)
垢版 |
2022/09/24(土) 09:24:05.61ID:d9shD/r20
>>218
お宅みたいな腐った奴は不要だ
0226The OED Loves Me Not (ワッチョイ 1689-JEMU)
垢版 |
2022/09/24(土) 09:56:11.15ID:5esuA3q30
>>217
(1) 原文: When I watch movies or anime, I often encounter
scenes in which the main characters are not able to swim.

この原文のうち、
scenes in which the main characters are not able to swim
これの意味がわかりにくい(in が何でこんなところにあるのかわからん)のだろ?
それなら、こう考えてはどうか?このフレーズが、もともと次のような形をしていた
と想像してみる。

The main characters are not abe to swim ★in these scenes★.
(主な登場人物は、こういう場面では泳げない(という設定になっている)。)

そしてこの in these scenes を前に持ってくる。
★in these scenes★ the main characters are not able to swim
そして these を消して scenes だけを前に持ってくる。しかしそのまま scenes を前に
持ってくると、空白ができてしまってわけがわからなくなるから、その代わりに
which をあとに(置き土産として)置いておく。
scenes ★in which★ the main characters are not able to swim
   (主な登場人物たちが泳げない(設定になっている)場面)

もし俺のこういう説明で何となくわかったような気がしたら、次に文法書にある
「関係代名詞」の項目、特に「前置詞 + 関係代名詞」の項目の説明を読む。
日本人にとってはこれはわかりにくいけど、まあ徐々にやっていくしかないわな。
0227名無しさん@英語勉強中 (ワッチョイ c793-qHUL)
垢版 |
2022/09/24(土) 10:19:23.00ID:TP98MGoE0
>>217
Sが泳げない【場面】/【scenes】in which the main characters are not able to swim.

先行詞【scenes】を修飾する形容詞節の元の文は
the main characters are not able to swim in which.

通常は関係代名詞を形容詞節の先頭にもってくる
which the main characters are not able to swim in.

形容詞節内での「前置詞+関係代名詞」という副詞句を作る場合、
関係性をわかりやすくする為に、副詞句ごと先頭に置く場合がある。
in which the main characters are not able to swim.
0228名無しさん@英語勉強中 (ワッチョイ 279d-SHZG)
垢版 |
2022/09/24(土) 10:28:58.10ID:y/ky1eHt0
in whichといえばGraham SwiftのWaterlandのこの一文が気になってる。
前後含めグーグルブックスでも見れる

To his library in Cable House he will add volumes in which are contained what human knowledge,
in the 1820s, has to offer on the mystery of the human mind.

in which以下が倒置になっていると思うけど、そのばあいwhatを受けるis containedとすべき
ところを、先行するvolumesに引っ張られてare containedにしてしまったのかなあと
そもそも受け身や進行形の倒置はわかりにくいし
0229The OED Loves Me Not (ワッチョイ 1689-JEMU)
垢版 |
2022/09/24(土) 10:51:58.52ID:5esuA3q30
>>228
あなたの言う通り、確かに先行する volumes に引っ張られて is が are になってしまっているね。
それがそのまま、編集者たちによる訂正も受けないで出版され続けているようだね。
are contained を is contained にして検索すると、何も出てこない。間違った方だけが
一般に普及しているようだ。
0230The OED Loves Me Not (ワッチョイ 1689-JEMU)
垢版 |
2022/09/24(土) 11:07:35.83ID:5esuA3q30
in which は現代ではなるべく避けようとしているみたいだが、それでもときどきは
出てくるね。そして、"in whch to [do]" はそのまま今でも大いに使われているように
見える。便利なんだろうね。それに、ここで in which を避けようとしても、
[先行詞] to [do] in
なんていうふうに which を前に持ってくるのを避けようとしても無理だから、
[先行詞] in which to [do]
をそのまま温存しないわけにはいかないんだろうね。
0232The OED Loves Me Not (ワッチョイ 1689-JEMU)
垢版 |
2022/09/24(土) 13:12:29.95ID:5esuA3q30
>>228 の英文をきちんと理解できない人のために、一応は説明しておこうか。

To his library in Cable House
   Cable House にある彼の蔵書に
he will add
   彼は加えるつもりだ (add volumes to his library というふうに続く)
volumes
   (add の目的語。「数々の本を」という意味)
in which are contained what human knowledge, in the 1820s, has to offer on the mystery of the human mind.
   volumes という先行詞に続く関係節。
   俺たちのようにろくに英語のできない人間にとって分かりやすい語順にすると、
   in which what <<< human knowledge, in the 1820s, has to offer on the mystery of the human mind >>> are contained
     この上の << >> で囲んだところは、what のあとの主語。それに続く動詞は are contained。
0233The OED Loves Me Not (ワッチョイ 1689-JEMU)
垢版 |
2022/09/24(土) 13:15:44.93ID:5esuA3q30
>>228 の英文の骨子は、
To his library in Cable House he will add volumes (このあとは volumes に続く関係節).
であって、それをわかりやすい語順にすると
He will add volumes to his library in Cable House.
   彼は、Cable House にある自分の書斎に(これこれこういう)書籍の数々を加えるつもりだ。
0234名無しさん@英語勉強中 (スフッ Sd32-ZJWq)
垢版 |
2022/09/24(土) 13:23:59.97ID:c1PjdkiNd
>>229
whatが単数を受けるのか複数を受けるのか、つまりofferの目的語がthat whichなのかthose whichなのか、どちらと言えるかは話者次第であって確定していないから、間違いとは言えないよ、バカな半可通で文法音痴のアスペ爺w
0235😉三年英太郎🌈 ◆3CZBjOt3.Y (ワッチョイW 1286-jMj8)
垢版 |
2022/09/24(土) 13:33:52.50ID:iy4141rM0
>>225
SVO であってます🥸
以下、さだおっちの分析

1.知的意味を変えずに、for説に書き換えられる

I want [for you to come here]

2. OCのOとされる部分に虚辞をおける。(虚辞だけではOにはなりません🙅🏻‍♂)

I don't want [there to be another war].
0236The OED Loves Me Not (ワッチョイ 1689-JEMU)
垢版 |
2022/09/24(土) 13:38:15.44ID:5esuA3q30
>>228 の原文の一部の語順を変えたもの

what <<< human knowledge, in the 1820s, has to offer on the mystery of the human mind >>> are contained

この what S V の部分を、いちいち複数で受ける必要はない。
what human knowledge... ★IS★ contained
で十分だ。
「1820年代に人類の知識が提供できるもの」と言えば、普通は複数じゃなくて
単数のものを想像する。たとえば truth, beauty, tragedy, comedy, human suffering など。
0237😉三年英太郎🌈 ◆3CZBjOt3.Y (ワッチョイW 1286-jMj8)
垢版 |
2022/09/24(土) 13:42:53.93ID:iy4141rM0
What

(1)単複呼応 what 節は単数扱いが一般的で, 主語として用いられる場合には[動]は単数形が普通だが, 補語が複数形であれば, それにひかれて複数形になる場合もある
👉🏻What we need most is [are] more houses.
最も必要とされているのはさらに多くの住宅である.
0238😉三年英太郎🌈 ◆3CZBjOt3.Y (ワッチョイW 1286-jMj8)
垢版 |
2022/09/24(土) 13:49:52.25ID:iy4141rM0
一般的で、ということは、当然一般的でない場合もあるだろう。文学なんて、そーゆーものを殊更利用するものなのに

どこまでが障害によるものなのか分からないのでアンタッチャブルな存在であるが、自分もわりと間違えてるのに(この一週間で最低3回)、どーして他人の間違いだけに異常に厳しいのか
0240名無しさん@英語勉強中 (ワッチョイ 12e2-mIyF)
垢版 |
2022/09/24(土) 14:08:17.59ID:82rSJeXY0
CGELのp.1193にある。wantなどの場合、

You are not wanted to study English by me.

みたいな受動態が作れないので目的語ではない。他に(can't) bear、desire、hate、like、love、prefer、want、wishなどもこのパターン。

preferの場合はforを付けるパターンもある。あっても無くても不定句の主語扱いなのでSVOになる(Oは不定詞)。

・Jack prefers his wife to drive the truck.
・Jack prefers for his wife to drive the truck.

・With all our hearts, we wished for the rain to stop.
0241名無しさん@英語勉強中 (ワイーワ2W FF6a-QymP)
垢版 |
2022/09/24(土) 14:11:41.58ID:FnkZbTgDF
>>235
I want you to come here.は第五文型の基本で載ってますよ。to不定詞は補語です。
0243The OED Loves Me Not @ Fucking old with Asperger's syndrome (ワッチョイ 1689-JEMU)
垢版 |
2022/09/24(土) 14:48:57.65ID:5esuA3q30
>>239
(1) whatの関係形容詞で訳は『全ての』ですか?

すまんけど、「関係形容詞」なんてものは、今回の英文には出てこないと思う。
何度もくどくど解説したけど、in which の which は volumes にかかる。
"what S V" は in which のあとの節の中の主語。"are contained" は述語。

(2) この場合、倒置を使う効果とは何でしょうか?

are contained という述語が短すぎて、主語があまりに長いから。
0244名無しさん@英語勉強中 (オッペケ Sr47-9B5+)
垢版 |
2022/09/24(土) 14:56:47.21ID:9yAUTT4ur
whatは関係代名詞でoffer X on だよ
0245😉三年英太郎🌈 ◆3CZBjOt3.Y (ワッチョイ 1286-XjGR)
垢版 |
2022/09/24(土) 15:07:31.82ID:iy4141rM0
オーレックス Planet Board 94

What [is] needed [is] books.

2つの [...] 内は is か are かという調査が載ってる。
一つ目は is が普通・一般的とされているにもかかわらず、それでも20%が are と答えてる。

(2つ目は books に引かれて、76%が are)
0246名無しさん@英語勉強中 (ワッチョイ c793-i8Eu)
垢版 |
2022/09/24(土) 15:18:30.43ID:k7Eavu4z0
I want you to come here.が第五文型であると仮定。
to come here は目的格補語だから
You are to come here.
と書けなければならないが、意味的に書けない。
よって仮定は間違い。

ちなみに俺が持ってる文法書だと基本5文型外に分類されている。
あらゆる英文を5つの文型だけに収めるのは無理ってことやな。
0248名無しさん@英語勉強中 (スップ Sd52-ZJWq)
垢版 |
2022/09/24(土) 15:51:11.87ID:kJNmWttqd
>>236
それはお前がそうとしか想像出来ないだけ
お前の「ぼくの考えた文法」なんて誰も求めてはいない
間違いだなんて言うのはお前が端的に文法に無知だった証拠なんだよ、アスペ爺さんw
0250名無しさん@英語勉強中 (スフッ Sd32-QymP)
垢版 |
2022/09/24(土) 16:29:56.28ID:iKPWE1Nod
>>246
その説明は第二文型の補語が名詞の場合ですね。第五文型の場合、目的語と補語はイコールだけでなく主語述語動詞の関係になる場合の2通り有ります。この文oがcする状態をsが積極的に求めている第五文型の典型です。youとto comeを繋いでいるのはwantです。意味を考えるとそうなってます。使役的な関係といいますか。文法的には第三文型でむりやり解釈もできるけど、それは変な感覚だと思います。
0251名無しさん@英語勉強中 (スフッ Sd32-QymP)
垢版 |
2022/09/24(土) 16:33:28.50ID:iKPWE1Nod
>>243
すいません。1は意味がわかりません。in whichは先ほどの解説で理解しました。

2は理解しました。有難うございます。
0252名無しさん@英語勉強中 (スフッ Sd32-QymP)
垢版 |
2022/09/24(土) 16:35:00.13ID:iKPWE1Nod
>>244
おお、家に帰ったら辞書で調べてみます。
有難うございます。
0254😉三年英太郎🌈 ◆3CZBjOt3.Y (ワッチョイW 1286-jMj8)
垢版 |
2022/09/24(土) 16:36:51.02ID:iy4141rM0
そもそも人類の知識が提供できるものは単数ってのがおかしい。たとえばfactsは? beauty
とtruthはそれぞれ不可算としても、2つあれば複数じゃん。

👉🏻 Beauty and truth are the same thing.
0258名無しさん@英語勉強中 (ワッチョイ 6b22-tX/F)
垢版 |
2022/09/24(土) 19:15:04.78ID:6Zd4Md3r0
今回の[are]の問題は、in whichは関係なくて、次の文だけを見ればよい。

What [主語] has to offer on the mystery of the human mind are contained in the volumes.
(人知が人間の心の神秘について提供するために持っているものが、そのvolumesの中に収められている。)

「人知が人間の心の神秘について提供するために持っているもの」ってひとつだけのわけないでしょ!

このwhatはthe things which(複数)で解釈すべきなのだから、areで受けるのが当然だよね
0259名無しさん@英語勉強中 (ワッチョイW 279d-9B5+)
垢版 |
2022/09/24(土) 19:16:53.96ID:y/ky1eHt0
それはto以下がrightの形容だからchallengeとは違うけど
文型表示svo to doの問題じゃない

でもthat節についてはそうだね
0262名無しさん@英語勉強中 (スププ Sd32-ZJWq)
垢版 |
2022/09/24(土) 19:38:03.94ID:kSFJ3rNmd
>>258
バカ高卒くんに今日はサービスで教えてあげる

ヒント has O to offerではないw

>>260
バカ高卒くんに今日は特別サービスで教えてあげる

このwhatが単数であるべきか複数であるべきかを文法規則から確定することはできない
なぜならwhatは話者の頭の中にあるから
逆の意味でアスペ爺と同じ誤解をしている

言及できることは
areをアスペ爺が言うように「間違い」だと文法規則としても意味論としても絶対に言えない、ということだけだ

わかるかな?w
0266名無しさん@英語勉強中 (スププ Sd32-ZJWq)
垢版 |
2022/09/24(土) 19:45:59.20ID:kSFJ3rNmd
しつこいがまとめる
isが正しいのか、areが正しいのか、この文について、文法規則としても意味論としても正誤の決定など出来ない、ということ

あ、ひょっとして!今気づいたけどバカ高卒くんはin whichのvolumesが複数だから、areじゃなきゃ駄目だと思っているわけか!

もうお前は二度と回答すんなや?wwwwww
0267名無しさん@英語勉強中 (ワッチョイ 6b22-tX/F)
垢版 |
2022/09/24(土) 19:49:24.86ID:6Zd4Md3r0
>>262
残念だが人違いだよ。もしかして「have to(~しなければならない)」で解釈したの? よくある間違いだ。 
じゃあ俺も君にサービスで教えてあげる。

What [人知] has to offer on the mystery of the human mind
これは、[人知] has the things to offer~の目的語が先行詞になったものなんだよ。 これぐらいは高校生でも知ってるよ。

さらに、「人知が人間の心の神秘について提供するために持っているもの」ってひとつの訳がないだろ。
さらに、著者がareを使ってるんだから、複数で確定してるだろ。

こんなことも分からないで人をバカ呼ばわりするって、君のほうこそバカだと思うよ
0268名無しさん@英語勉強中 (ワッチョイ 6b22-tX/F)
垢版 |
2022/09/24(土) 19:55:14.99ID:6Zd4Md3r0
著者がareを使ってるんだから、the things which(複数)に決定しているだろ。

このareを否定することはできない以上、正しい回答は「areで問題ない。それを否定する意見は間違い」となる。

この文のareは正しいか間違っているかという質問で、著者が複数を念頭において意図的に複数areを使ってるのに、「どちらでもいい」って回答は的はずれ。
0269名無しさん@英語勉強中 (ワッチョイW c793-QymP)
垢版 |
2022/09/24(土) 20:46:50.43ID:TP98MGoE0
>>252ですがなぜだか荒れてますね。民度と知能がかなり低いお方がおられる様で^_^ こんな所で時間の浪費も出来ませんので失礼させてもらいます。紳士的に対応してくれた方々には感謝しております。でわ
0270名無しさん@英語勉強中 (ワッチョイ 279d-SHZG)
垢版 |
2022/09/24(土) 21:07:04.43ID:y/ky1eHt0
A recent study found that unique among all ethnic groups in the United States, the life
expectancy of working-class white folks is going down. (Hillbilly Elegy)

ずっと放置してた用例で最近たまたま発見があったのがこのunique among
A recent study found that, unique...としたほうがいいのではないかとも思うが、
それはともかく、最近、unique amongがOxford Learner's Dictionary of Academic English
でわざわざ太字句で立項されているのを見つけて驚いた
unique among sb/sth
Birds are unique among vertebrates in that biparental care is the norm.

例文がOALDファミリーと被ってない、全部新しい、というのが
ウリだけど、細かいところもよく編集されているなと思った
0271The OED Loves Me Not (ワッチョイ 1689-JEMU)
垢版 |
2022/09/24(土) 21:38:13.31ID:5esuA3q30
>>270
>>A recent study found that, unique...としたほうがいいのではないか

それは僕も昔からよく思ってた。しかし最近は考えを改めている。というのも、
A recent study found that, unique... だったら、「A recent study は that (それ) を見出した」
とうっかり読み違えることがある。もちろんそのあとを読めばわかってくるけど、
それだったらなるべく最初のうちから構文が分かった方がいいので、
やっぱりここでは comma なしの方がいいような気がしている。

それはともかく、Academic English の辞書の存在は知っていたけど、academic と
言うからには専門用語ばっかり並んでいるんだろうから、専門用語辞典なら
他にいくらでもあるからいらんわいと思ってた。それがそんなにいいとは、
まったく想像もしなかった。さっそく買ってみる。ありがとう。

しなかった。
0276名無しさん@英語勉強中 (ワッチョイ 6b22-tX/F)
垢版 |
2022/09/24(土) 22:23:46.50ID:6Zd4Md3r0
>>272-273
反論できなくて、悪口しか言えないようだね。君は、もっと自分の頭と性格の悪さを自覚したほうがいいと思うよ。かわいそうな人だ。掲示板の雰囲気が悪くなるのでもう出てこないでね。
0282名無しさん@英語勉強中 (スププ Sd32-ZJWq)
垢版 |
2022/09/24(土) 22:58:08.37ID:kz1nUncwd
バカ高卒くんはモバイルも複数回線の契約がある
アハモを含むドコモ系、auKDDI系、MVNO
このバカは5ちゃんを荒らすことに命をかけてる(といってもスクリプトは一行も書けないから全てが手動のバカw)
0286名無しさん@英語勉強中 (アウアウアー Sa6e-D29d)
垢版 |
2022/09/26(月) 07:43:02.00ID:CvHtHRKTa
>>226
>>227

解説ありがとうございます。
御礼が遅くなってすみません。
0287名無しさん@英語勉強中
垢版 |
2022/09/26(月) 10:00:13.53ID:+dpvmtBe0
ええんやで!
0288名無しさん@英語勉強中 (ワッチョイW 9fe3-D29d)
垢版 |
2022/09/26(月) 20:34:59.98ID:uHAf34Ci0
>>226
In these scenes とは、
movies and animeのことを指すのでしょうか?
未だ理解が出来ません。(汗)
0290The OED Loves Me Not @ Fucking old with Asperger's syndrome (ワッチョイ 1689-JEMU)
垢版 |
2022/09/27(火) 04:03:44.91ID:RncLGZ5o0
さらに別の回答者が、日本の英字新聞で condolence diplomacy という言葉をカッコつき (" ")で
使っていると言っている。

Japan's Prime Minister Fumio Kishida on Monday kicked off "condolence diplomacy" with foreign dignitaries scheduled to attend the state funeral for former leader Shinzo Abe, trying to boost ties with other nations amid an unstable regional security environment.
(The Mainichi, September 26, 2022)
0292名無しさん@英語勉強中 (ワッチョイ c793-ZdqV)
垢版 |
2022/09/27(火) 21:07:05.68ID:NDSyNpXz0
日本の首相岸田文雄は、月曜日、不安定な地域安全保障環境の真っただ中にあって他国との絆を強めようと、
元リーダ安倍晋三の国葬へ参加予定の外国要人達と"お悔み外交"を開始した。
0294名無しさん@英語勉強中 (ワッチョイW 279d-9B5+)
垢版 |
2022/09/27(火) 21:33:50.86ID:ftg+L42h0
tokyo viceの邦訳は著者本人がやってると思うが
きちんとした日本語だが文体に違和感ある
分析できてないが内容がしっかりしていれば
ある種の味わいにもなりそう
0295名無しさん@英語勉強中 (テテンテンテン MMde-JEMU)
垢版 |
2022/09/28(水) 03:10:06.80ID:Mr7Dza0YM
>>262

>>ヒントではないw

いや、 has O to offer で合ってるだろ。
「ヒント」とか言って間違ってる奴、超ハズイな
0296名無しさん@英語勉強中 (テテンテンテン MMde-JEMU)
垢版 |
2022/09/28(水) 03:17:17.01ID:Mr7Dza0YM
あ、 >>267 でもう指摘済みの人がいたね。
おっしゃる通り、多分、have to do と解釈したんだろうね。

誰から構わず、高卒馬鹿って喚いてる基地外だからスルーするのがベストなんだけどね。
0297名無しさん@英語勉強中 (テテンテンテン MMde-JEMU)
垢版 |
2022/09/28(水) 03:18:56.43ID:Mr7Dza0YM
晒し上げ
↓苦笑

262名無しさん@英語勉強中 (スププ Sd32-ZJWq)2022/09/24(土) 19:38:03.94ID:kSFJ3rNmd
>>258
バカ高卒くんに今日はサービスで教えてあげる

ヒント has O to offerではないw
0298名無しさん@英語勉強中 (スプッッ Sd52-P12Y)
垢版 |
2022/09/28(水) 08:05:34.08ID:UAeVcbrId
>>295

has to offer O …@
とも
has O to offer …A
とも
どちらにも解釈できる
あとは文脈次第
俺は前後の脈絡はわからないこの一文のなかで@の解釈をとった

しかしこの一連の議論は>>234に俺が書いた点が本筋の話だ
0301名無しさん@英語勉強中 (テテンテンテン MMde-JEMU)
垢版 |
2022/09/28(水) 09:06:38.71ID:Mr7Dza0YM
>>299
別に怒ってはない。
人のことを、馬鹿だのなんだの言ってるくせに
テメーも馬鹿だろと、嘲っているだけだが 苦笑
顔を真っ赤にして怒ってるのは、お前だろw
0302名無しさん@英語勉強中 (テテンテンテン MMde-JEMU)
垢版 |
2022/09/28(水) 09:08:29.50ID:Mr7Dza0YM
>>300
お前らがどんなことをレスしてたかなんて
一々チェックしてないので知らんがな。
0303名無しさん@英語勉強中 (テテンテンテン MMde-JEMU)
垢版 |
2022/09/28(水) 09:12:43.10ID:Mr7Dza0YM
>>298
なんか、顔を真っ赤にして言い訳してるけど
あの一文だけを取り出したとしても
@よりAの方が自然な解釈としか思えない。

少なくとも
Aと解釈してる人に
それは間違っていて@が正しいなんて指摘できる根拠は皆無だ。

それとも何か根拠あるのか?
あるなら言ってみろよ、中卒。
0304名無しさん@英語勉強中 (スッップ Sd32-P12Y)
垢版 |
2022/09/28(水) 11:52:11.83ID:s27KwyZZd
>>301
夜中の三連投とか、充分怒ってると思うけどwww

高卒が頭にきたんなら謝るよ
高卒一般をバカにしたいわけじゃなくて「バカ高卒くん」と他の板で呼ばれてる単体の粘着を指してるだけだからw
0305名無しさん@英語勉強中 (スッップ Sd32-P12Y)
垢版 |
2022/09/28(水) 11:58:29.99ID:s27KwyZZd
>>303
お前が>>295で「間違い」とか根拠なく言うとるがなw

そもそもこの文のwhatが単数か複数かということにin whichやらvolumesが複数だとか全く関係ないだろマヌケ、という話のおまけだよ流れ読んでこい
関係無いお前が何日も経った夜中に議論としては瑣末なことに根拠なく噛み付いてきてることが間違い
0306名無しさん@英語勉強中 (テテンテンテン MMde-JEMU)
垢版 |
2022/09/28(水) 14:23:39.16ID:5HaSpvwXM
>>304-305
泣きながら言い訳してるけど
結局
Aと解釈してる人に
それは間違っていて@が正しいなんて指摘できる根拠は皆無ということでいいな?中卒。
0307名無しさん@英語勉強中 (テテンテンテン MMde-JEMU)
垢版 |
2022/09/28(水) 14:39:02.14ID:5HaSpvwXM
主要な議題ではないという言い訳も醜いからしない方がいい
そうであらうがなかろうが、お前がわざわざ話題にしたのに
今更、「俺の指摘が間違いだろうが、主要な議題ではないからスルーしろ」とか
カッコワル
指摘が間違ってたと認識したらなら、素直に訂正しろ、中卒
0309名無しさん@英語勉強中 (スップ Sd52-P12Y)
垢版 |
2022/09/28(水) 14:55:30.16ID:ByjAmG2Zd
>>307
わざわざ話題にしたって、>>262の中の一行だけですよね?w
しかも俺の指摘が間違いとか何で俺が考えてなきゃいけないんだよwww

お前がいちゃもん以外に結局何を言いたいのか全くわからない
0310名無しさん@英語勉強中 (テテンテンテン MMde-JEMU)
垢版 |
2022/09/28(水) 14:57:25.10ID:5HaSpvwXM
つーか、areについてだって
(ワッチョイ 6b22-tX/F)って、areになってる理由は
whatを著者が複数のものと認識してるからって最初から言ってるのに

馬鹿中卒の読解力が中卒レベルだから
勘違いな批判してるだけやんwww
0314名無しさん@英語勉強中 (テテンテンテン MMde-JEMU)
垢版 |
2022/09/28(水) 15:05:28.98ID:5HaSpvwXM
>>309
>>しかも俺の指摘が間違いとか何で俺が考えてなきゃいけないんだよwww

間違いじゃないと、今でも思ってるわけ?
苦笑

>>308
have O to offer と解釈する方が自然だから。
少なくとも、この解釈が間違いだとする道理が全くない。
それを、お前が間違いだといい始めたのだから
お前に立証責任がある。
早く、間違いだと思った根拠を言えよ、馬鹿
0315名無しさん@英語勉強中 (テテンテンテン MMde-JEMU)
垢版 |
2022/09/28(水) 15:06:36.71ID:5HaSpvwXM
>>312
違うというなら
その証拠となるレスを挙げてみろ、馬鹿
0316名無しさん@英語勉強中 (テテンテンテン MMde-JEMU)
垢版 |
2022/09/28(水) 15:07:57.46ID:5HaSpvwXM
>>313
意味論としても成立してないとも全く言ってないなら

>>ヒント has O to offerではないw

って、どんな意味なんだよ、基地外www
言い訳がアホ過ぎwwww
0318名無しさん@英語勉強中 (テテンテンテン MMde-JEMU)
垢版 |
2022/09/28(水) 15:09:19.67ID:5HaSpvwXM
>>317
感想じゃなくて事実です
中卒には無理かなw
0319名無しさん@英語勉強中 (テテンテンテン MMde-JEMU)
垢版 |
2022/09/28(水) 15:10:52.77ID:5HaSpvwXM
>>317
has to offer O という解釈で訳してみてw
低学歴www
0322名無しさん@英語勉強中 (テテンテンテン MMde-JEMU)
垢版 |
2022/09/28(水) 15:15:04.53ID:5HaSpvwXM
>>320
どのスレだよ。
このスレから逃走したいなら
お前がリンクくらい貼れ、役立たず。
0323名無しさん@英語勉強中 (テテンテンテン MMde-JEMU)
垢版 |
2022/09/28(水) 15:18:18.76ID:5HaSpvwXM
>>321
お前は「スプーキー婆」とは別人とのことだけど
頭の悪さと粘着の仕方が非常に似てるんだよな
ホントに別人?
0325名無しさん@英語勉強中 (テテンテンテン MMde-JEMU)
垢版 |
2022/09/28(水) 15:19:45.92ID:5HaSpvwXM
>>324
移動してやるから、まずお前がそこで訳を拾う披露な
低学歴
0326名無しさん@英語勉強中 (テテンテンテン MMde-JEMU)
垢版 |
2022/09/28(水) 15:21:15.07ID:5HaSpvwXM
訂正
×まずお前がそこで訳を拾う披露な
〇まずお前がそこで訳を披露な

以降、そちらで
0328名無しさん@英語勉強中 (テテンテンテン MMde-JEMU)
垢版 |
2022/09/28(水) 15:22:07.33ID:5HaSpvwXM
>>327
お前がまずそっちで訳を書けって言ってんだろ
カス
0330名無しさん@英語勉強中 (テテンテンテン MMde-JEMU)
垢版 |
2022/09/28(水) 15:25:43.53ID:5HaSpvwXM
>>329
訳してみろと言ってるだけなんだが・・・
結局、泣きながら逃走かよ 苦笑
0333名無しさん@英語勉強中 (スフッ Sd32-P12Y)
垢版 |
2022/09/28(水) 19:11:34.14ID:oSPjAH+7d
>>332
バカ高卒くんには何にもわからないんだから余計な口を挟まんでよろしいw
それより君のそのワッチョイ、末尾にBKつまりバカって大文字で出ているのなwwwwww
0335名無しさん@英語勉強中 (ワッチョイ 92f0-r4yT)
垢版 |
2022/09/28(水) 19:43:04.21ID:fhwmS5QE0
スップうざい
0341名無しさん@英語勉強中 (テテンテンテン MMde-JEMU)
垢版 |
2022/09/29(木) 02:59:40.78ID:xGqZLzw4M
>>338-340
まるで小学生同士が談笑してるようだなw
0342The OED Loves Me Not @ Fucking old with Asperger's syndrome (ワッチョイ 1689-JEMU)
垢版 |
2022/09/29(木) 09:57:26.62ID:+jdcs43U0
>>222 にて言ったように、たとえば "I challenge Dr. Carver to deny his involvement." などという
言い回しの意味について、それに似た例文を "to question the authority of someone" に似た
定義文のすぐ下に置いている二つの英英辞典の出版社である Longman と Collins との
2社にメールで質問した。

Collins からは返事がまだ来ていないが、Longman (実際には Pearson の会社) からは
あれから2回、返事が来た。一回目の返事では俺は納得できなかったので、
改めて質問しなおしたら、今さっき2回目の返事が来た。

(1) 一回目の返事: その定義文のところに "I challenge Dr. Carver to deny his involvement."
という例文を示したが、それは編集ミスではなくて、それで正しい。

次に二回目の質疑応答をこの下に示す。
0343The OED Loves Me Not @ Fucking old with Asperger's syndrome (ワッチョイ 1689-JEMU)
垢版 |
2022/09/29(木) 10:03:35.23ID:+jdcs43U0
>>222 に関連して
もともと Longman (Pearson) には俺は英語だけで質問したが、回答が日本人らしき
人(あるいは日本語の達者な中国人かもしれないと思いたくなるような名前でもあった)
が日本語で、担当者からの伝言を俺に伝えて来た。俺はそれに対して、
「もしも担当者が英語で回答してくれたのであれば、その英文をそのまま私に
送ってくれ。あいだでそれを日本語で私に伝えられると、私は担当者の真意を
つかみにくい」と言ったら、2回目には担当者による英文の返事が届いた。
0344The OED Loves Me Not @ Fucking old with Asperger's syndrome (ワッチョイ 1689-JEMU)
垢版 |
2022/09/29(木) 10:05:11.07ID:+jdcs43U0
>>222 に関して

(1) まず、2回目の俺からの Longman (Pearson) への追加質問

Thank you very much for your reply. I understand that you're saying that the example sentence "I challenge Dr. Carver to deny his involvement" is in its proper place in the dictionary in that particular context.

***********************************

For your reference, here's the paragraph I had quoted in my previous email:

Excerpts from the Longman Dictionary of Contemporary English for Advanced Learners (6th Edition, 2014)
challenge
(1) [Question something] to refuse to accept that something is right, fair, or legal:
[Examples] a boy with a reputation for challenging the authority of his teachers
challenge a view, an idea, an assumption, etc. challenge someone to do something:
[Example] I challenge Dr. Carver to deny his involvement!

************************

I had been wondering about this issue because this construction "to challenge someone to do something" (but NOT "challenge someone's authority or idea," which I already understand fully) in that particular place in the dictionary (namely, meaning "question the authority of someone") because, as far as I know, only two dictionaries from the English-speaking world has indicated it so far, that is, your Longman dictionary and Collins COBUILD.

< 俺からの質問メールは、この下に続く >
0345The OED Loves Me Not @ Fucking old with Asperger's syndrome (ワッチョイ 1689-JEMU)
垢版 |
2022/09/29(木) 10:05:52.93ID:+jdcs43U0
俺からの2度目の質問メールの続き

Now, please allow me to bother you again by furthering my question.

Does this sentence "I challenge Dr. Carver to deny his involvement" mean something along the lines of:
"I question the authority of Dr. Carver when he is saying that he is denying his involvement."
or, to put it otherwise,
"I don't think that Dr. Carver got involved in this particular incident (or happenings, etc.)."

Sorry for my clumsy wording. Could you please paraphrase this example sentence "I challenge Dr. Carver to deny his involvement"? I do fully understand the meaning of the construction "I challenge his authority" by the way.

Thank you and sorry for bothering you again.

< 俺の実名 >
0346The OED Loves Me Not @ Fucking old with Asperger's syndrome (ワッチョイ 1689-JEMU)
垢版 |
2022/09/29(木) 10:08:43.33ID:+jdcs43U0
以上のような俺からの2度目の質問メールに対して、担当者らしき人が次のような
返事をくれた。

****** 以下は引用 *****

Yes, your understanding is correct: "I question the authority of Dr. Carver when he is saying
that he is denying his involvement." But rather than specifically challenging authority
in the example sentence, the person is asking Dr。 Carver to prove something,
i.e. challenge him to deny his involvement.

1 QUESTION SOMETHING to refuse to accept that something is right, fair, or legal
a boy with a reputation for challenging the authority of his teachers
a. challenge a view/an idea/an assumption etc
Viewpoints such as these are strongly challenged by environmentalists.
They went to the High Court to challenge the decision.
b. challenge somebody to do something
I challenge Dr. Carver to deny his involvement!


Explanation for 1b:
(One assumes that) Dr. Carver was involved in something not right, unfair or illegal.
I challenge Dr. Carver to deny his involvement
= I refuse to accept that Dr Carver was not involved
Dr. Carver needs to prove (by answering the challenge) that he was not involved;
i.e. deny his involvement
0347The OED Loves Me Not @ Fucking old with Asperger's syndrome (ワッチョイ 1689-JEMU)
垢版 |
2022/09/29(木) 10:17:02.71ID:+jdcs43U0
>>346 に示した担当者からの返事の主な部分は次の通り。

>>I challenge Dr. Carver to deny his involvement
= I refuse to accept that Dr Carver was not involved
Dr. Carver needs to prove (by answering the challenge) that he was not involved;
i.e. deny his involvement

ということは、
I challenge you to jump from the roof!
と子供が別の子供に言っている言い回しとは、challenge の意味合いに関しては
さほど大きな違いはないと思う。「やれるもんなら、やってみろ。できもしないくせに」
という感じ。その子供は、相手の子供にそんな勇気はないと思っている。
どうせ飛び降りたりはしないと思っている。

今回の Dr. Carver の例文についても、本質的にはさほど大きな違いはないと思う。
つまり、「どうせその事件に関わってたんだろうが。関わってないって言うんなら、
証明してみろよ。どうせできもしないんだから」と話者は思っている。

だから chalenge someone to do AAA は、子供が言う台詞のときも、Dr. Carver に対して
言うときの台詞のときにも、本質的には、
「どうせ AAA なんてできないだろ?できるって言うんなら、やってみろよ。」
と心の中で思っているときの台詞のようだ。

それにしても、なんで今回はあんなに俺も他のみんなも混乱したんだろうか?
忘れてしまった。

俺の理解にもし変なところがあれば、指摘してほしい。
0349The OED Loves Me Not @ Fucking old with Asperger's syndrome (ワッチョイ 1689-JEMU)
垢版 |
2022/09/29(木) 10:30:22.18ID:+jdcs43U0
>>206
>>Democratic leaders have challenged the president to sign the bill.

ここでも、「"sign the bill" できるもんならやってみろ。できもしない癖に」と
democratic leaders は思っているということになるのだろうか。

>>204
>>He challenges teachers to rethink their approach.彼は教員たちにやり方を再考するよう求めている

この上の例文では「やれるもんならやってみろ」という感じではなくて、
「やってくれ」と単に求めているだけ、という感じだろうか?
0350The OED Loves Me Not @ Fucking old with Asperger's syndrome (ワッチョイ 1689-JEMU)
垢版 |
2022/09/29(木) 10:32:59.99ID:+jdcs43U0
一応、今の俺は、Longman (Pearson) に対しては、今回の回答だけで満足しておくことにして、
これ以上は尋ねないようにしようと思っている。

そして Collins から返事が来たら、その回答内容をここで再びみんなに公表する。
たぶん Longman (Pearson) と似たようなことを言ってくるとは思う。
0351名無しさん@英語勉強中 (アウアウウー Sa43-nysF)
垢版 |
2022/09/29(木) 10:33:03.49ID:TKFK1R/sa
ホンモノの解説は素晴らしいな
ゴミスレのクソ解説とはレベチだわ
0352名無しさん@英語勉強中 (オッペケ Sr47-9B5+)
垢版 |
2022/09/29(木) 11:13:31.40ID:+Cfv6W1+r
▶ He left a note at the scene of the crime, challenging detectives to catch him.
彼は犯行現場にメモを残し,捕えてみろと刑事に挑んだ.

cobuildのこれ見落としてたなあ
わざわざ語義分けしてある以上、両方あると考えるしかないのか
0354名無しさん@英語勉強中 (ニククエW 12f0-ohge)
垢版 |
2022/09/29(木) 13:29:15.12ID:vVr8hVIE0NIKU
Dr. Carverが関与の拒否してる事に異議を唱えてるんでしょ
だから異議にアンサーするには
Dr. Carverは関与してない事の証明しなければならない

関与の拒否をやれるもんならやってみろってのは
まだ関与の拒否をしていないという条件が必要
関与の拒否をすでにしていたら言えない事

まだしていないという条件が必要
[人]に[何かする事]を求める

しているか、まさにしようとしてるかの条件が必要
[人が何かする事]に異議を唱える
0355😉三年英太郎🌈 ◆3CZBjOt3.Y (ニククエW 1286-jMj8)
垢版 |
2022/09/29(木) 14:42:23.85ID:51JMXLK70NIKU
本質的に同義であるかは意見の分かれるところであるが、誤用レベルでは違うよね🤔 それとは別に

辞書は間違い!デタラメばっか言いやがって!ボクが正しいのに差別される🤬🤬🤬

ボクも【みんなも】誤解してたけど何でだろ❓😜テヘペロ

の人ね… 腫れ物扱いされるのは、そっちの側にも問題があるんじゃないの? 言っても通じてないだろうけどね
0359名無しさん@英語勉強中 (ニククエ 9233-JEMU)
垢版 |
2022/09/29(木) 19:02:45.79ID:EoU9Yg+W0NIKU
有名なCLRS著『Introduction to Algorithms Third Edition』に以下の文があります:

By Corollary 16.9, any elements that GREEDY passes over initially because they are not extensions of
∅ can be forgotten about, since they can never be useful.

この英文の構造が分かりません。

「because they are not extensions of ∅」などと途中に挟むのはOKなんですか?

「because」と「since」が一つづつ登場するのも奇妙です。
0360名無しさん@英語勉強中 (ニククエ Sd32-P12Y)
垢版 |
2022/09/29(木) 19:29:42.72ID:0uKxi8YbdNIKU
>>359
any elements [that GREEDY passes over initially because they are not extensions of
∅] can be forgotten about, since they can never be useful.

[]内はany elementsを先行詞とする関係詞節
主節の主語はany elements
述語動詞(受動態)はcan be forgotten about
sinceの前で一旦切れてる

becauseとsinceとの差異は辞書に載ってる
0361名無しさん@英語勉強中 (ニククエW 279d-9B5+)
垢版 |
2022/09/29(木) 19:36:48.76ID:Nmm9VC7Z0NIKU
構造はわかるけどこんな書き方するんだなってかんじ
0362OED Loves Me Not, a lonely frustrated old-timer w/Asperger's (ニククエ 1689-JEMU)
垢版 |
2022/09/29(木) 19:36:56.78ID:+jdcs43U0NIKU
>>359
数学のことはさっぱりわからんけど、英文だけを見て回答する。

(1) By Corollary 16.9, --- 「〜によって」

(2) any elements << that GREEDY passes over initially because they are not extensions of
∅ >> --- これが(長い)主語。that 以下は、elements にかかる関係節。

(3) can be forgotten about,
   --- これが述語動詞。

(4) since they can never be useful.
   --- それはなぜかというと、こうだからだよ、
   と (1), (2), (3) の全体に対する理由を述べている。
0368名無しさん@英語勉強中 (ニククエ 9233-JEMU)
垢版 |
2022/09/29(木) 20:22:06.21ID:EoU9Yg+W0NIKU
>>360,362
ありがとうございました。
0371エキセントリックなソフィスト ◆BWpafMA3P8me (ニククエ 6b22-tX/F)
垢版 |
2022/09/29(木) 22:10:01.41ID:gxUmlJsd0NIKU
>>347

だから、私が>>209

===========================
challenge+目的語+to不定詞は、次のように考えるとよい
基本は「できるものならやってみなよ」という意味。
===========================

と書いたではないか。俺は混乱してないので、よろしく。(^-^)
0373名無しさん@英語勉強中 (ワッチョイW b7b9-U/dG)
垢版 |
2022/09/30(金) 21:53:15.38ID:urN/mBOG0
>>371
けしかける、が適切な訳だと思う
0374The OED Loves Me Not, a cushed old daydreamer w/Asperger's (ワッチョイ cf89-vqPj)
垢版 |
2022/10/01(土) 03:20:39.84ID:m5Nrdqum0
>>187
COBUILD のその例文が変なところに引用されていて勘違いしやすいと俺は思って
Collins 社にメールを書いたが、返事が来た。Collins 社は「確かに変なところに
この英文を入れてしまった」と認めている。以下、俺の質問文と Collins 社の
返事とを引用する。

******************

俺から Collins 社へのメール
Dear Mesdames and Sirs:
First, could you please have a look at the following?

Excerpts from The Collins COBUILD Advanced Learners Dictionary, 9th Edition, 2018

challenge
[4] [Verb] If you challenge ideas or people, you question their truth, value, or authority

[verb + noun + to-infinitive] Democratic leaders have challenged the president to sign the bill.

Now, the above is a quote from your dictionary whose name is stated above. Is the example sentence above placed at the correct place? I was suspecting that it had perhaps been placed at the wrong place due to a minor error in editing (as the result of numerous additions, deletions, and corrections).

Could I have your answer as to whether the above example sentence is at the correct place?

By the way, as an English learner from Japan, I’ve always been enjoying your dictionaries.

Thank you.
0375名無しさん@英語勉強中 (ワッチョイ cf89-vqPj)
垢版 |
2022/10/01(土) 03:24:03.26ID:m5Nrdqum0
>>187 に関する Collins 社からの返事

dictionaries@harpercollins.co.uk

Hello,

Thank you for your email regarding an example sentence in the COBUILD Advanced Learner's Dictionary.

We agree that the example is not a very good match for the definition, and the editors will replace it as soon as possible.

Thank you very much again for bringing this to our attention, and thank you too for your kind words about our dictionaries.

Best wishes,
Collins Dictionaries
0376The OED Loves Me Not, A lonely crushed wanker w/Asperger's (ワッチョイ cf89-vqPj)
垢版 |
2022/10/01(土) 03:41:29.41ID:m5Nrdqum0
>>187
Collins 社のこの例文については編纂者たちがそれを修正すると言ってくれたが、
2日ほど前に返事をくれた Longman (>>185) にしても、俺が言ってたようにやっぱり
読者に誤解されやすい場所にそれが引用されていると思う。

Longman がいくらそれを俺に対して説明しなおしてくれたとはいえ、読者の中には
別の解釈をしてしまう人が現れる恐れもある。だからこそこのスレにていろいろと
疑問が出たのだ。

ちなみに、俺のこういう書き込みのどこがスレチなのだ?アホと違うか、と
思っていたが、クズどもに反論するとスレを消費するので、あえていつも黙っている。
0377OED Loves Me Not, a lonely crushed old daydreamer w/Asperger's (ワッチョイ cf89-vqPj)
垢版 |
2022/10/01(土) 03:50:59.97ID:m5Nrdqum0
というわけで、俺が最初から challenge の元々の意味から考えていた意味通りのことを、
この二つの辞書は言っていたに過ぎないということになる。

ところで、「それは俺がすでに言っていた」とかどうのこうの言うアホもいるが、
そいつらにもいちいち俺はコメントしない。

お前らが最初に言っていたとかいうが、俺も同じように思っていたのだ。第一、
そんなこと challenge の元の意味から考えれば当たり前のことだった。俺は誰が
何を最初に言っていたかなんて、覚えてもいない。

誰が何を最初に言おうが関係ない。真実を追求することだけが問題なのだ。いちいち
小山(こやま)の大将になりたがる奴が押し合いへし合いしたがるので、うるさくてかなわん。
0378OED Loves Me Not, a lonely crushed wanker w/Asperger's (ワッチョイ cf89-vqPj)
垢版 |
2022/10/01(土) 04:04:59.93ID:m5Nrdqum0
第一、辞書に書いてあることに疑問を持ったら、さっさとその辞書の編纂者に
質問したらおしまいだろが。それをここでどうのこうの、変な小理屈を並べ、
デタラメ理論を展開してた奴らはどこのどいつだ?

俺が代表して2つの会社にメールして回答をもらって、それをここに公表してやった
のに、それに対して「スレチ」だと?死ね。

これからも俺は、そういうクズどもからの煽りには応じない。ただ、このように
たまには応答する。
0379OED Loves Me Not, a crushed daydreaming wanker w/Asperger's (ワッチョイ cf89-vqPj)
垢版 |
2022/10/01(土) 04:07:05.24ID:m5Nrdqum0
それから、ネイティブに質問するときは、Skype みたいな肉便器を探し回る
連中のたむろするところで尋ねるなよな。それから、変なメールで個人同士
でこそこそと質問し、相手のネイティブの素性も何も外にいる俺たちにはわからないのに、
ここでそれを公表されてもまったく意味はない。

wordreference.com がたとえ匿名とはいえ、一応はコテハンをみんな名乗り、その素性も少しは
後悔しているし、それぞれの回答者が10年も20年も前から回答し続けていて、
俺たち閲覧者はそれぞれの回答者の過去の数十年間に蓄積された質疑応答を
クリックして簡単に閲覧できる。だから閲覧者たちは「ほう、この回答者は
20年も前からこういう誠実な回答を寄せてきたんだな。それなら信用できる」
と思うことができる。

しかし Skype での問答なんて、すぐ俺たち閲覧者の前からは消えるだろ?10年後に
俺たちがその回答を閲覧して、またここでそれについてみんなで論議するなんて
ことはできないだろ?だからそういう Skype とか個人同士のメールのやり取りなんて、
ここで公表しても意味がないと言うのだ。

第一、肉便器同士の質疑応答なんて、「また今度やらせてくれる奴だから、これに
答えておこうか」という乗りでしかネイティブも答えてないだろが。

さらには、質問者による英文の質問を公開しないで、英語ネイティブだけの
質問をここに公開しても意味がない。質問者がどういう英語の言い回しで質問しているか、
その言葉の端々によって相手のネイティブの答え方も変わってくる。

だからいつも俺は言っている。wordreference のように、数年後に閲覧者たちが
それを検索でき、しかもそれぞれの回答者の過去のログも閲覧できるようになっていて、
しかも質問者による英文の質問もしっかりといつでも閲覧できるようになったところで
ないと、まったく意味はない。そんな、自分の書いた下手な英文を公開したくない
などという卑劣な奴の情報が信用できるはずがない。それなら、kawakami とかいう
顔文字男の wordreference.com での質疑応答の方が、はるかに信頼性が高い。
0380😉三年英太郎🌈 ◆3CZBjOt3.Y (ワッチョイW ff86-nigo)
垢版 |
2022/10/01(土) 04:27:42.82ID:qHLzT9hN0
>自閉スペクトラム症の人たちは、特性を周囲に理解してもらいにくく、いじめ被害に遭う、一生懸命努力しても失敗を繰り返す、などのストレスがつのりやすいため、身体症状(頭痛、腹痛、食欲不振、チックなど)、精神症状(不安、うつ、緊張、興奮しやすさなど)、不登校やひきこもり、暴言・暴力、自傷行為などの「二次的な問題(二次障害)」を引き起こしやすいといわれています。そうなる前に家族や周囲がその子の特性を正しく理解し、本人の「生きづらさ」を軽減させて二次的な問題を最小限にとどめることが、自閉スペクトラム症への対応の基本となります。
https://www.smilenavigator.jp/asd/abc/

誰か周囲にいないのか❓🤕
0381😉三年英太郎🌈 ◆3CZBjOt3.Y (ワッチョイW ff86-nigo)
垢版 |
2022/10/01(土) 05:07:34.98ID:qHLzT9hN0
>We agree that the example is not a very good match for the definition, and the editors will replace it as soon as possible.

例文があまりよろしくないと言ってるだけで、定義がまちがってたとは一言も言ってない🤔

結局、長文の方は終始間違えてたのに、何を怒ってるのだろうか🤔
0382😉三年英太郎🌈 ◆3CZBjOt3.Y (ワッチョイW ff86-nigo)
垢版 |
2022/10/01(土) 05:16:30.27ID:qHLzT9hN0
編集の結果、本来入れるところを間違えて、変なところに入れてしまったんでしょ❓そうなんでしょ❓❓そうじゃないんですか❓❓❓

という圧の強い質問に、この定義に対して例文がよろしくないとしか言ってないのだが…🤔 わたくしは以前、似たような質問を見た。

これは驚きのwouldですよね❓そうでしょ❓❓そうに違いないでしょ❓❓❓

という質問に、大学の教授が仕方なく「~のギャップで驚きが出てますね~」と大人の対応をしているのを。
0386😉三年英太郎🌈 ◆3CZBjOt3.Y (ワッチョイW ff86-nigo)
垢版 |
2022/10/01(土) 06:24:42.63ID:qHLzT9hN0
辞書は間違い!デタラメばっか言いやがって!ボクが正しいのに差別される🤬🤬🤬

ボクも【みんなも】誤解してたけど何でだろ❓😜テヘペロ

例文が悪い😤編集者にメールしたぞ😤😤お前ら参ったか🖕🏻

😳⁉
0387OED Loves Me Not, a lonely crushed old daydreamer w/Asperger's (ワッチョイ cf89-vqPj)
垢版 |
2022/10/01(土) 07:26:29.29ID:m5Nrdqum0
なんか、俺が物を言うたびに例の肉便器の豚の名前が出てくるんだが、
もしかして何か言ってるのか?3年前からNG 登録してるから読めないし、読む気にもなれんけど。
相変わらず、肉便器たちからの英文の回答だけを披露して、自分が書いた稚拙な
英文の質問だけは卑劣にも隠し続けてるんだろ?
0388名無しさん@英語勉強中 (テテンテンテン MM7f-vqPj)
垢版 |
2022/10/01(土) 08:33:11.27ID:jKsMgXpLM
>>386
他人に間違いを認めるよう、しつこく絡んでるけど
だったら君も
The best way to see Tokyo is on foot.←コピュラ
On foot is the best way to see Tokyo.←コピュラ
The nearest way is through the wood.←存在
Through the wood is the nearest way.←コピュラ

↑こんな解釈が不自然だって、素直に認めなよw
0391ブタ耳 (ワッチョイW 8fe3-ZYgd)
垢版 |
2022/10/01(土) 08:52:03.17ID:eu5QAUZp0
圧倒的にOED氏が正論を述べていると思うよ。
「肉便器」などの罵倒語は、せっかくの正論を汚すので言わない方が良かった。
でもまあブチ切れたい気持ちは良く分かる。
0394ブタ耳 (ワッチョイW 8fe3-ZYgd)
垢版 |
2022/10/01(土) 09:03:25.61ID:eu5QAUZp0
このコテは久しぶりに使ったけど、OED氏が覚えててくれると嬉しいです。
随分前に、詩の解釈中の時制の問題で揉めた時に、「OED(仮)氏」という呼びかけで長々と氏を擁護したことがありました。
いつもためになる書き込みありがとうございます。
これからも三年さんと仲良くやっていってくださいw
0395OED Loves Me Not, a lonely crushed old daydreamer w/Asperger's (ワッチョイ cf89-vqPj)
垢版 |
2022/10/01(土) 09:09:06.56ID:m5Nrdqum0
>>391
久しぶりのブタ耳さんの登場。また現れてくれて、うれしいよ。凄まじいほどの
学識でこのスレを圧倒してください。

ところで、あなたではなくて他の奴が「肉便器」が女性への罵倒語だと言ったけど、
そんなことを俺は知らなかった。俺に言わせれば、男の方がむしろ肉便器だ。
第一、ネット上の辞書を見ても、この言葉を男に対して使うこともある、と書いている。

だいいち、この言葉を知ったのはこの5ちゃんねるの英語板で、しかもほんの
4年ほど前に同性愛の男同士の話をしている奴が言ってたので覚えただけ。
俺は英語板でしかこの言葉を使ったことがないし、女性を憎悪したことなんてない。

というわけで、またもや下らない雑談をしてしまった。純然たる英語の話あるいは
何か学術的な話しかしたくないんだが、どうもごく一部の奴らが盛んに攻撃を続けて
来るので、仕方なくたまには反論しておかないといけない。普段はひたすら黙ってるけど。
0396OED Loves Me Not, a lonely crushed old daydreamer w/Asperger's (ワッチョイ cf89-vqPj)
垢版 |
2022/10/01(土) 09:14:16.72ID:m5Nrdqum0
>>394
はっきりあの時のことを覚えていますよ。どうか、そのコテハンでもいいし、
他のコテハンでもいいから、これからもこの英語板を見守っていてください。

僕はネット上のあらゆる英語関係のサイトで20年ほど発言を続けてきたけど、
他のほとんどすべてのサイトに愛想を尽かし、この5ちゃんねる英語板
だけは、あなたのような人がごくたまにいてくれるので、これからも
やはりここがネット上の英語関係サイトで一番だと思っています。
0397名無しさん@英語勉強中 (ワッチョイW c3b9-t4m4)
垢版 |
2022/10/01(土) 09:24:16.58ID:SZxTmJPi0
>>377

おやまの たいしょう(オ)⑤ 、⓪-① 【《御山の大将】
㊀子供の遊び。低い盛り土などの上に立って、のぼってくる者を突き落としあう。
㊁小さな集団の中で自分が最も△えらい(すぐれている)と思っている人を、はたから からかって言う言葉。

小山の大将という言葉はないんだよね。
御山の大将です
0398名無しさん@英語勉強中 (ワッチョイ 8fe3-e5cN)
垢版 |
2022/10/01(土) 10:03:53.63ID:eu5QAUZp0
>>396
「凄まじいほどの学識」など生憎持ち合わせていませんし、「見守る」立場ではなく、学ばせてもらう立場で今後もこのスレに関わっていくつもりです。概ね名無しとして。
5ちゃんでのコテ使用が廃れつつある中で、このスレでコテを常用して発言に責任を持っている方たちは尊敬に値すると思います。

アスペルガーに関して公言してる人では深澤真紀さんとか有名ですね。YouTubeのポリタスTVなどで怒涛のしゃべりを聞くことができます。
他に最近はまってるのが韓国ドラマの「ウ・ヨンウ弁護士は天才肌」。かなり重度の障害を抱えながらも、本質を見抜く天才的能力を持った女性弁護士が活躍するドラマです。
こういった人たちからは、アスペルガーを障害としてではなくむしろ強力な武器として戦っている印象を受けました
0399名無しさん@英語勉強中 (ワッチョイW c3b9-t4m4)
垢版 |
2022/10/01(土) 10:09:41.79ID:SZxTmJPi0
>>398
ではアスペルガー呼ばわりされて喜ぶ人に、
アスペルガーというのは嬉しい言葉なんだよ
と言われて嬉しくなるのは、アスペルガーという言葉の
意味を知らない人くらいなものだ、と書いたら、
いやそうじゃないんだ、>>398を読んだらそれで理解するはず
と言われた時に、納得する人はどれだけいるんですか?
わかんないよ、他人のことだもん、と答えると思いますが、
自分自身が他人に対して共感する力に欠けているかもしれないと、
少しは疑うことが事実をありのままに見るのに役に立つかも
しれないと思った
0400名無しさん@英語勉強中 (テテンテンテン MM7f-vqPj)
垢版 |
2022/10/01(土) 10:11:54.01ID:jKsMgXpLM
>>390
単なる感想ではありませんけど。
主語と補語の関係が「同定」である場合
それを入れ替えても同様な意味のSVCの文ができるということは
安藤本にも書いてあることです。

ガチ中卒君は真正馬鹿なのだから
横から出てこないようにね。
0401名無しさん@英語勉強中 (テテンテンテン MM7f-vqPj)
垢版 |
2022/10/01(土) 10:12:07.55ID:jKsMgXpLM
>>390
単なる感想ではありませんけど。
主語と補語の関係が「同定」である場合
それを入れ替えても同様な意味のSVCの文ができるということは
安藤本にも書いてあることです。

ガチ中卒君は真正馬鹿なのだから
横から出てこないようにね。
0402名無しさん@英語勉強中 (テテンテンテン MM7f-vqPj)
垢版 |
2022/10/01(土) 10:13:36.02ID:jKsMgXpLM
>>393
ガチ中卒の真正基地外がまた発狂してるw
今度はブタ耳を「バカ高卒」扱いw
死ねばいいのにw基地外w
0403名無しさん@英語勉強中 (ワッチョイ cf89-vqPj)
垢版 |
2022/10/01(土) 10:20:51.18ID:m5Nrdqum0
>>397 
そんな言葉がないことを承知で、わざと書いてるんだろが。
だからこそ「 」でその言葉を括ってるんやろが。
国語力もない癖に、偉そうなこと抜かすな。クズが。
0404名無しさん@英語勉強中 (テテンテンテン MM7f-vqPj)
垢版 |
2022/10/01(土) 10:24:13.96ID:jKsMgXpLM
>>397
0405名無しさん@英語勉強中 (テテンテンテン MM7f-vqPj)
垢版 |
2022/10/01(土) 10:25:36.26ID:jKsMgXpLM
>>397
アスペ?

>>399 も、ちょっとなにいってるかわかんない
0406OED Loves Me Not, a lonely crushed old daydreamer w/Asperger's (ワッチョイ cf89-vqPj)
垢版 |
2022/10/01(土) 10:29:14.28ID:m5Nrdqum0
俺は既存の言葉をそのまま使って、横並び没個性な生き方が大嫌いだから、
わざわざ別の言葉を作って書いていても、それは間違いだなどと言って、
相手がどういうつもりでそう書いているか想像もできない奴らに、俺の
文章を読む資格はない。俺に話しかける資格もない。ふだんはせっかくそういう
個人攻撃に応じないように黙ってやってるが、だからと言ってつけあがりさらすな。
ブタどもが。
0407OED Loves Me Not, a lonely crushed old daydreamer w/Asperger's (ワッチョイ cf89-vqPj)
垢版 |
2022/10/01(土) 10:33:49.26ID:m5Nrdqum0
俺が「肉便器」と一言だけ書いても、途端に俺が女性憎悪しているなどと
完全に誤解し、俺をこき下ろそうとする。木を見て森を見ない。

俺をみんな Aspervger's syndrome を患っているなどと抜かすが、貴様らの方が
近視眼的で、よっぽど精神異常だろうが。

貴様らはみんな、ろくに本も読まずに何も考えずに「千擦り」(せんずり)ばかり
こいてやがるからそんなふうに想像力も何もなくなる。

ところで、俺は「千擦り」をわざとこのように書いてるんだから、またもや
そんな漢字を使うのは間違いだなどと低能が暴露するようなことを二度と書きさらすな。
0409名無しさん@英語勉強中 (テテンテンテン MM7f-vqPj)
垢版 |
2022/10/01(土) 10:46:31.72ID:jKsMgXpLM
>>408
お前、「バカ高卒」以外のコメントできないの?
内容のないレスばっかりしやがってw
よっぽど学歴にコンプレックがあるんだね(泣)
4流大学にやっと入れた御仁なのだろうか?
つか、割とマジでお前が高卒以下の可能性があると思ってるんだけど。
0410😉三年英太郎🌈 ◆3CZBjOt3.Y (ワッチョイW ff86-nigo)
垢版 |
2022/10/01(土) 11:59:04.75ID:qHLzT9hN0
>>390
A is B は The man is John.のような同定のコピュラ文でないと、B is A はコピュラ文にならない。

つか統語範疇とゆー生成用語使った時点で、それは生成の視点の話になるから「形容詞的用法」は✨絶対に間違いなの✨ 分かった❓🤦🏻‍♂
0411😉三年英太郎🌈 ◆3CZBjOt3.Y (ワッチョイW ff86-nigo)
垢版 |
2022/10/01(土) 12:04:28.64ID:qHLzT9hN0
テテンの特徴は、「Aはどうなの?」という質問に答えると、じゃあBは?Cは?Dは?と来て徐々に論点を動かしつつ、やや異論の余地がありそうなFが見つかると、ABCDEは捨ててFで一点論破(?)しようとする。FとABCDEはまた別の問題なのに🤦🏻‍♂

タラコと同じタイプやね🤦🏻‍♂
0414😉三年英太郎🌈 ◆3CZBjOt3.Y (ワッチョイW ff86-nigo)
垢版 |
2022/10/01(土) 12:27:57.08ID:qHLzT9hN0
>>390に関しても当初の問題は
1a. be on a trip.
2b. go on a trip.
の on a trip は何句かということであった。

cf.
2a. The road is through the forest.
2b. The road goes through the forest.

SVCだとすると、Cは多くの場合(本来的な意味での)形容詞句書き換え可能であるだが,当該文はどうであろうか。

He's on a trip.
👉 He['s traveling].

動詞だし、動詞的意味内容を持っているよな🤔
0417😉三年英太郎🌈 ◆3CZBjOt3.Y (ワッチョイW ff86-nigo)
垢版 |
2022/10/01(土) 12:50:00.83ID:qHLzT9hN0
>>395
また一つ新しい嘘をつく

彼は、「軽薄な」白人と付き合ってる日本女に対して「肉便器」と罵倒したのである。ホモ用語では「種壺」というので、ホモ界ではほぼ耳にしない用語であるし、おれはこの世にもおぞましい言葉は使わない。

何にせよ、すぐに豚だの死ねだの白痴だの口に出てしまう、感情のコントロールができない方が、いまさら「肉便器」の件だけ取り繕うのは滑稽である🤦🏻‍♂
0418😉三年英太郎🌈 ◆3CZBjOt3.Y (ワッチョイW ff86-nigo)
垢版 |
2022/10/01(土) 13:00:32.09ID:qHLzT9hN0
5chのどこかで「肉便器」という言葉を聞いて、ネトウヨ気質なこの方はそうだそうだ、こりゃあ便利な言葉だ、と思ったのであろう。

こーゆー言葉を(匿名掲示板であれ)口に出すのは、まともな感覚ではない。

しかも誰かから諌められたら、いやゲイから聞いたし、ゲイに言ってるから無問題と弁明するが、ゲイ相手なら許されるとでも?
0419😉三年英太郎🌈 ◆3CZBjOt3.Y (ワッチョイW ff86-nigo)
垢版 |
2022/10/01(土) 13:12:09.62ID:qHLzT9hN0
>>407
>低能

てい‐のう【低能・低脳ナウ】
〘名〙 知能が低いこと。知能の発育が遅れていること。
※神経病時代(1932)〈佐多芳久〉「脳の発育が悪いために多くは低能になる」

口を開けばこーゆー言葉が出てくるね🤦🏻‍♂
発達障害の人は、周りの人を不必要に傷つけることがある。本人だけの問題ではないんだから、ちゃんと医者にみてもらって適切なケアを受けましょう🏥
0423名無しさん@英語勉強中 (ワッチョイ cf8f-xADz)
垢版 |
2022/10/01(土) 13:54:21.63ID:Ey6i2/jr0
比較文、as〜asの2つ目のasは接続詞らしいですが、as節は何かと同格になるんでしょうか?
0425😉三年英太郎🌈 ◆3CZBjOt3.Y (ワッチョイW ff86-nigo)
垢版 |
2022/10/01(土) 15:39:16.64ID:qHLzT9hN0
> [verb + noun + to-infinitive] Democratic leaders have challenged the president to sign the bill.
>We agree that the example is not a very good match for the definition, and the editors will replace it as soon as possible.

何度読んでも、この例文はこの定義にはあまりよろしくない、としか言ってないと思うのだが…🤔

すでにこの定義が問題ないことはロングマンで確定している。

この例文がよろしくない(かもしれない)のは、
[the president] [to sign the bill.]
[大統領に][法案に署名するよう]挑んだのか
[the president to sign the bill.]
[大統領が法案に署名すること]に挑んだのか

たとえばアメリカ人なら、Democratic leaders が挑んでるのだから、大統領はRepublicanだろうと勘が働いて、「疑問視」だろうと察しがつくけれど、

そういう政治的知識がなければどちらにも読めてしまうので、あまりよろしくないと言ったのではなかろうか…🤔

とまれ、長文の方が勝ち誇る要素はどこにもなかろう
0426😉三年英太郎🌈 ◆3CZBjOt3.Y (ワッチョイW ff86-nigo)
垢版 |
2022/10/01(土) 15:43:22.05ID:qHLzT9hN0
政治の知識がない人が、「これは編集上のミスですよね❓そうなんですよね⁉」とクレームを入れたのに対して、

辞書の中の人が「あぁ、そーゆー知識がないと誤読することもあるんだぁ…例文差し替えます」

と言ってるだけでは?🤔
0428名無しさん@英語勉強中 (テテンテンテン MM7f-vqPj)
垢版 |
2022/10/01(土) 16:59:31.98ID:jtgniavdM
>>410
お前は↓これを同定のコピュラ文だと言ってんだぞ。
Through the wood is the nearest way.

頭涌いてんのか?

統語範疇じゃなく品詞というべき点は認める。
つか、しつこいなぁ。
お前こそ、一転論破だろwアホ
0429名無しさん@英語勉強中 (テテンテンテン MM7f-vqPj)
垢版 |
2022/10/01(土) 17:14:27.26ID:jtgniavdM
Through the wood is the nearest way.
安藤講義では、この文の主語を、前置詞句のThrough the woodだとしている。
で、その後にbe動詞と名詞が続いているのだから
安藤がこの文をSVCだと判断しているのは間違いない。

英太郎は最初、SVCと安藤は書いてないとイチャモンをつけていたが
SVCと判断しているという解釈以外はあり得ない。

で、俺が反論してようやく納得して
英太郎はこの文をコピュラ文だと考え直したのだ。
もう忘れてしまったようだが・・・ 苦笑
0430名無しさん@英語勉強中 (テテンテンテン MM7f-vqPj)
垢版 |
2022/10/01(土) 17:33:28.17ID:jtgniavdM
>>A is B は The man is John.のような同定のコピュラ文でないと、B is A はコピュラ文にならない。

Through the wood is the nearest way.は、同定のコピュラ文

The nearest way is through the wood. というコピュラ文ができあがる。

ここまでかみくだいて言わないと、通じないもんかね 苦笑

勿論、
「The nearest way is through the wood.
がコピュラ文となる場合があることは確かだが
存在文として作成される場合もあるのではないか」
という強弁は可能だが

少なくとも
コピュラ文であり得ることは認めざるを得ないはずだ。
0431名無しさん@英語勉強中 (テテンテンテン MM7f-vqPj)
垢版 |
2022/10/01(土) 17:38:05.40ID:jtgniavdM
>>411
タラコはお前だろ

別に論点をずらしてなんかねーわ
SVCかSVMという話題になったから
そういえば、昔こんな争いもあったなって
書いてるだけだろーが

ABCDEを捨てて?
何のことだよ、嘘つき

統語範疇と言ってしまったのは確かにミスだったけど
そのことかよ

お前の方が、いろんなことを捨ててんだろ
同定のSVCじゃないのに、主語と補語を入れ替えたりして・・・
指摘されても全くスルーしてんだろーが、カス
0432名無しさん@英語勉強中 (テテンテンテン MM7f-vqPj)
垢版 |
2022/10/01(土) 17:52:34.89ID:jtgniavdM
次に
He's on a trip. について

>>414
>>He's on a trip.
>>👉 He['s traveling].
>>動詞だし、動詞的意味内容を持っているよな

現在分詞は形容詞的な意味ももってるだろーが。

現に、現在分詞は準主格補語となることができるし
それに、現在進行形の文S is doing. をSVCと扱う本もあるくらいだ。
たしか『英文法を哲学する』という本にもそんなことが書いてあったと思う・・・

以下、俺はSVCだと思う文の、書き換え。
He is at work.
He is working.

His house is on fire.
His house is burning.

The workers are on strike.
The workers are striking.
0433名無しさん@英語勉強中 (テテンテンテン MM7f-vqPj)
垢版 |
2022/10/01(土) 18:05:22.62ID:rmkn9pYHM
>>415
お前がお礼しろ、アホ
0435名無しさん@英語勉強中 (ワッチョイ 8fe3-e5cN)
垢版 |
2022/10/01(土) 21:01:21.27ID:eu5QAUZp0
>>425
その法案がRepublicanにとって有利なものであるのか不利なものであるのか確定していない以上、どちら解釈の可能性もあるんじゃないですか?
だとすれば、例文が適切かどうかの判断に、政治的知識の有無は関係ないことになりませんか?
0436名無しさん@英語勉強中 (テテンテンテン MM7f-vqPj)
垢版 |
2022/10/01(土) 21:04:45.85ID:jtgniavdM
>>411>>テテンの特徴は、「Aはどうなの?」という質問に答えると、じゃあBは?Cは?Dは?と来て徐々に論点を動かしつつ、やや異論の余地がありそうなFが見つかると、ABCDEは捨ててFで一点論破(?)しようとする。

SVCが答えとなるような疑問文が全てWhat are you?じゃねーだろと突っ込んだり
「属性」の定義があいまいだから、問いただしたり
同定の関係でないSVCのSとCを入れ替えてるから、意味ねーだろと突っ込んだり
SVCの入れ替えと、SVAの倒置を混同してるから突っ込んだり・・・

誤解してるのを正したり
疑問な点を問うてるだけのつもりなのに

論点を動かして、揚げ足とりをしてるみたいな話になってて草
0437名無しさん@英語勉強中 (テテンテンテン MM7f-vqPj)
垢版 |
2022/10/01(土) 21:13:54.05ID:jtgniavdM
英太郎氏は、間違えて英文法書スレに、もう私にはレスをしないと逃走宣言を誤爆しております。

>>428-432
↑俺の主張はこれにまとまっている。別に論点をずらしていった覚えなない。
まぁ反論できないだろうから、泣きながら逃走すればいんじゃねーの

前のバトルについては、同じSVCかSVMかという話題だったから、
「文型の話で言えば・・・」という感じで出しただけ。
そりゃ、言及するでしょ。同じ相手と同じテーマで意見が対立した過去があったってことだもん。

それを、論点ズラシと主張してるのかもね。
0438😉三年英太郎🌈 ◆3CZBjOt3.Y (ワッチョイW ff86-nigo)
垢版 |
2022/10/01(土) 21:30:33.72ID:qHLzT9hN0
ディナーの後に辞書をひきつつもう一度考えたのだが

>>425
[the president] [to sign the bill.]
[大統領に][法案に署名するよう]挑んだのか



>>427の全てを撤回しよう。この例文は、[大統領が法案に署名すること]を疑問視した、としか読めまい
0439名無しさん@英語勉強中 (テテンテンテン MM7f-vqPj)
垢版 |
2022/10/01(土) 21:35:17.45ID:jtgniavdM
>>徐々に論点を動かしつつ、やや異論の余地がありそうなFが見つかると、
>>ABCDEは捨ててFで一点論破(?)しようとする。

もうレスしないそうだけど
論点を動かすとか、ABCDEを捨てるとか
全くなんのことがわからんので
できれば、具体的に示して欲しいものだ。

たんなる逃走の言い訳でなければだけど。
0440名無しさん@英語勉強中 (スップ Sd1f-+pZx)
垢版 |
2022/10/01(土) 21:58:39.17ID:I5R5pxMNd
>>398
文体が完全にバカ高卒くんでワロタwww

>>396
擁護し自分を誉めてくれれば書いてる内容の中身はなんでもよさそうだな爺さんはwwwwww
0441名無しさん@英語勉強中 (スップ Sd1f-+pZx)
垢版 |
2022/10/01(土) 22:00:50.77ID:I5R5pxMNd
バカ高卒くんが詩の講釈垂れながらアスペ爺をほめてアスペ爺が感心してる様が想像できすぎるwwwwww
0442名無しさん@英語勉強中 (スップ Sd1f-+pZx)
垢版 |
2022/10/01(土) 22:03:30.46ID:I5R5pxMNd
>>395
バカ高卒くんに凄まじいほどの学識なんてこれっぽっちもねえよwww
お前ほんとに権威主義だし知的コンプが凄いんだろうねえw
0443名無しさん@英語勉強中 (スップ Sd1f-+pZx)
垢版 |
2022/10/01(土) 22:06:45.98ID:I5R5pxMNd
私が見て来た限り、バカ高卒くんに騙されるのってアスペの人たちなんですよね、男女に関わらず
アスペの人って寂しいから「全肯定」されちゃうと脊髄反射でそれに飛びついてしまう
思考に深さは無いからそれ以上は考えない
0445😉三年英太郎🌈 ◆3CZBjOt3.Y (ワッチョイW ff86-nigo)
垢版 |
2022/10/01(土) 22:13:05.30ID:qHLzT9hN0
>challenged [the president] [to sign the bill.]
[大統領に][法案に署名するよう]求めた

わたくしが、この読みが無理であると考え直したのは、もう一度英英の定義を読み直したからである。

3 SOMETHING DIFFICULT
to test the skills or abilities of someone or something
https://www.ldoceonline.com/jp/dictionary/challenge

3 to test someone's skill and abilities
https://www.macmillandictionary.com/dictionary/british/challenge_2

法案に署名することはスキルとかアビリティなのだろうか?

>>220
>1. We challenged [the police] [to arrest him.]
警察が特定の人物を逮捕することはアビリティであろうか?

発端に立ち返る。
>>177
>We challenged the school to prohibit the use of bikes.
学校が自転車を禁止するのはアビリティであろうか?

否、否、否。一方、

>>204
>He challenges teachers to rethink their approach.

考え直すのはアビリティとも言えよう🧐
 👉ability to think 《the ~》物を考える力[能力]
0446😉三年英太郎🌈 ◆3CZBjOt3.Y (ワッチョイW ff86-nigo)
垢版 |
2022/10/01(土) 22:24:14.52ID:qHLzT9hN0
コウビルドの中の人が当該例文をあまりよろしくないと考えたのは、

やはり米のrepublican vs democratic の政治状況を知らないとピンと来ない人がいる、

ということでなかろうかと個人的には思っている😐
0447名無しさん@英語勉強中 (テテンテンテン MM7f-vqPj)
垢版 |
2022/10/01(土) 22:46:18.36ID:jtgniavdM
論破されて逃走するときのテンプレートです。

〇〇の特徴は、「Aはどうなの?」という質問に答えると、じゃあBは?Cは?Dは?と来て徐々に論点を動かしつつ、やや異論の余地がありそうなFが見つかると、ABCDEは捨ててFで一点論破(?)しようとする。FとABCDEはまた別の問題なのに🤦🏻‍♂
〇〇に触ると時間浪費するんで、今後はスルーさせていただきます。悪しからず🙇🏻‍♂

これで負けを認める必要はなくなります( *´艸`)
0448名無しさん@英語勉強中 (テテンテンテン MM7f-Od5l)
垢版 |
2022/10/01(土) 23:32:33.09ID:ee9AwFmdM
英語板のぴろゆきこと英太郎、発狂して俺のこと、あぼーん登録までしたんだってさw

俺はことごとく英太郎に論破されたのに
次々に論点をずらしていき
ある一点で揚げ足しをとって勝ち誇ってるというのが
ぴろゆきの主張なんだけどw

全然そんな認識ねーわ
馬鹿な返しばっかしてくるから反論していっただけだろ

He's on a trip.
The nearest way is through the wood.
この2つの文がSVCかSVAかというのがテーマ。

>>428-432
に書ているようなことが、揚げ足取りだと思いますか?
議論の主軸となるところでしょう。
ぴろゆきは論破されて、でたらめな言い訳を喚きながら、逃走したのですwww
0449名無しさん@英語勉強中 (スッップ Sd1f-+pZx)
垢版 |
2022/10/02(日) 02:27:19.80ID:zIR+jbcZd
>>444
バカ高卒くん、俺に読んで評価してもらいたいならお前の書いたレスの番号をそのURLの/のあとに半角数字で書いてここに載せてくれ
でないとdat落ちのスレを参照する気が起きない
0450名無しさん@英語勉強中 (スッップ Sd1f-+pZx)
垢版 |
2022/10/02(日) 02:28:22.91ID:zIR+jbcZd
>>448
ぴろゆきとか糞寒いからお前は消えろ、ゴミ高卒くんw
0451名無しさん@英語勉強中 (スッップ Sd1f-+pZx)
垢版 |
2022/10/02(日) 02:29:00.78ID:zIR+jbcZd
バカ高卒くん=高卒1号
ゴミ高卒くん=高卒2号

頼むからお前ら同士では喧嘩すなやwww
0452名無しさん@英語勉強中 (ワッチョイ f396-pIDl)
垢版 |
2022/10/02(日) 04:57:35.31ID:rLueIK2I0
>>377は『「それは俺がすでに言っていた」とかどうのこうの言うアホもいるが、俺も同じように思っていたのだ。』と自己弁護してるけど、あなたは>>347で、「なんで今回はあんなに俺も他のみんなも混乱したんだろうか?」と言ってるよね。あなたは、混乱してたのだ。あなたは、嘘もつく人なの?
それに対して、ソフィスト氏は、>>371で「私は、混乱してなかったし、確固たる考えを持っていたので、みんなが混乱してたなんて言うのをやめてくれないか」と正当な反論をしたにすぎないように見える

最近のやり取りを見てる限り、ソフィスト氏はあなたの間違いを正す尻ぬぐいをしてるだけのように見える。小山の大将になりたいのは、あなたのほうに見えるけど。高度な判断が求められる場面において、ソフィスト氏よりも英語力なさそうだし。
0453名無しさん@英語勉強中 (ワッチョイW 63fb-t4m4)
垢版 |
2022/10/02(日) 08:55:45.28ID:tTiv9BiK0
御山の大将や
小山やないで
0454ブタ耳 (ワッチョイ 8fe3-e5cN)
垢版 |
2022/10/02(日) 11:18:33.38ID:OimBwqcD0
過去スレ(>>444)で話題になった詩を読み返してみた。

Like smoke that fades within a glass
Seem the curled fancies of my pen.

最近、AIによる自動絵画作成で遊んでいるのだが、このイメージをAIに描かせたらどうなるだろうか?

AIは、入力された key words ("prompt" と呼ばれる)に沿うように画像を作成して出力する。
まずDALL-EというAIに上記2行をそのままの形で prompt として読ませたみた。

出力結果:
https://imgur.com/Sv857f6

ここまで描き出すとは予想していなかった。驚くしかない。
0455ブタ耳 (ワッチョイ 8fe3-e5cN)
垢版 |
2022/10/02(日) 11:31:11.00ID:OimBwqcD0
コップを逆さまに置いた画像が欲しかったので prompt を工夫してみたのだが、うまくいかない。

https://imgur.com/do9U4wt

使用プロンプト:
"an upset-down glass with curled white smoke in it, centered, realistic, by Andrew Wyeths"

upset-down以外の適切な表現がすぐに思い浮かばないんだけど、何かありませんかね?
0458名無しさん@英語勉強中 (スッップ Sd1f-+pZx)
垢版 |
2022/10/02(日) 12:02:54.59ID:giy1sbP1d
>>454-457から凄まじい程の学識を感じる🤣www
0460OED Loves Me Not, a lonely crushed old daydreamer w/Asperger's (ワッチョイ cf89-vqPj)
垢版 |
2022/10/02(日) 12:12:02.62ID:StQiaEX90
>>457
>>an upside-down glass with curled white smoke in it, centered, realistic, by Andrew Wyeths

どうやればそのサイトで、その言葉を入れて画像を変えられるのか、僕にはわからないけど、
an upside-down glass
がダメなら
(1) a glass turned over
とか
(2) a glass turned upside down
みたいな言葉を入れてもダメかな?
0461ブタ耳 (ワッチョイ 8fe3-e5cN)
垢版 |
2022/10/02(日) 13:42:41.52ID:OimBwqcD0
>>460
アドバイスありがとうございます。

Dall-Eは使用回数に制限があるので、制限の無いStable Diffusionを使ってあれこれ試行錯誤してみましたが、うまくいかないですね。
コップをひっくり返すことさえできないとは、情けないのは私なのかAIなのか。

"an overturned glass which captures curled white smoke in it, centered, realistic"
で、12枚同時出力した結果。
https://imgur.com/pZXCZeo

一番下の真ん中の画像は、ひっくり返っているように見えなくもないけど、コレジャナイ感が半端ない。
0464ブタ耳 (ワッチョイ 8fe3-e5cN)
垢版 |
2022/10/02(日) 14:36:20.61ID:OimBwqcD0
>>463
実写です。実物をスマホで撮影しました。
ナイスボケありがとうございますw

おまけ:
「高校を卒業したばかりのブタの肖像」 gererated by Stable Diffusion
"a portrait of a pig that has just graduated from high school, intricate"
https://imgur.com/KBhj0Dp
0465ブタ耳 (ワッチョイ 8fe3-e5cN)
垢版 |
2022/10/02(日) 14:58:12.70ID:OimBwqcD0
調子に乗ってもう1枚(これで最後にします)

「凄まじい学識をもって高校を卒業したばかりのブタの肖像 by レオナルド・ダ・ヴィンチ」
"a portrait of a pig that has just graduated from high school with tremendous academic knowledge, intricate, oil painting by Leonardo Da Vinci"
https://imgur.com/9JwM89D

なぜ鼻の穴が4つあるのかは謎である。
老翁の禿げ頭に生え残った頭髪のような頭頂部の体毛が、ブタにあるまじき「凄まじい学識」を表現している。
何よりも特筆すべきは巨大な両耳の美しさである。
0468名無しさん@英語勉強中 (スッップ Sd1f-+pZx)
垢版 |
2022/10/02(日) 15:43:18.51ID:EcYqdG3bd
>>466
バカ高卒くんが英語板全体に居着きはじめてからでしょうねw
とにかく彼は暇なのと嫉妬が物凄まじいので、当分はやめないでしょうw
彼が居着いたスレや板は今まで必ず過疎化していっていますw
理由は彼の自演がつまらなさすぎるからですwww
0470名無しさん@英語勉強中 (スッップ Sd1f-+pZx)
垢版 |
2022/10/02(日) 15:47:07.66ID:EcYqdG3bd
権威主義で知的コンプのすごいアスペ爺さんには『学識ある無知について』とか無知の知なんて思いも及ばないでしょうね
その点でバカ高卒くん程度にもすぐき云わされてしまうわけやなwww
0473😉三年英太郎🌈 ◆3CZBjOt3.Y (ワッチョイW ff86-nigo)
垢版 |
2022/10/02(日) 16:08:35.54ID:AI1u13JP0
長文の方は味方(※)が現れると自分でも思ってない美辞麗句で褒め殺しする癖がある

※テテンは別にあなたの味方じゃありませんよ。 またいつか"裏切られて"発狂することがないよう、忠告しときます🙇🏻‍♂
0474名無しさん@英語勉強中 (スッップ Sd1f-+pZx)
垢版 |
2022/10/02(日) 16:12:16.92ID:EcYqdG3bd
ですな

加えて老婆心ながら
映画スレの機械翻訳が酷い御仁のこともかなり信じているようだけどあれはアヤシいと睨んでるw
人の寂しさに付け込む三流高卒詐欺師のかおりがするw
0475ブタ耳 (ワッチョイ 8fe3-e5cN)
垢版 |
2022/10/02(日) 17:11:07.51ID:OimBwqcD0
>>474
>>262 を書いたあなたは、さっさと>>298の②の線にそって、以下の文を和訳を提示してみなさいよ。
>>319以降ずーっと逃げまくってるみたいだけど。

To his library in Cable House he will add volumes in which are contained what human knowledge,
in the 1820s, has to offer on the mystery of the human mind.

最善の策は、 私が>>332で書いたアドバイスに従うことだと思うよ。
あと、たぶん勘違いしてる思うけど、あなたが262で「バカ高卒くん」と読んだ人と私は別人だよ。
319のテテンテンテンの人とも別人。

混同しないように、今後このスレに書き込むときは必ずコテつけるようにしときます。
とはいえ、書き込みたいことはほとんど無いんだけど
0483ブタ耳 (ワッチョイ 8fe3-e5cN)
垢版 |
2022/10/02(日) 18:36:29.32ID:OimBwqcD0
To his library in Cable House he will add volumes in which are contained what human knowledge,
in the 1820s, has to offer on the mystery of the human mind.

問題の箇所については既に>>267氏が部分的に訳しているね
「人知が人間の心の神秘について提供するために持っているもの」

この訳をそのまま採用して全体を訳してみる。
「彼はケーブルハウスにある蔵書に書物を加えることになる。その書物の中には、1820年代において、人知が人間の心の神秘について提供するために持っているものが収録されている」

自分なりの訳をしてみると、例えばこんな感じでどうだろうか? (ケーブルハウスって何のか、彼が誰なのか、分からないまま訳してるんだけど)
「ケーブルハウスにある彼の蔵書には新たに書物が加えられることになる。その書物には、人間の心という神秘に関して、1820年代において人類の知恵が提供しうるかぎりの知見が収められている」

「提供しうる限りの」というのはかなり踏み込んだ訳だけど、なんかwhatにはそんなニュアンスがあるように感じられる。このあたりのニュアンスはOED氏にお尋ねしてみたい。

さて言うまでもなく、以上の訳は >>298の②の線に沿って訳したもの。

>>482氏は、①の線に沿った和訳を提示できないまま、情ない捨て台詞を残して逃げて行ったようです。もう帰ってこなくていいですよ。
0484OED Loves Me Not, a lonely crushed old daydreamer w/Asperger's (ワッチョイ cf89-vqPj)
垢版 |
2022/10/02(日) 19:53:28.52ID:StQiaEX90
>>483
>>what human knowledge, in the 1820s, has to offer on the mystery of the human mind.
>>「提供しうる限りの」... なんかwhatには
>>そんなニュアンスがあるように感じられる。... OED氏にお尋ねしてみたい。
====

なるほど。言われてみて初めて気づいたけど、確かにそう訳せばぴったりくるよね。
もしも「提供しうる限りの」と訳さずに、単に「人間の知恵が提供できるものを」と
訳したら、それは what human knowledge has to offer じゃなくて、
something that human knowledge has to offer を訳したものだ、ということになってしまうもんね。
0485名無しさん@英語勉強中 (ワッチョイ cf89-vqPj)
垢版 |
2022/10/02(日) 19:57:19.90ID:StQiaEX90
>>483
僕も cable house って何のことかわからないまま素通りしてたけど、cable を使った
house ではなくて、Cable という名前の人物が発案した house だそうだね。
以下は引用。

Cable House
The Cable House is a Richardsonian Romanesque-style house near Michigan Avenue
at 25 E. Erie St. in Chicago, Illinois, United States. The house was built in 1886
by Cobb and Frost for socialite Ransom R. Cable.
0487ブタ耳 (ワッチョイ 8fe3-e5cN)
垢版 |
2022/10/02(日) 21:24:07.76ID:OimBwqcD0
>>486
原文見つけました。
https://www.onlinereadfreebooks.com/en/Waterland-623007/8

Cable Houseが本文で初登場するのは、たぶんこの一節:
In 1818 - when violence has ceased but there is no less hardship - Thomas fits out the ground floor of his house in Kessling as offices for his maltings and makes the second move of his life, from Kessling to Gildsey, to a grand residence, Cable House (which still stands), north of the market square, but a minute’s walk from the brewery, with a view down the narrow lane, which was known then and is still known, though it is a wide thoroughfare now with a Boots and a Woolworths, as Water Street.

>>485のCable Houseとは別物のようですね。
0488名無しさん@英語勉強中 (テテンテンテン MM7f-vqPj)
垢版 |
2022/10/02(日) 23:08:20.17ID:Mb+AtezSM
>>478
なにが教えることはやめただ、馬鹿中卒

このバカは
>>ヒント has O to offerではないw
と書いてるのに

雑談スレで
has O to offerという解釈が間違いだなんて
行ってないという基地外じみた言い訳をして逃走してるよ

お前が何か有益なことをだれかに教えたことがあるんのかよ、ごみ
もう涌いてくるな、カス
0489名無しさん@英語勉強中 (テテンテンテン MM7f-vqPj)
垢版 |
2022/10/02(日) 23:13:12.54ID:Mb+AtezSM
因みに
https://lavender.5ch.net/test/read.cgi/english/1662991859/481
中卒馬鹿は雑談スレで
突然カマ言葉になりましたwww
>>最初の生理的な直感てあるわよね
>>あたしの場合はけっこう当たり外れあるけどw

ホモ太郎か誰かの別垢で混乱してんのか?www
それとも、単に中卒馬鹿もホモなんかね?w
0490名無しさん@英語勉強中 (テテンテンテン MM7f-vqPj)
垢版 |
2022/10/02(日) 23:16:43.67ID:Mb+AtezSM
>>473
裏切るもなにも
俺は別に普通に対応するだけだぜ

インチキな言い訳をして逃走するような卑怯者のホモ太郎
に対するような対応はしまんけどね。
0491名無しさん@英語勉強中 (テテンテンテン MM7f-vqPj)
垢版 |
2022/10/02(日) 23:21:54.94ID:Mb+AtezSM
ホモ太郎wwww
本人が見たらブチ切れる呼称だろうけど
俺のことはあぼーんしてる(という設定)らしいから、無問題

他の人も、気に入ったら彼にこの呼称を使ってくれwww
0494名無しさん@英語勉強中 (スプッッ Sd1f-+pZx)
垢版 |
2022/10/02(日) 23:30:26.02ID:oJihcTnbd
>>483
よく見たらおめえ、他人の訳した日本語を弄り回してるだけじゃねえかよwww
それで騙されるのはお前を"凄まじいほどの学識"の持ち主だと勘違いしたアスペの爺さんだけだわなw
0497名無しさん@英語勉強中 (テテンテンテン MM7f-vqPj)
垢版 |
2022/10/03(月) 00:04:30.64ID:WeS1m1uZM
>>495
ぷ。
何か有益な情報を教えたことあんのかよと煽ったら
発狂して
1レスだけ挙げてきたwww
「あたし」も有益な情報1つあげてますよって
すごい、すごい

こんだけ常駐しててそれだけなの?英語と全く関係ないしwww
「ほれw」だってwwかわいいな(嘘、超キモい)
0498名無しさん@英語勉強中 (テテンテンテン MM7f-vqPj)
垢版 |
2022/10/03(月) 00:05:20.02ID:WeS1m1uZM
>>496
ホモ太郎はホモ太郎2号を見て自分の姿を見せつけられているようで気持ち悪くて仕方ないだろうなwww
0499名無しさん@英語勉強中 (テテンテンテン MM7f-vqPj)
垢版 |
2022/10/03(月) 00:07:45.41ID:WeS1m1uZM
>>482
>>もう高卒とはここで会話はしません!
>>サヨーナラーwwwwww

>>もう高卒とはここで会話はしません!
>>サヨーナラーwwwwww

>>もう高卒とはここで会話はしません!
>>サヨーナラーwwwwww

鶏なみの脳みそなのかな?www
3歩、歩いたら、全部忘れるみたいなwwww
0500名無しさん@英語勉強中 (テテンテンテン MM7f-vqPj)
垢版 |
2022/10/03(月) 00:21:21.08ID:WeS1m1uZM
>>468
テメーがカオスの一要因である事実に
露ほどもも思いめぐらすことのできない
無残な基地外
0501ブタ耳 ◆2wLzAFYxl2 (ワッチョイ 8fe3-e5cN)
垢版 |
2022/10/03(月) 01:24:13.92ID:rUG4S+jS0
>>483で提示した訳に少々推敲を加えました。

To his library in Cable House he will add volumes in which are contained what human knowledge,in the 1820s, has to offer on the mystery of the human mind.
「ケーブルハウスにある彼の蔵書には数々の書物が加えられることになる。その書物には、人間の心という神秘に関して、1820年代において人類の知恵が提供しうるすべてが収められている」

ここのwhatに関しては、例えば次のような場合の what のニュアンスと重なるものがあるように思います。

Don't worry. I'll do what (=anyting that) I can for you.
心配するな。できるだけのことはしてやろう。[江川、英文法解説 p.86]

先の英文中の to offer という不定詞には、この江川の例文中の can (do) と共通する「可能」のニュアンスがある。
このように両者に重なるものがあるとすれば、「すべて」と訳してしまっても、あながちやりすぎとは限らないと思いますがいかがでしょうか?

前後の文脈を読まずに翻訳するのは愚かというものなので、現段階でこれ以上この訳文を「弄り回す」のは止めておきます。
が、より正確な解釈があれば、どなたであれ是非ご教示ください。よろしくお願いします。
0505名無しさん@英語勉強中 (テテンテンテン MM7f-vqPj)
垢版 |
2022/10/03(月) 02:09:32.64ID:WeS1m1uZM
>>503-504
馬鹿中卒くん
念のため、聞くけど
それ、マジレスなん?wwwwwww
0506名無しさん@英語勉強中 (テテンテンテン MM7f-vqPj)
垢版 |
2022/10/03(月) 02:12:39.66ID:WeS1m1uZM
馬鹿中卒くんがまた生き恥を晒していますねwww
普通の神経してたら学力がマジで中卒レベルであることを自ら晒しても
恥ずかしくて二度と出てこれないのいのだろうが
こいつは基地外馬鹿だから、どうせ居座り続けて「バカ高卒」って連呼し続けるのだろうねw
さっさと自決すればいいのに
0507名無しさん@英語勉強中 (テテンテンテン MM7f-vqPj)
垢版 |
2022/10/03(月) 02:16:15.76ID:WeS1m1uZM
>>482 で本当に
>>サヨーナラーwwwwww しとけば
恥の上塗りをしなくて済んだのにねwwwwwwwwwwwwwwwwww

真正中卒(下駄を履かせてあげてますwww)馬鹿
0508名無しさん@英語勉強中 (テテンテンテン MM7f-vqPj)
垢版 |
2022/10/03(月) 02:23:51.72ID:WeS1m1uZM
>>506
訂正
×普通の神経してたら学力がマジで中卒レベルであることを自ら晒しても
〇普通の神経してたら学力がマジで中卒レベルであることを自ら晒したら

こいつの学力を中卒レベルと評するのは
全国の中学生に申し訳ない気がしますが
ご勘弁を
0509名無しさん@英語勉強中 (テテンテンテン MM7f-vqPj)
垢版 |
2022/10/03(月) 02:31:43.04ID:WeS1m1uZM
ここで予想しておきます
have to offer のときのように
中卒馬鹿くんは
「この I can for you と言い回しを、間違っているといったわけではない
ただ、テストとして答えられるかどうかブタ耳に聞いてみただけだ」
とかなんとか言い訳をし始めすよwwww
0510名無しさん@英語勉強中 (テテンテンテン MM7f-vqPj)
垢版 |
2022/10/03(月) 02:57:16.46ID:WeS1m1uZM
いくらなんでもさすがに釣りなのか?
と思ってメアド欄確認してけど、sageしか書いてないwwwwwwwwwwww
まぁでも、釣りだと強弁しそうwwwwwwwwwwwwwwww
0511名無しさん@英語勉強中 (ワッチョイ cf89-vqPj)
垢版 |
2022/10/03(月) 03:45:39.88ID:rYo0mENO0
>>501
>>先の英文中の to offer という不定詞には、この江川の例文中の can (do) と共通する「可能」のニュアンスがある。
>>このように両者に重なるものがあるとすれば、「すべて」と訳してしまっても、
>>あながちやりすぎとは限らないと思いますがいかがでしょうか?
============

>>484 でも書いたことに重なるけど、あなたの言うように、「すべて」という
ふうに訳してもいいような感じがする。
0513名無しさん@英語勉強中 (ワッチョイ f396-pIDl)
垢版 |
2022/10/03(月) 08:08:08.20ID:cWBVS2GD0
>>453
「小山(こやま)の大将」とは、このスレでは次のような皮肉的意味を持つ。

「こんなどうでもよい掲示板を山だと錯覚して(実際には山じゃなくちょっとした土盛りにすぎないが)、そんな貧弱な土盛りで一番であることを維持するために不当に他者の悪口を言っておとしめようとする様子。
0514名無しさん@英語勉強中 (ワッチョイ fff0-ufYE)
垢版 |
2022/10/03(月) 08:24:06.36ID:dwVI0Zb+0
スップ消えて
0515名無しさん@英語勉強中 (ワッチョイ f396-pIDl)
垢版 |
2022/10/03(月) 08:30:22.19ID:cWBVS2GD0
>>501 >>511
「anything=all」と定義づけてる英英辞書はあるのでしょうか?

She could be anything between 30 and 40.は、She could be any age between 30 and 40.の意味であり、「すべて」ではなく「どれでもあり」の意味のはずです。

Don't worry. I'll do anyting that I can for you.は、「心配するな。あなたのために、できることはどれでもやってあげるよ」が原義で、「ひとつ残らずやってあげるよ」の意味だとは思えませんが?
0516名無しさん@英語勉強中 (ワッチョイ f396-pIDl)
垢版 |
2022/10/03(月) 08:48:09.84ID:cWBVS2GD0
関係詞what には 「the thing(s) which」と「anything that」の2つの意味がありますが、「anything that」は「どれでも選択できる」という文脈でのみ使うべきだと思います。

従って、「what human knowledge ,in the 1820s, has to offer ~」は、the things whichで解釈するのが妥当だと思います。
0517名無しさん@英語勉強中 (ワッチョイ f396-pIDl)
垢版 |
2022/10/03(月) 09:03:44.66ID:cWBVS2GD0
「what human knowledge ,in the 1820s, has to offer ~」は、the things whichで解釈するのが妥当であることは明らかです。theがついているため「すべて」の意味が内在されているのだという反論はあるかと思いますが、これは関係詞で修飾されたためのtheであり、「すべて」のを意味するtheではないと思います。そもそも人類の叡智がすべて収められている本なんで存在するのでしょうか。抜けはまったくない本なのでしょうか? そんな神がつくったような完璧本なんて存在するのでしょうか?「すべて」と訳すことはその点からも不自然に感じます。

what most people like = the things which most people likeとあったときに、このthe thingsはtheがあるからと言って「すべて」と訳してよいものでしょうか?おかしいですよね。従って、the things whichは基本的に「すべて」と訳さないほうがよいのです。
0518名無しさん@英語勉強中 (テテンテンテン MM7f-o1nH)
垢版 |
2022/10/03(月) 09:11:21.63ID:koKziISdM
>>517

>>そんな神がつくったような完璧本なんて存在するのでしょうか?

human knowledgeが
in the 1820sに
0520名無しさん@英語勉強中 (ワッチョイW f396-7MMU)
垢版 |
2022/10/03(月) 09:27:59.11ID:rPAzCvhb0
単なるレトリックと捉えることも出来るかも
その中の誇張法が当てはまりそう
0522ブタ耳 (ワッチョイ 8fe3-e5cN)
垢版 |
2022/10/03(月) 09:34:28.72ID:rUG4S+jS0
安藤 「現代英文法講義」 p.195 から引用します
*****
what
[B] = anything that, as much as
(5) I will do what (= anything that) I can. (私にできることは何でもします)
(6) Give me what (= as much as) there is. (あるだけのものを全部ください)
*****

>>515
anything と all は異なるという指摘は重要ですね。
"what human knowledge ,in the 1820s, has to offer ~"のwhatは、上の引用例(6)に近いように感じますがいかがですか?

また、anything that と解釈した場合でも、
ある蔵書に、(a)ある年代の、(b)ある特定の分野に関して、「どんな情報でも」記載されているとすれば、
その蔵書には (a) と (b) の「すべての情報が」記載されていることになりませんか?
0523名無しさん@英語勉強中 (テテンテンテン MM7f-o1nH)
垢版 |
2022/10/03(月) 09:40:29.58ID:Z9g6ATHFM
「心の神秘について
その当時人類の英知が到達していた知識の全てが収められている群書」

という日本文が特に不自然だと思わないけど。

「全て」といったって、なにか抜けがあるはずだろ。神が作ったのではないのだから!
っていうツッコミをする人なんている?
0524ブタ耳 (ワッチョイ 8fe3-e5cN)
垢版 |
2022/10/03(月) 09:42:13.96ID:rUG4S+jS0
ロイヤル英文法からも引用しておきます p.647

*****
(4) what が anything that の意味を表す場合
この場合の what は whatever に置き代えることができる。
What I have is yours.
(私が持っているものははすべてあなたのものです)
Choose what you want for dinner.
(夕食に欲しいものをなんでも選びなさい)
*****

この二つの文のwhatには明らかに違いがあって、
前者は、「すべて」と訳せるのに対し、
後者は「夕食に欲しいものをすべて選びなさい」とすることはできませんね。
0525名無しさん@英語勉強中 (ワッチョイW 839d-3Mnr)
垢版 |
2022/10/03(月) 09:43:10.38ID:AyNi8PQQ0
100点満点だよって褒められて
完璧なんて世の中にあるんですか見せてください的な
0526名無しさん@英語勉強中 (テテンテンテン MM7f-o1nH)
垢版 |
2022/10/03(月) 09:45:01.98ID:Z9g6ATHFM
つか
I can for you !!!!!!!!!! について
誰も突っ込んでないの草

みんな優しいのか?
それとも
完全にゴミ扱いでスルーなのか?wwwwwwwwwww
中卒馬鹿キングカワイソース
0527名無しさん@英語勉強中 (ワッチョイ f396-pIDl)
垢版 |
2022/10/03(月) 09:45:38.07ID:cWBVS2GD0
関係詞whatは、the thing(s) that/whichの意味があると一般的に言われますが、このtheは関係詞で修飾されて特定のものをさす場合が多いのでtheがついているだけで、実は、theは必須ではありません。
「what= thing(s) that」(定冠詞なし)でも書き換えができます。


"know what will occur in the future" = "know things that will occur in the future"
 (未来のことで特定できない事柄なのでtheはつけてはいけない)

関係詞whatはtheの意味が内在されていない場合もあるのです。これからみても、whatを「すべて」と訳す危険性が理解できると思います。
0528名無しさん@英語勉強中 (テテンテンテン MM7f-o1nH)
垢版 |
2022/10/03(月) 09:50:29.49ID:Z9g6ATHFM
>>527
ここでは
そんなことは対して重要ではないので

>>523 に答えてもらえませんか?
0529ブタ耳 (ワッチョイ 8fe3-e5cN)
垢版 |
2022/10/03(月) 09:55:06.80ID:rUG4S+jS0
「は」が1個多かったので訂正

What I have is yours.
(私が持っているものはすべてあなたのものです)

ついでに安藤本の引用の続きも書いときます。
>>522の引用部分にそのまま続く部分で、他の文法書では「関係形容詞」に出てるような内容です。
前回の引用部分も含めておきます。

*****
what
[B] = anything that, as much as
(5) I will do what (= anything that) I can. (私にできることは何でもします)
(6) Give me what (= as much as) there is. (あるだけのものを全部ください)

この意味では, 限定詞的にも用いられる。
(7) Show me what books (= all the books that) you have.
(君がもっているどんな本でも見せてくれ)

little, few と共起すると, 「少ないながらも全部」という意味になる.
(8) I gave him what little money I had.
(わずかだがもっている金はそっくり彼に与えた)
*****

(7)の例文では、(all the books that)と置き代えておきながら、訳文は「どんな」となってるところがおもしろいですね。
0530ブタ耳 (ワッチョイ 8fe3-e5cN)
垢版 |
2022/10/03(月) 10:01:21.32ID:rUG4S+jS0
あれ? 今引用した後に気づいたんだけど、

(7) Show me what books (= all the books that) you have.
(君がもっているどんな本でも見せてくれ)

これって、what を疑問形容詞として、「君がどんな本をもってるのか見せてくれ」と解釈した方が自然じゃないのかね。
0531名無しさん@英語勉強中 (ワッチョイ f396-pIDl)
垢版 |
2022/10/03(月) 10:04:05.98ID:cWBVS2GD0
>>525
Give me what (= as much as) there is. なら、文脈的に理解できますね。ただし、問題となってる文の文脈では、難しいと思います。
なぜなら、「人類の叡智が心の神秘さに寄与するもの」ってかなり抽象的で「これですべてだ」と提示できる性質のものではないからです。

>「どんな情報でも」記載されている」
これは、「anything」ではなく、「everything」で表現しなければなりません。anythingにはallの意味はないのですから。

>>523
>知識の全てが収められている群書
「人類の叡智が心の神秘さに寄与するもの」ってかなり抽象的で「これですべてだ」と提示できる性質ものではないのだから、こんなこと言ったら誇大広告で訴えられるよ

>>524
>この場合の what は whatever に置き代えることができる。
「whatever = everything that」と定義づけてる英英辞書はないですよね?
What I have is yours.も原義は、「私が持っているものはどれでもあなたのものにしてよい」ということで、選択の意味は消えていないのです。
0532ブタ耳 (ワッチョイ 8fe3-e5cN)
垢版 |
2022/10/03(月) 10:09:32.63ID:rUG4S+jS0
>>531
>また、anything that と解釈した場合でも、
>ある蔵書に、(a)ある年代の、(b)ある特定の分野に関して、「どんな情報でも」記載されているとすれば、
>その蔵書には (a) と (b) の「すべての情報が」記載されていることになりませんか?

この理路についてはどうお考えですか?
0533名無しさん@英語勉強中 (ワッチョイ f396-pIDl)
垢版 |
2022/10/03(月) 10:10:02.09ID:cWBVS2GD0
>>529
確かに whatが名詞を修飾するとき、すべてので意訳するとうまく行く場合があります。

しかしながら、I gave him what little money I had. は、日本の参考書で「なけなしのお金をすべて」と書いてあるけど、英英辞典で「what little」を引いても、all の意味は載ってないんですよね。

英英辞典で「what little」をallで解釈してるのってあるでしょうか?
0535名無しさん@英語勉強中 (テテンテンテン MM7f-o1nH)
垢版 |
2022/10/03(月) 10:12:18.66ID:24Kh8OY6M
>>531
そうか・・・
「すべて」という言葉を使うときは
万が一抜けがあったら訴えられる可能性があるから
使ってしまったら戦々恐々としていなくてはならないのかぁw
0536名無しさん@英語勉強中 (テテンテンテン MM7f-o1nH)
垢版 |
2022/10/03(月) 10:35:56.83ID:uBn25MqrM
結局この人の主張をまとめると

「「人類の叡智が心の神秘さに寄与するもの」ってかなり抽象的で「これですべてだ」と提示できる性質のものではない」
から
「すべて」という訳語を使うのはおかしい!
ってことのようです。

申し訳ないけど、くだらない。
別にそんな数学的な厳格さで、すべての言語活動が行われるなんてことはありません。
0537ブタ耳 (ワッチョイ 8fe3-e5cN)
垢版 |
2022/10/03(月) 10:44:17.42ID:rUG4S+jS0
>>534
ランダムハウス英和大辞典
anything
【2】((肯定文)) なんでも,どんなもの[こと]でも

この日本語を採用して、先の命題を修正
>また、anything that と解釈した場合でも、
>ある蔵書に、(a)ある年代の、(b)ある特定の分野に関して、「どんなもの[こと]でも」記載されているとすれば、
>その蔵書には (a) と (b) の「すべてのもの[こと]が」記載されていることになりませんか?

この理路は誤謬を含んでいますか?

また、一般に、現実には存在しえないもの(例: 全ての情報が記載された蔵書)であったとしても、そういうものが存在すると述べる言語表現が不可能であることにはなりません。
分かりやすい例でいえば、「何でも知っている人間」など現実には存在しえないのに、「あいつは何でも知っている」という言語表現は可能です。
0538名無しさん@英語勉強中 (テテンテンテン MM7f-o1nH)
垢版 |
2022/10/03(月) 11:12:09.78ID:uBn25MqrM
しかもさぁ
前後の文脈を把握してるわけじゃないけど
この英文は
「これが全部だ」って言ってるわけじゃなくて
「これから、その年代における、全部の知識を網羅してるような群書を収集しよう」
って言ってるんじゃねーの?

誇大広告になるわけねーじゃん。
0539名無しさん@英語勉強中 (ワッチョイ f396-pIDl)
垢版 |
2022/10/03(月) 12:19:49.64ID:cWBVS2GD0
>>537
その日本語は意訳です。英英辞典を使わないと真意は見えないでしょう。
「anything that」を、「evertything that」とかallで書き換えられると言っている英英辞書は存在しません。

anythingは「どれでも」と解釈すべきで、本来は「all」じゃないことの説明↓
Anything will do. (どれでもいいよ。すべては欲していないけど。)
I will do anything I can do to help.<ジーニアスからの例文> (どれでもやるよ。なにがして欲しいか選んでね)
The store is about anything. (その店は、どれが欲しい場合でも、役にたつよ。)

つまり、何かを選択する文脈で使うのです。
0540名無しさん@英語勉強中 (ワッチョイ f396-pIDl)
垢版 |
2022/10/03(月) 12:20:14.95ID:cWBVS2GD0
□□□□□□□□□□
私の主張を整理します。
□□□□□□□□□□

「what human knowledge has to offer on 人心の神秘」のwhatは、「things that」の意味であり、「anything that」の意味ではない理由は次のとおりです。

肯定平叙文でのanythingは「どれでも」という意味であり、「all」「everything」と交換できません。そのため、必ず「どれでも選択できるよ」という含みがあるのです。anythingを「all」「everything」で説明している英英辞書はありません。

「人類の英知が人心の神秘に関して寄与するもの」は、抽象的で「これがすべてだ」と表現するのにふさわしくないものです。使う人もいるでしょうが、私は、もし誰かが「この本には人類の英知が人心の神秘に関して寄与しているすべてのものが含まれているんだ」と言ったなら、その人を誇張癖・詐欺師的傾向のある人だとみなすことでしょう。宗教団体が使いそうな表現ですね。

問題文のwhatを「すべて」と訳さなければならない必然性がありません。

以上のことから、このwhatを「すべて」と訳すのは問題があります。
0541ブタ耳 (ワッチョイ 8fe3-e5cN)
垢版 |
2022/10/03(月) 12:42:03.47ID:rUG4S+jS0
>>539
anythingは「どれでも」と解釈すべきとおっしゃってますが、
ランダムハウスの「なんでも,どんなもの[こと]でも」は、それとは別の「意訳」だということなんですか?

ちょっとよく分からない。
0542ブタ耳 (ワッチョイ 8fe3-e5cN)
垢版 |
2022/10/03(月) 13:14:43.07ID:rUG4S+jS0
>>533
>英英辞典で「what little」をallで解釈してるのってあるでしょうか?

what little ではなく ただの what ですが、Longman 4th Ed.に次の例文が出ていました。

・She gave him what money she had (=all the money she had, although she did not have much) .


本日の書き込みは以上です。
cya
0543ブタ耳 (ワッチョイ 8fe3-e5cN)
垢版 |
2022/10/03(月) 13:20:35.84ID:rUG4S+jS0
あ、今の引用でちょっとひらめいたので追記

Longmanの例文、littleついてないのにalthough she did not have muchって解釈してるじゃないですか。

ということは件の例文も、
「1820代に人の心という神秘について書かれたものなんてわずかしかなかったんだけど、それを全部」
というニュアンスだった可能性があるのでは?
考えすぎかなー
0548名無しさん@英語勉強中 (テテンテンテン MM7f-o1nH)
垢版 |
2022/10/03(月) 15:10:50.70ID:uBn25MqrM
>>544-547
よく涌いてこれたな、基地外
何にせよ、このバカ中卒が言ってることの反対が概ね正しいですw
0549名無しさん@英語勉強中 (テテンテンテン MM7f-o1nH)
垢版 |
2022/10/03(月) 15:16:47.30ID:uBn25MqrM
>>540
話が逆転してんだよw
そもそものテーマがwhatを「全て」と解釈でき得るか、或いは、すべきかという
話なのだったら・・・
anythingを、all と説明してる英英辞典が無いとか、とりあえず関係ない

whatのこの用法のことだという説明で十分
(6) Give me what (= as much as) there is. (あるだけのものを全部ください)
0550名無しさん@英語勉強中 (テテンテンテン MM7f-o1nH)
垢版 |
2022/10/03(月) 15:27:40.24ID:uBn25MqrM
Don't worry. I'll do what (=anyting that) I can for you.
という英文への中卒馬鹿くんの反応をおさらいw

>すまん、I can for you とは?wwwwwwwwww

>高卒英文法
>I can for you !!!!!!!!!!
>WWWWWWWWWWWWWWWWWWWWWWWWWWWWWWWWWWWWWWWWWWWWWWWWWWWWWWWWWWWW


中卒馬鹿くんが登場するたびにこれをコピペしてレスしますので
あしからずwwwwwwww
0551名無しさん@英語勉強中 (スプッッ Sd1f-+pZx)
垢版 |
2022/10/03(月) 17:01:59.55ID:NkDFVmqfd
>>548
>何にせよ、このバカ中卒が言ってることの反対が概ね正しいですw

高卒くん2号はやたら感情的だよねwww
高卒って言われてからずっーとそんな調子w
ハイハイwww
0552名無しさん@英語勉強中 (テテンテンテン MM7f-o1nH)
垢版 |
2022/10/03(月) 20:10:51.31ID:uBn25MqrM
>>551
つか、なんでお前涌いて出てこれるの?w
0554名無しさん@英語勉強中 (テテンテンテン MM7f-o1nH)
垢版 |
2022/10/03(月) 20:43:39.66ID:uBn25MqrM
>>553
ああ、確かに at a trp を
形容詞的用法で補語だと書いたけどそれがどうしたの
中卒馬鹿w

>>432
でもホモ太郎に反論してるけど
なんか再反論でもあるの、ゴミw
あいつは逃走してけど、何かあるならどうぞw

これ以上、恥の上塗りをせずに
さっさと自決しろよ、カス
0555名無しさん@英語勉強中 (テテンテンテン MM7f-o1nH)
垢版 |
2022/10/03(月) 20:47:25.87ID:uBn25MqrM
Don't worry. I'll do what (=anyting that) I can for you.
という英文への中卒馬鹿くんの反応をおさらいw

>すまん、I can for you とは?wwwwwwwwww

>高卒英文法
>I can for you !!!!!!!!!!
>WWWWWWWWWWWWWWWWWWWWWWWWWWWWWWWWWWWWWWWWWWWWWWWWWWWWWWWWWWWW

この板で、お前の学力は真意外なく最底辺なんだよw
お前にレスする権利はねーんだよ池沼
のこのこ涌いてくるなよ、ゴミ
0557名無しさん@英語勉強中 (スップ Sd1f-3ust)
垢版 |
2022/10/03(月) 20:58:28.41ID:buPuY3qFd
>>555
上の方に書いてやったろ
夜中の2時くらいに泥酔しながらスレ見てて
I can for you.
しか目に入らなかった
単独の文として書いてるのかと思ったんだと思う
しょせんは脊髄反射だよ

まあそもそも俺は素面であってもバカ高卒くんとか高卒くん2号の書いたレスをまともに読んだことはないけどもwwwwww
0558名無しさん@英語勉強中 (テテンテンテン MM7f-o1nH)
垢版 |
2022/10/03(月) 21:05:58.59ID:uBn25MqrM
>>556
客観的事実を書いているだけですwww
残念w

必死にバカが言い訳してるwww

Don't worry. I'll do what (=anyting that) I can for you.
の’I can for you’という部分を
単独の文だと書ているのだと思ったってwwww

誰がそんなバカげた言い訳を信じると思ってんのw
言い訳の内容が小学生がいいそうな、異常な馬鹿さ加減wwww

涌いてくるなゴミムシ
0561名無しさん@英語勉強中 (テテンテンテン MM7f-o1nH)
垢版 |
2022/10/03(月) 21:11:21.54ID:uBn25MqrM
しかも
単独の文だとしたって
文脈によっては
重複を避けるための省略はあり得るのだから

泥酔してた(嘘w)とかって言い訳してるけど
本当だと仮定したって
意識なくなるレベルでなく、5ch見て投稿できるレベルでなら
省略かな?とふつうは疑う。

馬鹿だからそれができないだけwwwwwwwwww
0562名無しさん@英語勉強中 (テテンテンテン MM7f-o1nH)
垢版 |
2022/10/03(月) 21:12:34.68ID:uBn25MqrM
>>559
馬鹿www
お前が得するために嘘ついてるだろwww
基地外

お前の小学生レベルの学力を隠そうとしてんだろーが

やっぱ、基地外なのかな?
0563名無しさん@英語勉強中 (テテンテンテン MM7f-o1nH)
垢版 |
2022/10/03(月) 21:14:44.59ID:uBn25MqrM
>>559
>>お前もビール2リットルくらい飲みながら5ちゃんやってみろよwww

ぷ、夜中にビール2リットル飲みながら(嘘www)
5chだってwwwwwww

アル中だろwww
馬鹿で、アル中で、ホモって・・・・wwwwww
0564名無しさん@英語勉強中 (スップ Sd1f-3ust)
垢版 |
2022/10/03(月) 21:15:28.71ID:buPuY3qFd
>>561
お前のいうふつうとやらで何でも推し量れると思うところが高卒w
本当に泥酔してた
てかほぼ毎晩飲んでますけどw
毎晩泥酔してるわけではないけど夜中はたいてい外か内かで呑んでるね
0567名無しさん@英語勉強中 (テテンテンテン MM7f-o1nH)
垢版 |
2022/10/03(月) 21:20:59.65ID:uBn25MqrM
中卒馬鹿、お前の学力は本当は小学生レベルでないということを証明するチャンスを
与えてやるよwww

He is on a trip.

on a trip を形容詞的用法だとするのは間違いだと言っている。
https://lavender.5ch.net/test/read.cgi/english/1663090161/505

その根拠をホモ太郎に代わって書いてみろ。
お前が、間違いだと判断している根拠を書いてみろ。

馬鹿だからどうせ逃走するだけなのはわかってて
言ってるんだけどねwwwwwwwwwwwwwwwwwwwwwwwww
0570名無しさん@英語勉強中 (テテンテンテン MM7f-o1nH)
垢版 |
2022/10/03(月) 21:25:32.50ID:uBn25MqrM
>>565
普通じゃないね
お前のいう「普通」とやらで
なんでも推し量れると思うな、中卒

飲み屋でそのくらい飲むことは
俺にだって普通にあるけど

夜中の2時に
5chみながら
ビール2リットルwwwwwwwww

アルコール依存症の可能性があるから
病院行ってこい、池沼
まぁ嘘なんだろうけどw
0571名無しさん@英語勉強中 (テテンテンテン MM7f-o1nH)
垢版 |
2022/10/03(月) 21:26:57.71ID:uBn25MqrM
>>569
知ってますw

>>567
>馬鹿だからどうせ逃走するだけなのはわかってて
>言ってるんだけどねwwwwwwwwwwwwwwwwwwwwwwwww
0574名無しさん@英語勉強中 (テテンテンテン MM7f-o1nH)
垢版 |
2022/10/03(月) 21:29:33.49ID:uBn25MqrM
>>572
「飲み屋」でなんで笑ってるのか意味不明w

基地外だから、無意味に笑うのだろうけどw
0575名無しさん@英語勉強中 (テテンテンテン MM7f-o1nH)
垢版 |
2022/10/03(月) 21:31:23.30ID:uBn25MqrM
>>573
夜中に2リットルのビールを飲みながら
5chでアホを晒してる低学歴が

引きこもりがどうとか言ってて草
0576名無しさん@英語勉強中 (テテンテンテン MM7f-o1nH)
垢版 |
2022/10/03(月) 21:34:25.69ID:uBn25MqrM
これからなんでも
アホを晒したら
アルコールのせいにするのだろうね
良かったねw
0577名無しさん@英語勉強中 (テテンテンテン MM7f-o1nH)
垢版 |
2022/10/03(月) 21:34:26.48ID:uBn25MqrM
これからなんでも
アホを晒したら
アルコールのせいにするのだろうね
良かったねw
0578😉三年英太郎🌈 ◆3CZBjOt3.Y (ワッチョイW ff86-nigo)
垢版 |
2022/10/03(月) 21:39:42.47ID:Y7LX6w9y0
ブタ耳にしないってテテンじゃないの?
2019年の8月はバカンスでおドイツ🇩🇪にいたから記憶にないな😎

それは別として、長文の方が正論とゆーことはないんで(コウビルドの中の人の名誉を守るためにも)、一言しておきました😎
0580名無しさん@英語勉強中 (テテンテンテン MM7f-o1nH)
垢版 |
2022/10/03(月) 21:42:42.43ID:uBn25MqrM
>>579
淋病のホモ太郎さま
違いますけど
0582名無しさん@英語勉強中 (テテンテンテン MM7f-o1nH)
垢版 |
2022/10/03(月) 21:45:34.93ID:uBn25MqrM
ホモ太郎も馬鹿中卒も、馬鹿で卑怯でホモだから
同一人物だと思ったこともあったけど

さすがに違うな
馬鹿中卒の低能ぶりは群を抜いてるもんな
0586名無しさん@英語勉強中 (スフッ Sd1f-3ust)
垢版 |
2022/10/03(月) 21:53:14.87ID:lwbh2Robd
>>578
そういや英太郎さんのドイツの知り合いの人の話面白かったなあ
ゲーテの文章の格を今の若いドイツ人はよう読み書きしないで前置詞使うことの方が普通になってきてるって話(俺のこの理解の通り英太郎さんが書いてたわけではないから勘違いしてたらごめん)
0587名無しさん@英語勉強中 (テテンテンテン MM7f-o1nH)
垢版 |
2022/10/03(月) 21:54:53.24ID:uBn25MqrM
>>585
多くのIDを全て同一人物だという妄想
で発狂してたスプーキー婆という基地外が英語板に過去にいたんだが

お前は大丈夫なんか?www
0588名無しさん@英語勉強中 (スフッ Sd1f-3ust)
垢版 |
2022/10/03(月) 21:58:45.02ID:lwbh2Robd
>>587
一つ教えておいてやろう
バカ高卒くんとは他の複数の板でも交流してる
英語板でバカ高卒くんを俺が「発見」したのはここ最近のことだ
いろんなスレがおかしいしアホなスレ立ってるやろ?
あれの多くはバカ高卒くんによるものだ
バカ高卒くんの暇さ加減と粘着加減を、あまり舐めない方がいい
0590名無しさん@英語勉強中 (テテンテンテン MM7f-o1nH)
垢版 |
2022/10/03(月) 22:01:52.26ID:uBn25MqrM
>>588
そんなくだらんこと別に教えてもらわなくて結構です
それより形容詞的用法の件を、逃げずに説明しろよ、ボケ
0591名無しさん@英語勉強中 (テテンテンテン MM7f-o1nH)
垢版 |
2022/10/03(月) 22:10:34.46ID:uBn25MqrM
>>588
つか、ほぼ「バカ高卒」って貶すだけの
無意味な内容のやりとりを
他の板でもやってんの?
「交流」だったwwwwwwwww

英語板だけ見ても
俺と同レベル(w)の粘着ぶりなのに

複数の板でwwwwwww

夜中2時に2リットルのビールを飲みながらw

お前が引き篭もりだろw
アル中引き篭もりwwwwwwwwwwwwwwwww
0592ブタ耳 (ワンミングク MM9f-ZYgd)
垢版 |
2022/10/03(月) 22:12:04.20ID:vWDYft/uM
>>588
それもしかして俺のこと言ってたりする?
雑談スレに書き込んだことは一度も無いよ。
たぶん人違いだから、俺のことは高卒3号とでも呼べばいいんじゃね?
0593名無しさん@英語勉強中 (ワッチョイ f396-pIDl)
垢版 |
2022/10/03(月) 22:15:30.96ID:cWBVS2GD0
539のThe store is about anything. という英文は、不適切な英文でした。取り消します。

>>542
whatが名詞を修飾する時、確かに「すべて」と訳せる場合があるのは認めます。参考書に載ってますし。しかし、今回の英語は名詞を修飾してないのだし、なぜそんなものを無理に当てはめようとするのでしょうか?
「things that」で解釈することになにか不都合なことがあるのでしょうか?そこが不思議です。不都合がないなら、一番オーソドックスな「things that」を適用させるべきでしょう。

>>549
「すべて」と訳さなければならない必然性が今回の英語にはないのです。「このwhatの意味がas much asだ」とする論拠は何でしょうか?
そもそも、「人知が心の神秘に関して寄与するすべてがこの本に載ってるよ」なんて怪しい闇業者が使うような台詞を、信頼性の高い筆者が使うワケがないでしょう。「すべて」を使うと文の質が低下するのがわかりませんか?
0594名無しさん@英語勉強中 (テテンテンテン MM7f-o1nH)
垢版 |
2022/10/03(月) 22:15:56.79ID:uBn25MqrM
>>592
 
やっぱ、中卒馬鹿って
見えない敵と戦ってんだろうね
スプーキー婆と完全に同類の基地外なんだろう
量産型基地外wwwwww
0596名無しさん@英語勉強中 (テテンテンテン MM7f-o1nH)
垢版 |
2022/10/03(月) 22:17:42.63ID:uBn25MqrM
>>593
「人知が心の神秘に関して寄与するすべてがこの本に載ってるよ」

そんなこと書いてないけど。
0598名無しさん@英語勉強中 (テテンテンテン MM7f-o1nH)
垢版 |
2022/10/03(月) 22:21:27.73ID:uBn25MqrM
>>595
お前の言ってる煽り(?)言葉が
むしろお前に当てはまるのだから
嫌味でパクるに決まってんだろw
そっくりも何も多数の人がやってんだけどなw

こういう推論、
(「こんなことするなんて、〇〇にそっくり」)

全部同一人物扱いしてんじゃねーだろうなぁwwww
とか
0601名無しさん@英語勉強中 (テテンテンテン MM7f-o1nH)
垢版 |
2022/10/03(月) 22:27:05.37ID:uBn25MqrM
>>596
volumesに定冠詞はついてません
そして複数形になってます

それから
will add となってるよね

本は目前にあるわけではない
全部の知識を網羅できるように
複数の本を
これから集めようという
彼の意志をあらわしてるのだと思うのだけど

怪しい闇業者って何のこと?
0603名無しさん@英語勉強中 (テテンテンテン MM7f-o1nH)
垢版 |
2022/10/03(月) 22:29:41.59ID:uBn25MqrM
>>599
揚げ足とりじゃねーだろ、アホ
「怪しい闇業者が使うような台詞を、信頼性の高い筆者が使うワケがない」
って相手が主張してきてんだろ。

だから、お前は馬鹿なんだから涌いてくるなと言ってんだろ、ゴミ
日本語わからないあるか?
0604名無しさん@英語勉強中 (テテンテンテン MM7f-o1nH)
垢版 |
2022/10/03(月) 22:30:53.16ID:uBn25MqrM
>>599

>>596 の補足説明ですけどw
お前は本当に一々レスの内容が間違ってんだよw
基地外
0605名無しさん@英語勉強中 (テテンテンテン MM7f-o1nH)
垢版 |
2022/10/03(月) 22:32:05.04ID:uBn25MqrM
>>599
じゃなく
>>602 でした。
失礼
0607名無しさん@英語勉強中 (テテンテンテン MM7f-o1nH)
垢版 |
2022/10/03(月) 22:37:04.48ID:uBn25MqrM
俺はホモ太郎と違ってあぼーんなんてしたことなかったけど
さすがに中卒馬鹿はあほーんするわ
基地外の上に尋常じゃない馬鹿で、レスの内容が一々間違っていて
それで
「お前は間違いというが何故だ?」と聞いても
「なんで教えなくちゃいかんの?」としか言わねーもんw

さすがに、全く読む価値が皆無のゴミだもんねw
これ以上のゴミはないだろうね。

じゃあな、ゴミ
0608名無しさん@英語勉強中 (スッップ Sd1f-+pZx)
垢版 |
2022/10/03(月) 22:37:40.70ID:vZ/WvXkJd
>>603
それはその人が人間の全ての知識、なんて言葉遣いはいんちき商売や宗教のやうだと主観的に考えているからでお前がとやかく言うことじゃない
それから~あるか?とか品性が下劣すぎて語学板に相応しくないから、今すぐに立ち去りなさい、高卒
0610名無しさん@英語勉強中 (テテンテンテン MM7f-o1nH)
垢版 |
2022/10/03(月) 22:39:03.63ID:uBn25MqrM
>>606
普通の読解力があればわかるから
大丈夫なんだよ ゴミ
じゃあ、これからあぼーんするから
失礼するね

これが最後のレスになるよ
元気でな(嘘、さっさと自決しろw)
0612名無しさん@英語勉強中 (ワッチョイ f396-pIDl)
垢版 |
2022/10/03(月) 22:44:15.89ID:cWBVS2GD0
いずれにしても、「what human knowledge has to offer ~」のwhatをanything thatであるとするのは、誤りです。
anythingは、肯定平叙文では、「なんでも(どれでも)」であって、「すべて」という意味じゃないのですから。

You can buy anything at this store. は、決して「すべてのものを買う」という意味ではありません。
0613名無しさん@英語勉強中 (ワッチョイ f396-pIDl)
垢版 |
2022/10/03(月) 23:10:50.31ID:cWBVS2GD0
>>597
素晴らしい反論でしたね! 分かりやすくしてみました。

What human knowledge has to offer on the mystery of the human mind are contained in the books.」という英文で、
whatがanything that や as much asの意味を持つなら、areはおかしい。

「Anything are contained」とか「Much are contained」と言えない以上、このwhatは、anything thatやas much asで置き換えてはいけない。
0615名無しさん@英語勉強中 (テテンテンテン MM7f-o1nH)
垢版 |
2022/10/03(月) 23:36:57.31ID:uBn25MqrM
>>613
意味的に置き換え可能とされている
anything thatやas much asについての話をしてるのですか?
意味的に置き換え可能
でも文法的な性質が全て同じになるなんてことは
安藤氏も意図してないでしょう。

実際
whatが全部という意味合いで使われるケースでも
areで受けることがあるという
アンケートがありますよね。
0616名無しさん@英語勉強中 (テテンテンテン MM7f-o1nH)
垢版 |
2022/10/03(月) 23:47:47.22ID:uBn25MqrM
>>245

まぁこれは第二文型で、補語が複数形の場合だけど・・・

でも、話者が複数のものだという感覚があれば、areで受けることもあるのでは?

確か、中卒馬鹿も、話者の認識次第とか言ってなかったか?wwww
0617名無しさん@英語勉強中 (テテンテンテン MM7f-o1nH)
垢版 |
2022/10/03(月) 23:59:00.00ID:uBn25MqrM
>>616
補足w バカが理解できないみたいなのでね

一つ目のis/areについては
「第二文型で、補語が複数形」ではなく
当該英文と同様に受動態ですね。

まぁでも主節の複数形の補語の影響が皆無とは言えないでしょうが・・・
0619名無しさん@英語勉強中 (テテンテンテン MM7f-o1nH)
垢版 |
2022/10/04(火) 00:35:14.65ID:CKtn0scHM
allが「全て」という意味合いで使われている可能性について
文法的に不可能ではないという説明をしたけど

意味的に
>>「すべて」と訳さなければならない必然性
についてだけど

whatを修飾する節が、義務とか必要性の意味合いがあるときは、そのように解釈するのが
なんとなく意味が通るように感じるから

例えば
what is(are) needed
what you should do
とか

当該英文は
what have 空所 to do という形で、このto do は「〜すべき」という意味でしょう。
(馬鹿中卒にも言ったけど、この構文がhave to do という言い回しの語源)

「whatを修飾する節が、義務とか必要性の意味合いがあるときは、そのように解釈するのが
なんとなく意味が通るように感じるから」
では、あまりにあいまいな根拠であることは自認していますので
もう少し具体的なことがわかったら、今後説明します。
まぁ間違ってるかもしれないけど。
0620ブタ耳 (ワッチョイW 8fe3-ZYgd)
垢版 |
2022/10/04(火) 01:14:51.36ID:xGYTum6p0
>>619
そのto do には「〜することができる」という「可能」の意味合いがあって、それが「すべて」という解釈を導く要素なんじゃないか、と自分は考えてる。
更に、いくつかの限定的な語句。「1820年代」とか。

あと必要なのは結局「前後の文脈」かな。
1820年代とはどんな時代なのか、heがどんな人物なのか、何のために本を集めてるのか、これくらいは知っておかないと、あの一文の正確なニュアンスはおそらく把握できない。
自分の予想としては、「1820年代という困難な(?)時代に、手に入る情報はわずかしかなかったんだけど(?)、それをできる限り全部集めようとした」というニュアンスかなー、と。
これは考えすぎかもねー。

暇をみてあの一文の前後の文章を確認してみるつもり。
0622名無しさん@英語勉強中 (テテンテンテン MM7f-o1nH)
垢版 |
2022/10/04(火) 03:05:36.49ID:CKtn0scHM
眠れない・・・

当時の人類の英知が提供すべき情報
当時の人類の英知が提供できる情報

「〜すべき」も「〜できる」も意味合いは連続的ですよね。
0626名無しさん@英語勉強中 (テテンテンテン MM7f-o1nH)
垢版 |
2022/10/04(火) 08:44:40.03ID:wf+qc7DeM
>>624
>>おれを不当扱いした以上、本来は説明責任あるだろう

テメーがそんなこと人に言える立場か、カス
0628名無しさん@英語勉強中 (テテンテンテン MM7f-vqPj)
垢版 |
2022/10/04(火) 10:19:19.55ID:zgd/DGhWM
Lost and Found what can i do for you?

1 Yes, that is so
2 I lost my way
3 May I ask your phone number?

toeicのリスニングらしいのだが、これの回答が納得できなかった。
ちなみに回答は…いったん伏せさせてほしい。
ぶっちゃけ回答のやつ、受け答えがあってなくない?と思うんだが誰か解説してほしい。
0629ブタ耳 (ワッチョイ 8fe3-e5cN)
垢版 |
2022/10/04(火) 10:26:20.25ID:xGYTum6p0
>>624
「おれを不当扱いした」というのは>>391の私のレスのことですね。
確かに、「圧倒的にOED氏が正論を述べていると思う」と述べたということは、OED氏の意見に与しない人を不当扱いしたことになりますね。

challengeの用法に関する議論はこのスレで正確にフォローしてきませんでした。
今更議論の流れを最初からたどるのは私も「めんどくさい」です。(what関連で手一杯なので)
もっとも、OED氏は3年前からあなたをブロックしているということですので、「議論」は最初から成立していなかったのかもしれませんが。

>>391で私が述べた感想は、主に、「OED氏が2つの辞書に懇切丁寧な問い合わせをし、そのやり取りの一部始終を正確に紹介してくれた」ことに関するものです。
この労力に対して何の感謝もしないあなたのレス、例えば>>380-384に見られるような嘲笑的態度に対しては、これを「不当」と非難してもよかろうと今でも考えています。

一方、繰り返しになりますが、OED氏についても「肉便器」は言い過ぎでした。

二人ともいがみあわずに仲良くやればいいじゃないか、と以前名無しでレスしたことがありましたが、たぶんお忘れでしょうね。
傍からそんな提案をすることの空しさを痛感せざるを得ませんが、二人が罵り合ってるこの状況は英語スレにとって大きな損失なんじゃありませんか?

ん? 罵り合いこそ5ちゃんの華、という声がふと聞こえてきた気がしましたが、たぶん気のせいです。
0630ブタ耳 (ワッチョイ 8fe3-e5cN)
垢版 |
2022/10/04(火) 10:31:47.60ID:xGYTum6p0
>>628
A: こちら遺失物取扱所です。どうされましたか?
という意味だと思うけど、適切な応答がないような…

2が I lost my wallet. とかの誤植だったりして?
0631名無しさん@英語勉強中 (オッペケ Sr47-i+A0)
垢版 |
2022/10/04(火) 10:57:14.18ID:nRi1Y/sHr
置き忘れたマイウェイのDVDを探してるとか
0632名無しさん@英語勉強中 (テテンテンテン MM7f-CuO3)
垢版 |
2022/10/04(火) 11:55:25.05ID:d8vcI8/PM
英作文の教科書(和文英訳の修業)の練習問題に次のものがありました.
問 私が病気になると母はまるで自分のことのように苦しみました.
解答 When I was ill, mother suffered as if my illness were her own.

後半の部分ですが,過去の事実に反することを書いてあるので,as if my illness had been her ownとなりそうな気がします.このような場合は,解答にあるようにwere とするのが正しいのでしょうか?その場合,文法的にはどう理解するのが良いでしょうか?

コメントいただけると助かります.
0633名無しさん@英語勉強中 (テテンテンテン MM7f-vqPj)
垢版 |
2022/10/04(火) 12:02:43.51ID:zgd/DGhWM
>>630
やっぱそう思うよね
ちなみにこれよくわからんyoutubeチャンネルで流れた問題の一つ。
結構誤字もあったので、適当なのかもしれないと感じたけど、これも問題文自体が違ってるのかな
0634ブタ耳 (ワッチョイ 8fe3-e5cN)
垢版 |
2022/10/04(火) 12:17:02.23ID:xGYTum6p0
>>633
そのYouTubeのソースを提示してくれないと判断できないな。
どれを正答として、どんな解説をしたんですか?

まあ、問題自体が間違ってて、解説も間違ってた、という可能性が高いようにと思うけど。
0637名無しさん@英語勉強中 (ワッチョイ a344-H63Z)
垢版 |
2022/10/04(火) 14:43:30.63ID:BNcX6Oxx0
>>628
答え:1
遺失物取扱所です。どうされましたか?

あ、はい、それです(指さしながら)
0638ブタ耳 (ワンミングク MM9f-ZYgd)
垢版 |
2022/10/04(火) 14:51:20.50ID:jcyuTApGM
>>636
敵意に満ちた曲解をするとそういうことになりますね。
謝罪はしないでおきます

ついでながら、自分が激しく同意してヘッドバンギングしたのはOED氏の>>379の書き込みです。
0640三年英太郎 ◆3CZBjOt3.Y (ワッチョイ ff86-7uza)
垢版 |
2022/10/04(火) 15:04:17.99ID:KvIpEFBr0
この問題に関して、辞書が間違ってる(怒)!と、他のスレにまで過去の恨みつらみレスを貼っていたのは、唯一、長文の方で、辞書サイドに問い合わせたのは彼自身の(精神安定の)ためである。

彼以外に辞書に問い合わせる必要など感じてなかったのに、なぜ感謝しろよと?
おれがあんたのレスを労力をかけて絵文字でデコったら、それで感謝してくれるんだ?
その言葉、こんどずっと覚えよくように。

********

LDOCEから来た返事は、彼ら的には記述に間違いはないということである。

コウビルドから来た返事(※)は、ある定義にかんして、それにあまりマッチしない文だったと回答してるだけで、定義そのものが間違っているとは言ってない。
まして、編集の末に変なところに文が入ったとこも言ってない。(じゃあ、【元来入るべき場所はどこだ?】
長文の人が自分があってたという要素はひとつもない。

(※) ロングマンより簡潔で問題も絞られているので、長文の方が勝手に書き換えた可能性あり。実際に例文の書き換えがなされるまで信用すべきでない。

********

challenge sb to V に関して、LDOCEは三つの分類に入れていて、もっとも整理されている印象を受ける。
この記述自体に疑いをかけてる人を、長文の人以外に見た記憶がない。
しかし結局は次の通りであった。

辞書が間違ってる~。alcomがどうたら~、ボクが正しいの差別される~(全部ソースあるよ?)

(ソフィスト先生のレスと同じことを主張し始めて)ボクも【みんなも】混乱してたけどなんでだろ(テヘペロ
0641三年英太郎 ◆3CZBjOt3.Y (ワッチョイ ff86-7uza)
垢版 |
2022/10/04(火) 15:12:46.18ID:KvIpEFBr0
長文の人がおれ(やソフィスト先生)をNGにしてるのは嘘である。観察眼なさすぎ

一例をあげる

スレッドを立てるまでもない質問スレッド Part 375
https://lavender.5ch.net/test/read.cgi/english/1663090161/90

この問題は、おれの間違いを指摘を読まなかったら、あーだこーだ長々と言い訳する必要はなかったのである。
このスレもそうで、おれやソフィスト先生のレスを読んでなかったら>>376-379で発狂することもなかった。

この程度の幼稚な嘘も見抜けない(あるいは見抜てるけども、その嘘に付き合う程度にバイアスがかかってる)のがブタ耳である。
0643三年英太郎 ◆3CZBjOt3.Y (ワッチョイ ff86-7uza)
垢版 |
2022/10/04(火) 15:16:08.04ID:KvIpEFBr0
彼はおそらく誰もNGにしてない。
それが障害からくるルーティーン遵守のためなのか、そもそもNGのやり方知らないのか
そこは判断はできないが、その点だけは評価しよう^^
0644ブタ耳 (ワンミングク MM9f-ZYgd)
垢版 |
2022/10/04(火) 15:21:42.29ID:jcyuTApGM
>>641
なんだNGにしてるわげじゃなかったんですか?
それは私の目が節穴でした。
申し訳ありません。

節穴といえば、>>621で自演の疑いをかけてしまったことにもお詫びします。
ソフィイスト先生、せっかくまともな反論をしてくださっているところ、あらぬ疑いをかけてしまい、本当に申し訳ありませんでした。
0645名無しさん@英語勉強中 (ワッチョイ a344-H63Z)
垢版 |
2022/10/04(火) 15:31:30.37ID:BNcX6Oxx0
>>639
2は、Lost and Foundの典型的ひっかけ
3は、疑問文に対して疑問文だからTOEIC的にその時点で罰

すると答えは1
「それが、そうです。」となる
0646628 (テテンテンテン MM7f-vqPj)
垢版 |
2022/10/04(火) 15:46:12.68ID:zgd/DGhWM
レスありがとう。
正解は1となってる。

動画はこれ
https://youtu.be/mpL21FCsA9c?t=1617

what can I do for you?が「遺失物管理です。今日はどうされましたか?」みたいなニュアンスなのはなんとなくわかるんだけど、それに対する返答が1の「はい、そうです」ってのがすごく違和感あるんだよなぁ。
消去法だと確かに1しかないんだけども
0647名無しさん@英語勉強中 (ワッチョイ a344-H63Z)
垢版 |
2022/10/04(火) 15:50:34.71ID:BNcX6Oxx0
>>646
落とし物係へ行ったら
係員が届いたばかりの自分の荷物を持ちながら・・・
「落とし物係ですがなにか?」
「あ、それ」

こういうシチュだな、同じような経験したことあるよ
0648名無しさん@英語勉強中 (ワッチョイ 7fe2-0qRf)
垢版 |
2022/10/04(火) 15:51:24.15ID:nTSuWXsE0
そんな出典もロクにない怪しいYoutube動画はスルーが賢明
0649ブタ耳 (ワンミングク MM9f-ZYgd)
垢版 |
2022/10/04(火) 15:52:27.78ID:jcyuTApGM
>>646
おおー、予想外に好感度の高いイケメン外人が出てきた。
どうせ嘘くさい日本人のYoutuberが出てきて適当な解説するんだろうと思ってた。
あとでじっくり聞いてみる
0650628 (テテンテンテン MM7f-vqPj)
垢版 |
2022/10/04(火) 15:53:52.80ID:zgd/DGhWM
>>647
うーんなるほどそういうシチュなのか。
whatの質問文だけど、Yesから始まるのが正解ってなんか意地悪だなぁ。
0651ブタ耳 (ワンミングク MM9f-ZYgd)
垢版 |
2022/10/04(火) 16:03:50.30ID:jcyuTApGM
疑問点いくつか。
対面状況で係がいきなりLost and Foundと名乗るのは不自然。電話の会話としか考えられない。
対面だったとしても、いきなり「はい、それです」と答えるのは不自然。
「はい、それです」と言いたかったとしても、その場合は、Yes, that’s it! と言うのではないか?

結論: 問題が不自然

ところでどなたか>>632に答えてあげてください
0652名無しさん@英語勉強中 (ワッチョイ a344-H63Z)
垢版 |
2022/10/04(火) 16:05:17.95ID:BNcX6Oxx0
>>650
ここでのYESは
同意や肯定的な意見を表すyesじゃなくって
感嘆詞としてのyesなんじゃねの 
Oh Yeah!とかYay!みたいに
疑問文に対して感嘆詞や間投詞で返答するのはありよ
0654名無しさん@英語勉強中 (ワッチョイW 839d-i+A0)
垢版 |
2022/10/04(火) 16:12:35.20ID:f2LF4XGH0
係が美人でいきなりナンパして電話番号を聞いた
0655名無しさん@英語勉強中 (ワッチョイ a344-H63Z)
垢版 |
2022/10/04(火) 16:18:21.24ID:BNcX6Oxx0
>>653
itの方が一般的だが、soの方がよりフォーマルだからこの場合はいいんじゃね
that’s itだと「お前の言うとおりだ」みたいにもなっちゃうし
0656名無しさん@英語勉強中 (ワッチョイ a344-H63Z)
垢版 |
2022/10/04(火) 16:24:21.76ID:BNcX6Oxx0
>>632
私が病気なったのと、母が来る死んだ時が同じだから過去形で良いんじゃね
0657名無しさん@英語勉強中 (ワッチョイ a344-H63Z)
垢版 |
2022/10/04(火) 16:27:43.19ID:BNcX6Oxx0
as if my illness had been her own だと

まるで(その時より以前に)病気であったかのように、みたいになるんじゃね
0660名無しさん@英語勉強中 (ワッチョイ a344-H63Z)
垢版 |
2022/10/04(火) 17:37:47.22ID:BNcX6Oxx0
テンテンとスプーキーはいつも仲良くていいな
うらやましいよ
0666三年英太郎 ◆3CZBjOt3.Y (ワッチョイ ff86-7uza)
垢版 |
2022/10/04(火) 19:42:53.42ID:KvIpEFBr0
GARIUS SPENCER: I think they're crying foul, but they don't have anything to prove that with, and I challenge them to prove that.

これは、異議・疑問視だろう。まあ「刺激型」にも読めないこともないが
0672名無しさん@英語勉強中 (スップ Sd1f-k55t)
垢版 |
2022/10/04(火) 23:19:17.91ID:E9EO7rq2d
>>670
ど田舎に住んでるの?マックなんて何処にでもあるじゃん
0674632 (ガックシ 067f-CuO3)
垢版 |
2022/10/05(水) 08:03:06.77ID:66+l9Tx66
>>656,657

>>632
です.コメントありがとうございます.
大変,参考になります.
なんとなくわかった気がしました.
0675ブタ耳 (ワッチョイ 8fe3-e5cN)
垢版 |
2022/10/05(水) 11:12:15.12ID:8tOmnpPT0
>>674
仮定法と「時制の一致」の問題は結構複雑。

江川, 『英文法解説』 (p.468) から引用すると、
「仮定法の動詞は事実を述べるのではなく, 事柄を心の中で想定して述べるのであるから, 直接方の動詞に適用される時制の一致には制約されない。」

I think/thought if she were a little older she would have more sense.
(彼女がもう少し大人なら, もっと分別があるだろうと思います/思いました)

ただし、
「仮定法の動詞には時制の一致は適用されないと一般に言われているが, 間接話法の場合には必ずしもそうは言えない」 (同書, p.475)

長くなってしまうので, 間接話法の場合についての具体例は,江川本で確認してください。pp.474-475です。
0677632 (テテンテンテン MM7f-CuO3)
垢版 |
2022/10/05(水) 13:45:55.04ID:1BzFQwRSM
>>675

ありがとうございます.

紹介していただいた本にあたってみます.
具体的なページも教えていただき,大変たすかります.
0678名無しさん@英語勉強中 (スップ Sd1f-+pZx)
垢版 |
2022/10/05(水) 18:04:18.04ID:q49HxL2rd
英太郎さん他、変形生成文法について知っている方、教えてください

以下の理解で大まかには合っているでしょうか?
特に指定部と付加部について自信がありません
他にも根本的に間違っているかも知れませんが、よろしくお願いします

①指定部 specifier
冠詞相当語句や複数形の語尾のこと

②主要部 head
いわゆる主語を含む句や節のこと

③補部 complement
述語動詞とその必須要素(目的語や補語、義務的な副詞語句)のかたまりのこと

④付加部 adjunct
上記以外の部分?
0679ブタ耳 (ワッチョイ 8fe3-e5cN)
垢版 |
2022/10/05(水) 18:27:40.27ID:8tOmnpPT0
>>646
このYouTube動画、イケメンはサムネイルだけで、動画は音声だけ。
こういうのを詐欺ネイルと呼ぶことにしてはどうだろうか。

聞いてみて、もしかしてこれ自動音声サービスで作った朗読なんじゃね?
という疑問が生じたので、自分で試してみることにした。

利用したのはこのサイト。何回か無料で試せる。
https://ondoku3.com/en/

このサイトのMatthewさんに、>>628と同じテキストを読んでもらった音声がこれ。
https://uploader.cc/s/rpfn3jyluuuo9raacsp8v4rb86vkg7z5d6brjiojhmptxzv0k72padptwha8q54h.mp3
問題の動画の音声とは違う声だが、自動音声でもここまで読んでくれる。

同じ音声が出てきたら面白いなと思っていたんだが、残念ながら(?)別の声だった。
問題の動画では実際にネイティブスピーカーが読んでるのかもしれない。
しかし、この問題を読まされて選択肢がクソであることを指摘しなかったのだとしたら、そのネイティブスピーカーもクソだと思う。
0680😉三年英太郎🌈 ◆3CZBjOt3.Y (ワッチョイW ff86-nigo)
垢版 |
2022/10/05(水) 18:28:29.16ID:nB5Y4JTa0
>>678
違うなー

その4つは、すべての種類の句(動詞句、名詞句、副詞句、前置詞句、形容詞句)が持ってる要素。(実際の運用では4つ必ずしも揃ってるわけじゃありません)

だからどの種類の句かによって、4つの要素の実際に指すものも違うのです🧐

動詞句の場合、headは述語動詞、spécifier は主語、complement とadjunct は>>678で合ってる👌🏻

名詞句の場合はheadが名詞、specifierが限定詞(冠詞などなど)、補部が the picture [of Eichan] とか the fact [that SV] 、adjunctはそれ以外のおまけ部分

その他の例は↓で👌🏻
https://lavender.5ch.net/test/read.cgi/english/1628868251/29
0686ブタ耳 (ワッチョイ 8fe3-e5cN)
垢版 |
2022/10/05(水) 18:37:57.18ID:8tOmnpPT0
>>680
英太郎さん、>>671みたいなクソレス書いてる暇あったら、そっちの良スレを進行させてくださいよ。
ずっと待ってるんですけど。
向こうの126と128は私が書いたレスです。
0687名無しさん@英語勉強中 (スッップ Sd1f-+pZx)
垢版 |
2022/10/05(水) 18:40:29.87ID:sMNmbxdWd
>>680について英太郎さんすまん、
the picture [of Eichan]
the fact [that SV]
の[]内の呼び方が補部ではない、という訂正をしてくださったのだと理解していますが、では[]内てなんでしょうか?
これは名詞句内の付加部?
0691ブタ耳 (ワッチョイ 8fe3-e5cN)
垢版 |
2022/10/05(水) 20:03:31.27ID:8tOmnpPT0
>>646
これが自動音声サービスによる朗読であることを証明してみようと思う。

3問の朗読を録音し、選択肢の発音部分から one two three だけを抜き取ってみる。
3問分の one two three を並べた音声がこちら。
https://uploader.cc/s/3oh81il61kk9slb68w0okvvef0vh6naovugh2k35o2jxcvsqv5oy3q8p7bc40v6p.mp3

波形のイメージはこんな感じ。
https://imgur.com/ildvlhm

すべての one two three は、まったく同じ音声であるようだ。
生身の人間がこれほど同質の one two three を発音し続けることは不可能。
よって、これは自動音声サービスによる朗読であると考えるのが自然である。

厳密に同一性を確かめる方法もあるといえばあるんだけど、めんどくさいんでやめとく。
0692名無しさん@英語勉強中 (ワッチョイ cf8f-UJKs)
垢版 |
2022/10/05(水) 20:34:23.32ID:M0lZMz880
He is as tall as his father.

2こめのasは接続詞でhis father is tallのis tallを省略した形だそうですが、このas節は副詞節として何かを修飾するんでしょうか?
tallを修飾?
0694エキセントリックなソフィスト ◆BWpafMA3P8me (ワッチョイ f396-pIDl)
垢版 |
2022/10/05(水) 22:40:12.84ID:TXA9lFl+0
Lost and Found. What can i do for you?

1 Yes, that is so
2 I lost my way
3 May I ask your phone number?

こんな意味じゃないの?

係:Lost and Found (遺失物取り扱い窓口)です。どうされましたか?
客:道をlostしました。(道に迷いました)
係:おもしろい!でも、ここは貴方のお笑いを披露する舞台ではありませんよ。

つまり、笑い話で、2番が正解でしょ。笑い話という設定もありなんだから、2番を×にしたら私は怒ります!(^-^)
0696エキセントリックなソフィスト ◆BWpafMA3P8me (ワッチョイ f396-pIDl)
垢版 |
2022/10/05(水) 22:54:19.23ID:TXA9lFl+0
>>695
お笑い的には、「道を落しました」のほうが面白いと思うけどなあ (^O^)
0698エキセントリックなソフィスト ◆BWpafMA3P8me (ワッチョイ f396-pIDl)
垢版 |
2022/10/05(水) 23:03:20.20ID:TXA9lFl+0
>>697
「私は落せない」←うまい! でも、これ日本語でしか通じないという・・・
0700名無しさん@英語勉強中 (JP 0H87-umC8)
垢版 |
2022/10/06(木) 03:18:34.56ID:KDSEQaSLH
>>692
副詞は要するに比較的前後に縛られずズドンと挟めるあぶれ者やし厳密に考えるんは文法学者に任せたらいいと思うけど、文法的にどの文でも対応しようとするんやったら規則通り動詞を修飾するとなるけど感覚的にはやっぱり形容詞への修飾ちゃうか
0703名無しさん@英語勉強中 (ワッチョイ cf8f-UJKs)
垢版 |
2022/10/06(木) 19:34:41.53ID:Vs87i7Iu0
>>701
>>702
ご回答ありがとうございます。副詞のasにかかっているという根拠を教えていただけると幸いです。
0704ブタ耳 (ワッチョイW 8fe3-ZYgd)
垢版 |
2022/10/07(金) 00:53:54.70ID:Q2vdk9X00
>>703
“as tall” にかかる、と考えるのが自然だと思います。比較級から順を追って説明してみます。

1) He is taller than his father.
「彼はtallerである」と先ず述べ、続いて「何に比べてtallerなのか」を[than his father] が説明。

[than his father] は [taller] にかかっている、と考えて良い。
これを次のように図示する。
He is [taller] ← [than his father].

2) He is more famous than his father.
これは(1)と同様に、
He is [more famous] ← [than his father].

ここで、[than his father] が [more] だけにかかると考えるのは不自然である。

[続く]
0705ブタ耳 (ワッチョイW 8fe3-ZYgd)
垢版 |
2022/10/07(金) 00:54:47.62ID:Q2vdk9X00
3) He is as tall as his father.
これは(2)の文と同じ構造。
He is [as tall] ← [as his father].

[than his father] が [more] だけにかかると考えるのが不自然であったのと同様に、[as his father] が [as] だけにかかると考えるのは不自然。

[as his father]は[as tall]にかかる、と考えるのが自然である。
0706名無しさん@英語勉強中 (ワッチョイ 0393-LIzE)
垢版 |
2022/10/07(金) 02:31:59.40ID:Auxjn4KI0
>>704
>ここで、[than his father] が [more] だけにかかると考えるのは不自然である。
どうだと自然でどうだと不自然と?
0707名無しさん@英語勉強中 (テテンテンテン MM7f-vqPj)
垢版 |
2022/10/07(金) 03:21:21.71ID:bcaWw2hYM
I'm going to Rome when I have finished my Italian lessons.
という文章で、
以下のようにするのは文法上間違いですか?
I'm going to Rome when I finished my Italian lessons.
0708名無しさん@英語勉強中 (テテンテンテン MM7f-o1nH)
垢版 |
2022/10/07(金) 03:27:49.05ID:45iRI9ooM
>>707
文法上っていうか
まず意味的にあり得ないでしょ
0709名無しさん@英語勉強中 (テテンテンテン MM7f-o1nH)
垢版 |
2022/10/07(金) 03:39:27.02ID:45iRI9ooM
I'm going to Rome ←現在の状態・行為
when I finished my Italian lessons ←過去の時点を表してる
0710名無しさん@英語勉強中 (テテンテンテン MM7f-o1nH)
垢版 |
2022/10/07(金) 03:52:42.43ID:45iRI9ooM
I'm going to Rome は予定だから未来という人もいる


I'm going to Rome when I have finished my Italian lessons.
の英文なら「未来」というべきだな
0711名無しさん@英語勉強中 (テテンテンテン MM7f-o1nH)
垢版 |
2022/10/07(金) 03:55:03.42ID:45iRI9ooM
つか
I'm going to Rome when I have finished my Italian lessons.
の意味をわかってるなら
have finished → finished に書き換えるわけない
0712名無しさん@英語勉強中 (テテンテンテン MM7f-o1nH)
垢版 |
2022/10/07(金) 04:09:42.49ID:45iRI9ooM
>>706
more+原級 と 比較級
を同等に扱うべしという思考をしてるのでしょうね。

でも
形容詞の比較級をthan節が修飾するのと同様に
副詞の比較級moreをthan節が修飾しても
何も不自然ではないと私は思います
0713ブタ耳 (ワッチョイW 8fe3-ZYgd)
垢版 |
2022/10/07(金) 05:54:32.05ID:Q2vdk9X00
>>712
そう、正にそういう思考。

[than his father] が [more] だけにかかるのだとすれば、
He is taller than his father.の場合、[than his father]は[tallerという語の中に含まれるmore的要素]だけにかかるという、かなり無理な考えかたを強いられるのではないか?

仮に,[tallerという語の中に含まれるmore的要素] = [er] とすれば、
? He is tall[er] ← [than his father].
これはだめでしょう。

以上、「高卒3号が独自に考えた高卒英文法」でした。それなりに筋は通っているように思う。
0714ブタ耳 (ワッチョイ 8fe3-e5cN)
垢版 |
2022/10/07(金) 09:17:37.54ID:Q2vdk9X00
ちょっと補充して修正、更に追記。

旧)[than his father] が [more] だけにかかるのだとすれば …

新)He is more famous than his father. という文で、[than his father] が [more] だけにかかり、 famous にはかからないのだとすれば、
He is taller than his father.の場合、[than his father]は[tallerという語の中に含まれるmore的要素]だけにかかるという、かなり無理な考えかたを強いられるのではないか?

追記:
ロイヤル英文法の「比較の副詞節を導く接続詞」の項(p.626)に次の例文あり。
Philip is much more intelligent than Bill.
(フィリップはビルよりもずっと頭が切れる)

ここで ★more だけ★が斜字体、that が太字体になっており、
that以下の副詞節が ★more だけ★を修飾しているという著者の考えが垣間見える。
これは高卒英文法の考えとは異なる。
0716ブタ耳 (ワッチョイ 8fe3-e5cN)
垢版 |
2022/10/07(金) 10:04:04.09ID:Q2vdk9X00
あ、もう1個タイポ発見。
×that が太字体
○than が太字体

タイポの修正に限らず、考え方自体に誤りがある場合も即座に修正したいと思いますので、何かお気づきの方は御指摘下さい。
0717名無しさん@英語勉強中 (テテンテンテン MM7f-o1nH)
垢版 |
2022/10/07(金) 13:16:44.91ID:pN5F20+PM
>> [more] だけにかかり、 famous にはかからないのだとすれば、
>>He is taller than his father.の場合、
>>[than his father]は[tallerという語の中に含まれるmore的要素]だけにかかるという、
>>かなり無理な考えかたを強いられるのではないか?

そんな無理な考えが強いられことになるわけないと思うけど。
than節が形容詞の比較級tallerにかかるのと同様に
副詞の比較級moreにかかっているというだけの話でしょ。
そのmoreが原形の形容詞を修飾しているというだけ。
0718ブタ耳 (ワッチョイ 8fe3-e5cN)
垢版 |
2022/10/07(金) 13:24:59.12ID:Q2vdk9X00
>>717
more が famous を修飾してるのは確かだけど、than 以下が修飾しているのは [more famous] だと考えるほうが整合性がありませんか?

He is [taller] ← [than his father].
He is [more → famous] ← [than his father]
0720ブタ耳 (ワッチョイ 8fe3-e5cN)
垢版 |
2022/10/07(金) 13:36:08.49ID:Q2vdk9X00
整合性があるというのは、どちらも形容詞を修飾していることになるからです。

taller 形容詞 tall の比較級
more famous 形容詞 famousの比較級
0721名無しさん@英語勉強中 (テテンテンテン MM7f-o1nH)
垢版 |
2022/10/07(金) 13:40:44.71ID:pN5F20+PM
>>718
何度も言いますが
than節が形容詞の比較級tallerにかかるのと同様に
副詞の比較級moreにかかっているというだけの話だという認識です。

「more+原級 と 比較級 を同等に扱うべし」という縛りは必要ないと思います。
0722名無しさん@英語勉強中 (テテンテンテン MM7f-o1nH)
垢版 |
2022/10/07(金) 13:44:04.04ID:pN5F20+PM
>>more famous 形容詞 famousの比較級

便宜上、そう表現されることもあるかもしれませんが
実際は、'more famous'は、famousの比較級ではありません。
famousには比較級がないので、程度を表す副詞の比較級によって修飾するという話です。

動詞を修飾するmoreと同様です。
0723ブタ耳 (ワッチョイ 8fe3-e5cN)
垢版 |
2022/10/07(金) 13:44:55.46ID:Q2vdk9X00
>>721
それだと、more と famous が分離してしまい、
than 以下が副詞のmore を修飾し、その副詞が、famousという形容詞の原級を修飾するという、いささかややこしいことになりませんか?

more famous を taller 同等の「1個の比較級」「形容詞の1単位」として捉えた方が自然ではないかということです。
0724名無しさん@英語勉強中 (テテンテンテン MM7f-o1nH)
垢版 |
2022/10/07(金) 13:47:58.02ID:pN5F20+PM
>>723

>>than 以下が副詞のmore を修飾し、その副詞が、famousという形容詞の原級を修飾するという、いささかややこしいことになりませんか?

特にややこしいことになるとは思いません。
副詞moreが動詞を修飾するのと同様です。
0725ブタ耳 (ワッチョイ 8fe3-e5cN)
垢版 |
2022/10/07(金) 13:57:07.36ID:Q2vdk9X00
>>724
1) He is [taller] ← [than his father].
2) He is [more → famous] ← [than his father]
と考えれば、2つの文が同じ構造を持つものとして捉えられるのに対し、

2)の文で、than 以下が 副詞 more のみにかかるとすると、2つの文が別の構造を持つことになってしまいます。
そう捉える方が適切だとは思いません。
0726名無しさん@英語勉強中 (テテンテンテン MM7f-o1nH)
垢版 |
2022/10/07(金) 14:04:56.63ID:pN5F20+PM
>>725
実際、前者が形容詞の比較級を使った文であるのに対して
後者は副詞の比較級と使った文という意味では
構造が違うわけですから

「構造を持つことになってしまいます」
と言われれても困ります。

タイトルは忘れましたが
別の参考書でも
0727名無しさん@英語勉強中 (テテンテンテン MM7f-o1nH)
垢版 |
2022/10/07(金) 14:07:43.58ID:pN5F20+PM
途中で投稿してしまった。

まぁいいや。
まぁ、私はmoreだけを修飾をしているという解釈は変わりませんが

[more → famous] ← [than his father]
という解釈でも、別に構わないのではないですか?
0728ブタ耳 (ワッチョイ 8fe3-e5cN)
垢版 |
2022/10/07(金) 14:11:04.33ID:Q2vdk9X00
>>727
はい、私はそのよりシンプルな解釈をとります。

このまま論じ続けても並行線をたどることになりそうですね。
とはいえ、「別の参考書でも」の続きが気になります。
そこだけでも続きを書いてもらえませんか?

他の方々はどうお考えでしょうか?
0730名無しさん@英語勉強中 (ワッチョイ c3b9-BgLw)
垢版 |
2022/10/07(金) 19:41:00.27ID:1auMcQ/Y0
>>729
この板は会話を重視した板なんですか?
0731ブタ耳 (ワッチョイ 4ee3-3fXN)
垢版 |
2022/10/08(土) 00:22:58.40ID:932AQ9bu0
私が>>483からこだわり始めた関係代名詞 what の解釈について、とりあえず自分なりの結論に達したので報告しておきます。
結論から言うと、>>543で考えたことが正しいのだろうということです。

問題の英文(初出は>>228):
To his library in Cable House he will add volumes in which are contained what human knowledge,
in the 1820s, has to offer on the mystery of the human mind.

原文(Waterland, Graham Swift)はここで読める: https://www.onlinereadfreebooks.com/en/Waterland-623007/8

その後 >>501 で和訳を提示。
>>620で文脈を調べる必要があると主張。

で、前後の文脈を調べてみました。
0732ブタ耳 (ワッチョイ 4ee3-3fXN)
垢版 |
2022/10/08(土) 00:27:01.40ID:932AQ9bu0
※Heとは: トーマス・アトキンソンという、ビール醸造で富を築いた60過ぎの老人。
この老人には、20歳以上若い美しい妻がいるのだが、あるとき嫉妬に狂った老人がこの妻を殴り倒してしまう。
その結果、妻は物言わぬ廃人となってしまう。
この妻を何とか元通りにしようとして、老人はあらゆる手をつくそうとする。

※ケーブルハウスとは: トーマスの邸宅

※1820年代とは: 医学が未発達だった時代という設定。ヨーロッパの戦乱と関係があるのかと予想していたが、そうではなかった。
同じ9章に、「1800年から1815年」において、"but medical science is still in its infancy" との記述あり。
0733名無しさん@英語勉強中 (ワッチョイ 4ee3-3fXN)
垢版 |
2022/10/08(土) 00:29:19.80ID:932AQ9bu0
問題の文を含む一段落の原文と、私の和訳を提示します。

The times cannot be numbered when Thomas Atkinson will ask, Why? Why? And again Why? (For heartache, too, inspires its own sad curiosity.) Not content with the verdict of physicians, he will embark, himself, on the study of the brain and the nervous system. To his library in Cable House he will add volumes in which are contained what human knowledge, in the 1820s, has to offer on the mystery of the human mind. Where once he pored over the topography of the Fens and the innumerable complexities of drainage, flood control and pumping systems, he will pore over the even more intricate topography of the medulla and the cerebellum, which have, so he discovers, their own networks of channels and ducts and their own dependence on the constant distribution of fluids.

But this is an internal land which cannot be redeemed, cannot be reclaimed, once it is lost.


トーマス・アトキンソンは、数えきれないほど何回も尋ねることになる。なぜだ、なぜだ、さらに重ねて、なぜだ、と。(というのも、心が痛むときには、その痛み自体が悲しい好奇心を呼び起こすものだからだ) 医者たちの下した診断に満足しなかった彼は、脳の神経システムの研究に自ら乗り出していく。■ケーブルハウスにある彼の書斎には新たに数々の書物が加えられることになる。その書物には、人間の心という神秘に関して、1820年代において人類の知識が提供しうるすべてが収められている。■ 昔、沼沢地帯の地形学と、複雑に絡み合う無数の排水路、洪水制御、揚水システムの研究に没頭したその場所で、今や彼は髄質と小脳が織りなす更に複雑な地形学の研究に没頭することになる。そして彼は発見する。今度の対象は、それ自身の水路と導管のネットワークを持ち、絶え間なく分配される体液に独自のしかたで依存しているということを。

けれどもそれは人の内面に広がる土地であり、いったん失われてしまえば、取り戻すことも干拓することもできないものなのだ。
0734ブタ耳 (ワッチョイ 4ee3-3fXN)
垢版 |
2022/10/08(土) 00:34:06.22ID:932AQ9bu0
実は、この小説は翻訳も出ています。『ウォーターランド』、真野泰 訳、新潮社
>>733の和訳はこれは参照する前に訳したものです。後半は真野訳の方が分かりやすいですね。)

以下、真野訳を引用。p.117

*****
数えきれないほどの回数、トマス・アトキンソンは自問する --- なぜ? なぜ? そしてもう一度、なぜ? (というのも、心痛もまた独特の、悲しい好奇心を生むのである)。医師たちの判定に満足せず、トマスは自分で、脳と神経系の研究に着手する。■ケイブル邸の彼の蔵書に新たに加わる一群の書物には、1820年代の人類が、人間精神の神秘についてすでに得ている知識の全部が詰まっている。■ かつてフェンズの地形について、また排水や治水、ポンプによる揚水システムに関する無数の、複雑極まりない問題について本を読みふけった、その同じ部屋で彼は、こんどはさらに入り組んだ、脊髄と小脳の地形について読みふけるのである。読んでみれば脊髄と小脳もやはり、水路と導管の網状組織をもち、やはり、液体が常時各所に供給されることにより成り立っている。

しかしそれは、ひとたび失われたら最後取り戻すことのできない、排水して干拓してふたたび陸地化するというわけにはいかない、内面の土地なのである。
*****

真野訳でも、「全部が詰まっている」と解釈されています。
我が意を得たり、と思いました。
0735ブタ耳 (ワッチョイ 4ee3-3fXN)
垢版 |
2022/10/08(土) 00:47:20.22ID:932AQ9bu0
あ、あと真野さんの訳は初版本を底本にしているものです。
その後改訂版が出て、かなり内容の変更があるとか。
私の引用した英文が初版なのか改訂版なのかは分かりません。
(たぶん初版本だろうと思いますが)
0736ブタ耳 (ワッチョイ 4ee3-3fXN)
垢版 |
2022/10/08(土) 10:43:20.09ID:932AQ9bu0
追記:
引用した英文は1992年の改訂版だった。
引用元サイトの1ページ目にちゃんと書いてあった。
(ところでここ、どう考えても違法サイトなんで、もうurlは貼らないでおく)

1983年の初版本は結構プレミアついてて2万円くらいで出回ってるようす。
初版本でななくとも1992年までに出版されたペーパーバック版とかでも内容は同じはず。
運がよければ、いずれ手に入るかもしれない。今のところ買うつもりはないけど。
0737名無しさん@英語勉強中 (ワッチョイ de8f-Rol6)
垢版 |
2022/10/08(土) 12:53:05.04ID:TepmLDG70
Not only he but you are to blame.

このNot onlyはheにかかってますが品詞はなんでしょうか(形容詞?)。
he (is to blame)が省略されている形?
0739名無しさん@英語勉強中 (ワッチョイ de8f-Rol6)
垢版 |
2022/10/08(土) 13:41:43.49ID:TepmLDG70
Not onlyは副詞句で名詞heにかかっているように見えます。
辞書のどの部分を引くとNot onlyは形容詞的に作用していると書いてあるのか知りたいところですが笑
0741名無しさん@英語勉強中 (ワッチョイ de8f-Rol6)
垢版 |
2022/10/08(土) 13:52:27.30ID:TepmLDG70
もしあなたが辞書のどの部分を引けばいいのか知っているのならば、教えていただけますか。
そうでないならそれ以外の文を書きなさい。
0743ブタ耳 (ワッチョイ 4ee3-3fXN)
垢版 |
2022/10/08(土) 15:08:19.39ID:932AQ9bu0
「副詞は名詞以外を修飾する」。「名詞を修飾するのは形容詞」と教えられることが多いが、実は副詞は名詞を修飾することもある。

ジーニアス第5版、「副詞」only の語法説明からちょっと引用
*****
[語法]
(1)only は通例修飾する語・句・節の直前か, 時に直後に置く:
Only he [He only] solved that problem yesterday. 彼だけが昨日その問題を解いた /
He solved only that problem yesterday. 彼は昨日はその問題だけしか解いていない /
以下省略
*****

江川『英文法解説』から引用>>741

******
§92. 副詞の用法
動詞・形容詞・他の副詞を修飾する他に, 次の用法がある。
(1) 名詞を修飾 一部の副詞に限られ, 一般の副詞には通用しない。
--- quite と even の例文省略 ---
I don't need a bandage; it's only a scratch.
(包帯はいらないよ。ちょっとひっかいただけだから)
******
0744ブタ耳 (ワッチョイ 4ee3-3fXN)
垢版 |
2022/10/08(土) 15:13:21.70ID:932AQ9bu0
ロイヤル英文法の副詞の定義(?)はもっと大胆というか大雑把というか適当というか何というか…ともあれ p.50から引用

*****
動詞, 形容詞, 副詞, 名詞, 句, 節, または文全体を修飾する語を副詞という。
---中略---
He is only a child. <彼はまだほんの子供だ> (名詞を修飾)
*****
0745ブタ耳 (ワッチョイ 4ee3-3fXN)
垢版 |
2022/10/08(土) 15:21:28.73ID:932AQ9bu0
>>741
2つ前のレスでアンカーが変なところについちゃったが気にしないように。

only を辞書でひいて 「副詞」 のところを見れば、名詞または代名詞を修飾してる例文が出てるはず。

ついでながら、「Not onlyは形容詞的に作用している」と書いてある辞書は存在しないので、辞書のどの部分を見ても見つけられません。
0746名無しさん@英語勉強中 (ワッチョイ 8a5d-Rol6)
垢版 |
2022/10/08(土) 15:35:59.37ID:+7h9jxMS0
ブタ耳さんありがとうございます!
0747ブタ耳 (ワッチョイ 4ee3-3fXN)
垢版 |
2022/10/08(土) 17:58:00.28ID:932AQ9bu0
ところで、私のwhat 解釈について、長々と反論してくださった方が少なくとも二人いて、一人はソフィスト先生。
>>515以降の一連のレス。 (ワッチョイ cWBVS2GD0)
ソフィスト先生からは、この件について改めて何か一言コメントいただきたいのですが、いかがでしょうか?

あともう一人、スプッッ、スッップで「高卒」好きの人がいたようですが、その人はどうでもいいです。
0748ブタ耳 (ワッチョイ 4ee3-3fXN)
垢版 |
2022/10/08(土) 20:50:24.96ID:932AQ9bu0
修正。ワッチョイじゃなくてIDでした。

ついでながら、この小説を映画化した Waterland を紹介しときます。
https://www.youtube.com/watch?v=RW6BmZ16hH0
英語字幕付きで全編視聴可能。合法なのかこれ?

残念ながらトマス・アトキンソンのエピソードは出てきません。

小説の訳者の真野さんのあとがきから引用すると、
「結果として映画は、メロドラマチックに面白い(つまり物語性が強くでている)箇所をつないだ形となっているから、小説を読んでから観ると食い足りない。逆に観てから読めば、言葉をひとつひとつ直線的に並べるしかない小説にこんなこともできるのかと、感嘆する喜びがまっているだろう。」

ただ、真野さん、「反抗的な生徒のプライス(映画ではマシュー)」とか書いてるけど、映画でもプライスなんだよね。
本当に観たのか、ちょっと疑わしい。
0749エキセントリックなソフィスト ◆BWpafMA3P8me (ワッチョイ 6f96-kHT+)
垢版 |
2022/10/08(土) 22:45:17.43ID:VPV9yn8M0
>>747
粘るね~ では、反論(^-^)
What human knowledge has to offer on 人間の心の神秘 are contained in the volumes. という英文で考えてみよう。

この「what」を「すべての(all the)」で解釈できるのは、「what」を関係形容詞で解釈するときだけである。辞書や参考書でもそうなっているはずだ。
関係形容詞のwhatのみが「all」で解釈できるのだ。これ重要。 anything thatはallの意味でないことは前に説明したとおり。
「すべての人知」「人知のすべて」と訳者が訳したということは、訳者がこれを関係形容詞のwhatで解釈し、「What human knowledge」がひとつの句で、hasの主語になっていると解釈したということになってしまう。(訳者が文法をきちんと知っているならそうなる)

しかし、そう解釈すると上記英文のhas toは、「しなければならない」と訳さなければならなくなり、正確には次のように訳さなければならなくなる。
「人間の心の神秘に関して提供しなければならない義務を負う人知の全部がその一群の書物に含まれている」(offerの目的語の欠損)

以上より、「しなければならない」と訳すと文の構造が崩壊するのだから、whatを関係形容詞で解釈した訳は誤訳であると考える。
よって、上記英文は、「Things that human knowledge has to offer on 人間の心の神秘 are contained in the volumes.」と解釈し、正しい訳は「その一群の書物には、人間の心の神秘について人知が寄与しているものが(いろいろ)含まれている」となる。
"ARE contained"なので、「what=things that」と複数で解釈し、「いろいろ」と意訳するのがベストだろう。
0750ブタ耳 (ワッチョイ 4ee3-3fXN)
垢版 |
2022/10/08(土) 22:47:31.33ID:932AQ9bu0
>>749
なんだ、まだ認めないんですか?
頭冷やしたらどうですか?

>「what」を関係形容詞で解釈するときだけである。辞書や参考書でもそうなっているはずだ。
>関係形容詞のwhatのみが「all」で解釈できるのだ。これ重要

そんなことないですよ。
0751ブタ耳 (ワッチョイ 4ee3-3fXN)
垢版 |
2022/10/08(土) 22:51:53.29ID:932AQ9bu0
>>614のあなたの言い方をもじって言うと、
これ進学校の高校生程度でも、中には全部と訳せるwhatだと気づく人がいることでしょう。
正しい文法知識と文脈を読み取るセンスがあれば、ですが。
0752ブタ耳 (ワッチョイ 4ee3-3fXN)
垢版 |
2022/10/08(土) 22:56:38.37ID:932AQ9bu0
あと、"the mystery of the human mind" は、「人間の心の神秘」じゃなくて「人間の心という神秘」だと解釈しています。
同格の of ですね。
0753エキセントリックなソフィスト ◆BWpafMA3P8me (ワッチョイ 6f96-kHT+)
垢版 |
2022/10/08(土) 23:00:54.40ID:VPV9yn8M0
>>751
誤訳を盲信してるね~(^-^) もう宗教信者と同じだからなにを言ってもダメなんだろうなあ

>そんなことないですよ。

こんな反論で納得できるわけがないですね。ちゃんとした証拠を出しましょう。
「関係代名詞whatは、thing thatよりもall the で解釈すべきである」という権威ある文献を出して来たら、再考してあげてもいいですよ。
0755ブタ耳 (ワッチョイ 4ee3-3fXN)
垢版 |
2022/10/08(土) 23:11:21.12ID:932AQ9bu0
>>753
すいません。「そんかことないですよ」じゃなくて「それ誤解」とレスすべきでした。

>「what」を関係形容詞で解釈するときだけである。辞書や参考書でもそうなっているはずだ。
>関係形容詞のwhatのみが「all」で解釈できるのだ。これ重要

それ誤解。

権威ある文献とか今更探さなくても、私の過去レスたどれば、判断に必要なことはたぶん全部書いてあります。
テテンさんもこの件に関してまともなこと書いてましたよ。
全部読み返し来たらどうですか?
0757エキセントリックなソフィスト ◆BWpafMA3P8me (ワッチョイ 6f96-kHT+)
垢版 |
2022/10/08(土) 23:25:25.58ID:VPV9yn8M0
>>755
だから、具体的に書いてみて。全部論破してあげるから。どうしてそんなあいまいな逃げるような反論ですませようとするの?

関係代名詞whatは、どっちで書き換えるのがふつう?「thing(s) that」か「all the」かどっち?
1万人中9999人が 「things that」を選ぶよ。これに抗うのは、もう宗教信者レベルだと考えます。

>>756
「人間の心という神秘」という訳し方のみが正解で、「人間の心の神秘」は間違いだとする感覚にも問題がありますね。

なんか宗教信者と議論している感じなので、いくら正論を提示してもラチがあかないので、もう相手にするのやめるね (つ∀-)オヤスミー
「関係代名詞whatは、thing thatよりもall the で解釈すべきである」という権威ある文献を出して来たら、相手してあげる。
0758ブタ耳 (ワッチョイ 4ee3-3fXN)
垢版 |
2022/10/08(土) 23:40:10.67ID:932AQ9bu0
>>757
>1万人中9999人が 「things that」を選ぶよ。これに抗うのは、もう宗教信者レベルだと考えます。

このスレだけでも、OED氏、テテン氏が私の解釈に賛成してくれてるよ。
他にも賛同者結構いるんじゃないのかな。アンケートでもとりますか?
ああ、あとこの小説訳した真野氏がいるね。
0759ブタ耳 (ワッチョイ 4ee3-3fXN)
垢版 |
2022/10/08(土) 23:46:56.22ID:932AQ9bu0
>>757
>関係代名詞whatは、どっちで書き換えるのがふつう?「thing(s) that」か「all the」かどっち?

ふつうは thing(s) that で書き換えて意味が通じる場合が多いでしょうね。
それとは別に、what だけでも、「全部」というニュアンスを含む場合があるということを、どうしてそれほどまで頑なに認めようとしないのか?
理解に苦しみます。

そういえば自説の間違いを一向に改めようとせず、結局さんざん叩かれてスレを去っていった人がいたような気がしますが、その人の二の舞を踏まないほうがいいんじゃないですか?
さもないとまたコテを変えるはめに、おっとこれは失言でした。
0760ブタ耳 (ワッチョイ 4ee3-3fXN)
垢版 |
2022/10/08(土) 23:53:03.80ID:932AQ9bu0
そういえば最近、関係代名詞 の what を辞書で調べていて、「全部」という説明が書いてあるのを見て、「おっ」と思ったことがあったんだけど、今手元の辞書を見ても見当たらない。
ということは、その辞書は別の場所で見た Wisdom だった可能性が高い。

Wisdom 持ってる人、確認してみてくださいませんか?
もし what 単独で「全部」という訳が載ってたら、例文付きで引用してください。
よろしくお願いします
0761名無しさん@英語勉強中 (テテンテンテン MM86-rqSc)
垢版 |
2022/10/08(土) 23:59:46.78ID:FjrxXkjaM
>>この「what」を「すべての(all the)」で解釈できるのは、
>>「what」を関係形容詞で解釈するときだけである。
>>辞書や参考書でもそうなっているはずだ。
>>関係形容詞のwhatのみが「all」で解釈できるのだ。

『英文法精解』木村明 p.135 ()は私の注です。
what には、このような単純な用法(the thing(s) which と同様な意味を表す用法のこと)
のほかに、先行詞を、all;as much … as;everyなどの語句で強めたのと、同じ意味を持つ用法がある。
(意味と言ってるだけで、文法的に完全に同じということは言っていない)

all the money that I have 私のもっているすべてのお金
what I have 私のもっているすべてのもの
(矢印で’all the money that’と’what’が対応させている)

He spends what(=all that) he earns in a day. ※この()は私の注ではなく、引用そのまま
彼はもうけるだけ、みんな一日で使ってしまう。

He gave out what(=all that) he had to his son. ※この()は私の注ではなく、引用そのまま
彼は持っているだけ、みんなむすこにやってしまった。

※私は義務、必要の意味合いが関係詞節にあると、「全部」という意味合いになりがちだと予想してけど
’have’なのかな?

つか、ソフィスト先生ってあんまり大したことないね。
ホモ太郎が褒めはやしたててたような気がしたが。。。
0762名無しさん@英語勉強中 (ワッチョイW 3a8a-1hu+)
垢版 |
2022/10/09(日) 00:01:18.30ID:/qw+X1u60
>>760
これ?

する物[事]は何でも[すべて](anything that)
▶You may take what you want.
君が欲しいものは何でも持っていってよろしい
▶Show me what you’ve got in your pockets.
ポケットに入っているものをみんな見せなさい
▶We’ll do what we can to help you.
お手伝いできることは何でもやりましょう
0763ブタ耳 (ワッチョイ 4ee3-3fXN)
垢版 |
2022/10/09(日) 00:19:21.89ID:9R+4fEMC0
>>761
おおー、分かりやすい例ですねー。

あと、「文法的に完全に同じということは言っていない」。ここ重要ですよね。
>>615でも同様のことも同様のことを言ってて、激しく同意しました。

>※私は義務、必要の意味合いが関係詞節にあると、「全部」という意味合いになりがちだと予想してけど
>’have’なのかな?
そうなんですよね、どういう場合に「全部」という意味合いになるのか、その条件を詳しく述べているような参考書なり研究なりがあればいいんですけどね。

>>762
Wisdomですか?「全部」って日本語書いてありませんでした?
たぶんその [すべて] が、記憶の中で歪曲されて「全部」になっちゃってるんですね。
つい数日前のことなのに情けない…
ともあれ、ありがとうございました。
0764名無しさん@英語勉強中 (テテンテンテン MM86-rqSc)
垢版 |
2022/10/09(日) 00:28:43.18ID:yXmIu8c4M
つか
上で引用されている
↓これで、もう勝負はついてるんだけどね

安藤 「現代英文法講義」 p.195 から引用します
*****
what
[B] = anything that, as much as
(5) I will do what (= anything that) I can. (私にできることは何でもします)
(6) Give me what (= as much as) there is. (あるだけのものを全部ください)
*****
0765名無しさん@英語勉強中 (テテンテンテン MM86-rqSc)
垢版 |
2022/10/09(日) 00:33:17.18ID:yXmIu8c4M
で、安藤氏はこの直後にこう言っている

「この意味では、限定詞にも用いられる」と。
「この意味では」ってね。

そこで挙げられている例文が
Show me what books(=all the books that) you have.
I gave him what little money I had.
What few friends he had helped him.
0766ブタ耳 (ワッチョイ 4ee3-3fXN)
垢版 |
2022/10/09(日) 00:42:59.61ID:9R+4fEMC0
つか、小説の原文自体が、それ以外の解釈を許さないだろ、と。
せっかく前提となる文脈まで要約して、段落丸ごと和訳してみせたのに。
さらにプロの翻訳者の和訳まで引用したのに。
あと、せっかく「ドヤ顔の高卒ブタ」のAI生成画像用意しておいたのに。

のに、のに、のに。
0767ブタ耳 (ワッチョイ 4ee3-3fXN)
垢版 |
2022/10/09(日) 00:50:28.15ID:9R+4fEMC0
有効に使う機会は訪れそうにないし、この際せっかくだから貼っとくわ。
[DALL-E] https://imgur.com/7BEMbxj

prompt: "a portrait of a pig grinning proudly, which has just graduated from high school, centered, by Ansel Adams"
0769名無しさん@英語勉強中 (ワッチョイW b39d-ZDf4)
垢版 |
2022/10/09(日) 06:30:29.89ID:QAtEHrZU0
関係代名詞で、さらに文脈からみて「すべて」の用法を選んでよいのでは
0770ブタ耳 (ワッチョイ 4ee3-3fXN)
垢版 |
2022/10/09(日) 09:55:58.50ID:9R+4fEMC0
OED 2d edition CD-ROM (v.4.0)
what
* C. Relative and allied uses.
** I. pron.
*** 2. With special implications. a. Expressing quantity or amount: So much (or many) as, as much as.
(用例は古すぎるんで省略。1646年から1789年のものしか出てない)

こういう意味合いの後に、can とか have とかなどの意味をもつ構成要素が連結したときに、「すべて」という意味合いが生じてくるのではないだろうか?

『実践ロイヤル英文法』の記述「~するだけの量[数]」は、OEDの記述 so much (or many) as, as much as を踏まえているのかもしれない。
*****
129 what の用法
129 A (4) 「~するだけの量[数], ~しようと, ~であろうと」の意味で
We all should do what we can.
(私たちは皆できるだけのことをすべきです)
-中略-
●このように使われるwhatは, whatever [=anything wich [that]]の意味に極めてちかいが, whatever の方が幾分か強調的な表現になる。
whateverについては 140C(p.286)を参照のこと.
***** 綿貫陽・マーク・ピーターセン 『実践ロイヤル英文法』 p.272

で、同書の whatever を見ておくと
*****
140 C whatever
-中略-
(1) 「~するものは何でも」の意味で名詞節を導く。
-中略-
Whatever you say will be recorded on this desk.
(言うことはすべてこのディスクに録音されます)
****
0772ブタ耳 (ワッチョイ 4ee3-3fXN)
垢版 |
2022/10/09(日) 10:19:28.49ID:9R+4fEMC0
以下、COBUILDから引用

what
5 [CONJ]
You use what to indicate that you are talking about the whole of an amount that is available to you.
 ・ He drinks what is left in his glass as if it were water...

★the whole of an amount★
0775ブタ耳 (ワッチョイ 4ee3-3fXN)
垢版 |
2022/10/09(日) 11:49:21.95ID:9R+4fEMC0
>>773
徹底的にやるつもりですよ。

ところで、このwhatの件についてあなたの見解をお聞きしてもよろしいですか?
私の解釈とソフィスト氏の解釈、どちらが正しいとお考えですか?

>>774
お、スプッッ、スッップの人だ。 まだいたの?
おじさん構文の人って誰?テテン氏(高卒2号)?

私(高卒3号)は4連休3日目なんで時間あるから大丈夫だよ。
0776ブタ耳 (ワッチョイ 4ee3-3fXN)
垢版 |
2022/10/09(日) 12:13:03.82ID:9R+4fEMC0
>>757
>なんか宗教信者と議論している感じなので、いくら正論を提示してもラチがあかないので、もう相手にするのやめるね (つ∀-)オヤスミー
>「関係代名詞whatは、thing thatよりもall the で解釈すべきである」という権威ある文献を出して来たら、相手してあげる。

この発言を盾にとって、こちらが「権威ある文献を出す」までレスしないことを正当化しているのかもしれないが、この発言にはいくつも問題がある。

1)「なんか宗教信者と議論している感じ」 議論の筋道を外れた人格攻撃。現在巷で問題になっている某統一教会を暗に匂わせているところが悪質。
2)「いくら正論を提示してもラチがあかない」 正論ではないものを正論だとして提示しているのだからラチがあかないのは当然。
3)「関係代名詞whatは、thing thatよりもall the で解釈すべきである」 これが一番問題である。こちらの主張を歪めている。
こちらが主張しているのは、
「関係代名詞 what は 普通 の用法 [the thing(s) that] の他に、[all that] のように解釈できる特殊なケースがある」ということ。
単純化すると、「what には普通の用法の他に特殊な用法がある」ということ。
これをソフィスト氏はその名にふさわしく詭弁的にこう歪曲する。
「what は、普通の用法より特殊な用法で解釈すべきである」
もちろんそんなことを述べている「権威ある文献」は存在しないので、こちらから提出することも不可能である。
不可能な課題を一方的におしつけておいて、その課題をはたさなければ議論には応じないという。
議論からの卑劣な逃亡方法というべきである。
0781ブタ耳 (ワッチョイ 4ee3-3fXN)
垢版 |
2022/10/09(日) 13:42:50.45ID:9R+4fEMC0
>>777
>>733に英文を長めに引用しています。

To his library in Cable House he will add volumes in which are contained what human knowledge, in the 1820s, has to offer on the mystery of the human mind.
関係代名詞 what を含むこの英文に対しての,

1)私の翻訳
「ケーブルハウスにある彼の書斎には新たに数々の書物が加えられることになる。その書物には、人間の心という神秘に関して、1820年代において人類の知識が提供しうるすべてが収められている。」

2)真野泰氏の翻訳
「ケイブル邸の彼の蔵書に新たに加わる一群の書物には、1820年代の人類が、人間精神の神秘についてすでに得ている知識の全部が詰まっている。」

この訳文中の「すべて」と「全部」は、関係代名詞 what の解釈として適切だと考えますか?

返答しだいでソフィスト氏がクールダウンするような気もしますが、どうなんでしょうね。
もっともすでにノックダウンされてる状態なのかもしれませんが。
0782ブタ耳 (ワッチョイ 4ee3-3fXN)
垢版 |
2022/10/09(日) 14:07:17.31ID:9R+4fEMC0
>>777
おおまかな文脈は>>732に示してあります。

返答が無い場合、ソフィスト氏に対する何らかの配慮で意図的に立場を曖昧にしているのだろうと憶測させていただくことになると思います。
0784ブタ耳 (ワッチョイ 4ee3-3fXN)
垢版 |
2022/10/09(日) 14:18:43.07ID:9R+4fEMC0
>>777
as/more .... を修飾してるっていうのは、as/more それぞれを単独に修飾してるということですか?
それとも、その後に続く .... を含めたもの [as/more ....] を修飾してるということですか?

具体的に言えば、
He is as tall as his father.
で、[as his father] が修飾してるのは、 [as] 単独なのか、それとも [as tall] なのか、ということです。

この件、私とテテン氏が論争してた問題です。
あまりに紳士的な論争だったせいか、一人二役の自演だと思った人がいるようです(>>729)。
0788😉三年英太郎🌈 ◆3CZBjOt3.Y (ワッチョイW ca86-zOhc)
垢版 |
2022/10/09(日) 14:46:35.37ID:2uGe830o0
人類の知識は全部書けるっつーのは啓蒙主義の百科全書派なんだよ
1820年代はそーゆーのがとっくに終わって、ロマン主義の時代なのよ
青い花探求の時代なの
分かるかなー?
0790😉三年英太郎🌈 ◆3CZBjOt3.Y (ワッチョイW ca86-zOhc)
垢版 |
2022/10/09(日) 14:52:07.50ID:2uGe830o0
>>784
たぶんあーた、ロマン主義の文学も音楽も美術も知らないでしょ

フリードリヒもヴァーグナーもノヴァーリスも知らない人が、英語の一文だけ読んで、全てがどうのとか変なとこで争って、ちゃんちゃらおかしいです😂
0793😉三年英太郎🌈 ◆3CZBjOt3.Y (ワッチョイW ca86-zOhc)
垢版 |
2022/10/09(日) 15:03:31.83ID:2uGe830o0
訳文の、すべて、は原文にございません

それを、あるという前提に立って、whatに入ってるに違いないとかいって辞書引き回ってもムダ

説明すべきことは全て終わったのでこれ以上はやりません😌🤷🏻‍♂
0795😉三年英太郎🌈 ◆3CZBjOt3.Y (ワッチョイW ca86-zOhc)
垢版 |
2022/10/09(日) 15:32:02.67ID:2uGe830o0
他人に質問しておいて自分が欲しい答えが返ってこないとキレる人ってなんなん?🤷🏻‍♂

それに誰がどのレスを読むか、どのレスに答えるかはその人の自由であって、あーたに指図される覚えはないよ?

おれは「whatすべて問題」に一切かかわって来なかったのにね🤷🏻‍♂
0797ブタ耳 (ワッチョイ 4ee3-3fXN)
垢版 |
2022/10/09(日) 15:42:16.31ID:9R+4fEMC0
いや、別に切れてないですよ。失望しただけです。

AIにロマン主義風の絵画を作成してもらったところなかなか見事な絵が出来上がったんで貼っておきます。
[DALL-E] https://imgur.com/yXThyHA

prompt: a full‐length figure of Siegfried seen from behind, standing at the edge of a cliff, looking over the endless blue sea, with a f blue flowers in his hand, intricate, oil painting by Caspar David Friedrich.
0798名無しさん@英語勉強中 (ワッチョイ d393-6jYr)
垢版 |
2022/10/09(日) 15:54:48.39ID:4ooGR61f0
比較級の比較構文と対比させて論ずることがそもそもおかしい
[more+形容詞] は [副詞の比較級+形容詞] ではなく [形容詞の比較級] である
だからthan節が修飾するのは [more+形容詞]=[形容詞の比較級] である
一方、as as 構文の [as+形容詞] は [副詞+形容詞] である。そもそも構造が異なるのだから対比させて論ずるのは不適当
0799ブタ耳 (ワッチョイ 4ee3-3fXN)
垢版 |
2022/10/09(日) 15:56:17.92ID:9R+4fEMC0
タイポ修正: with a blue flower in his hand

あと、ついでと言ってはなんですが、
>>460
ようやくコップをひっくり返すことに成功しました。
例の詩の私の脳内イメージにかなり近いところまできました。
https://i.imgur.com/AVwq82G.jpg

prompt: an upside-down glass which captures curled white smoke within it, placed upside-down on a wooden table, centered, realistic.
これ文法的には色々とおかしいですが、upside-down を2度重ねたのが有効だったのかもしれません。
0801ブタ耳 (ワッチョイ 4ee3-3fXN)
垢版 |
2022/10/09(日) 16:33:15.51ID:9R+4fEMC0
>>800
>>784でも質問しましたけど、それ [as him] が修飾するのは、DegP [as tall] を修飾する、という理解でいいんですか?
Deg [as] 単独を修飾するのではなく。
0802ブタ耳 (ワッチョイ 4ee3-3fXN)
垢版 |
2022/10/09(日) 16:35:40.74ID:9R+4fEMC0
あ、ごめんなさい、日本語変になった。仕切り直し。

それって、 [as him] が DegP [as tall] を修飾する、という理解でいいんですか?
Deg [as] 単独を修飾するのではなく。
0803名無しさん@英語勉強中 (ワッチョイ 83b9-tCk3)
垢版 |
2022/10/09(日) 17:46:01.17ID:tXJhIjVN0
>>788
なるほど。勉強になる。
wikiにガロアのところで
"シュヴァリエは遺書に従って1832年に『百科評論雑誌』(Revue encyclopédique)に「死者小伝」
と題したガロアの論文等を掲載"
(wikipedia日本版のエヴァリスト・ガロアの項目 より)
とあった。百科全書にガロアの書いたものが載ったのかと
思ってた
0804名無しさん@英語勉強中 (ワッチョイ 83b9-tCk3)
垢版 |
2022/10/09(日) 18:06:38.84ID:tXJhIjVN0
whatについては
(…する)ものは何でも[どれほどでも](whatever)

直訳すると、
To his library in Cable House he will add volumes in which are contained
what human knowledge, in the 1820s, has to offer on the mystery of the human mind.
人間の心の神秘についてのことで、人間の1820年代での知識が
提供できることを書いている本を、彼はケーブルハウスにある自分の蔵書の
中に加えようとする。
0805名無しさん@英語勉強中 (ワッチョイ 83b9-tCk3)
垢版 |
2022/10/09(日) 18:09:22.07ID:tXJhIjVN0
知識の全部が詰まっている。
は正確には間違っている。
すでに全部揃っているのではなくて、見つけ次第、買って加えるということ。
0807名無しさん@英語勉強中 (ワッチョイ 83b9-tCk3)
垢版 |
2022/10/09(日) 18:19:26.26ID:tXJhIjVN0
whatをウィズダムで引いてみたら

…する物[事]は何でも[すべて](anything that)
▶You may take what you want.
君が欲しいものは何でも持っていってよろしい
▶Show me what you’ve got in your pockets.
ポケットに入っているものをみんな見せなさい
▶We’ll do what we can to help you.
お手伝いできることは何でもやりましょう.
と書いてあった
0811😉三年英太郎🌈 ◆3CZBjOt3.Y (ワッチョイW ca86-zOhc)
垢版 |
2022/10/09(日) 18:59:33.43ID:2uGe830o0
OEDのお話も出てる

もちろん、世界最大の英語辞典のことで、あの偉大な書物の名をコテにつける お こ が ま し い 人物のことではない

OEDという字面を見て、あの人が脳裏によぎらないようにしたいものである🤭
0812ブタ耳 (ワッチョイW 4ee3-7V8K)
垢版 |
2022/10/09(日) 19:20:39.09ID:9R+4fEMC0
ブロックされたような気がするので煽ってみるテスト。
>>811
読解力ではOED氏の足元にも及ばないんじゃないですか?
>>734程度の英文もまともに読めない分際で、おこがましいとはどの口が言うのかと、ちゃんちゃらおかしいです😂
0813😉三年英太郎🌈 ◆3CZBjOt3.Y (ワッチョイW ca86-zOhc)
垢版 |
2022/10/09(日) 19:26:22.28ID:2uGe830o0
ドラえもんの道具とかネギまのアーティファクトなら分かるよ
そーゆーすごい本があって、なんか事件を解決してくとかさー
でも違うんでしょ、その本🤦🏻‍♂

じゃあ、具体的に1820年代に人の心のなんちゃらを余すことなく書き記した本てどんながあるのよ? 数冊あげてくんない?🤷🏻‍♂
0816名無しさん@英語勉強中 (ワッチョイ 4ee3-3fXN)
垢版 |
2022/10/09(日) 19:38:20.77ID:9R+4fEMC0
>>813
私のレスから(>>537)から引用します。

>一般に、現実には存在しえないもの(例: 全ての情報が記載された蔵書)であったとしても、そういうものが存在すると述べる言語表現が不可能であることにはなりません。
>分かりやすい例でいえば、「何でも知っている人間」など現実には存在しえないのに、「あいつは何でも知っている」という言語表現は可能です。
0817😉三年英太郎🌈 ◆3CZBjOt3.Y (ワッチョイW ca86-zOhc)
垢版 |
2022/10/09(日) 19:43:23.81ID:2uGe830o0
だからドラえもんなら分かるって言ってんじゃん
時代考証を考えた小説だとしたら、そんなことを書いたらとたんにリアルさが欠けて萎えるね

what は無標ではすべての意味はない。まず無標で取って意味がとれるなら有標にする必要ありますか??
0820ブタ耳 (ワッチョイW 4ee3-7V8K)
垢版 |
2022/10/09(日) 19:51:49.63ID:9R+4fEMC0
>>817
時代考証を考えた小説とはちょっと違います。
語り手は50代の歴史教師で、本来の歴史授業から大幅に脱線し、生徒たちに、自分や自分の家族にまつわる過去の物語を様々なエピソードを交えて語っていくというスタイルで書かれた小説です。
もちろん語り手もトマス・アトキンソンも、想像上の架空の人物です。
この歴史教師が語る物語がどういう性質のものかは、先に紹介した映画をちょっと見てみれば分かると思いますよ。
0822😉三年英太郎🌈 ◆3CZBjOt3.Y (ワッチョイW ca86-zOhc)
垢版 |
2022/10/09(日) 20:17:12.54ID:2uGe830o0
諌めただけなのに、ムダに巻き込まれてしまった🥺

ウィキペ見るとポモ文学らしい

ポモ文学の特徴(のひとつ)ってあいまい性にあるんでしょ。読者がそう読んでそう思うならそれでいいんでね(誤解も理解です)。それを「これが絶対の読みだ!」なんて言われたらポモの死だよ🥺
0824ブタ耳 (ワッチョイ 4ee3-3fXN)
垢版 |
2022/10/09(日) 20:28:31.68ID:9R+4fEMC0
>>822
曖昧性がポストモダン文学の特徴だったとしても、この一節中の what の文法的解釈までが曖昧性を帯びる必然性はまったくありません。
あなたまだ>>731-733を丁寧に読んでないでしょ?
0825ブタ耳 (ワッチョイ 4ee3-3fXN)
垢版 |
2022/10/09(日) 20:33:48.92ID:9R+4fEMC0
ところで、いまさらですけど、>>228でこの小説の話題を初めてこのスレに持ち込んだ人っていったいどなたですか?
まだこのスレ見てます?なんか大変なことになってしまいましたがw
0826😉三年英太郎🌈 ◆3CZBjOt3.Y (ワッチョイW ca86-zOhc)
垢版 |
2022/10/09(日) 22:06:39.00ID:2uGe830o0
>>824
該当の箇所のどこにもwhat=allが絶対正しいとするだけの理由がないじゃん🤔
強いて言えば日本語訳にあるというだけ

自分はそうだと思い込んでるだけで伝わってない
もはや宗教と誰かが言ったのは同意

それ以上の根拠が出せないならおれを説得できないんでもう良いです🙇🏻‍♂
0827エキセントリックなソフィスト ◆BWpafMA3P8me (ワッチョイ 6f96-kHT+)
垢版 |
2022/10/09(日) 22:07:21.06ID:02qfBh9k0
では反論いきま~す (^-^)
What human knowledge has to offer on 人間の心の神秘 are contained in the volumes. が問題文。
このwhatの意味が何であるかの問題。

>>764
だから、anything that, as much asだと動詞は単数扱いで、「are」にはならないの。筆者は、anything that, as much asを意図してないわけ。ダメダメの反論。

>>765
関係形容詞ならallと訳せると>>749で言ってるだろ。関係代名詞の事柄を関係形容詞で証明しようとしている点でダメダメの反論。
0828エキセントリックなソフィスト ◆BWpafMA3P8me (ワッチョイ 6f96-kHT+)
垢版 |
2022/10/09(日) 22:07:29.28ID:02qfBh9k0
>>770
用例が古いのなら今回の英文には当てはまらない可能性が濃厚。はい、ダメダメの反論。

>>771
anythingもeverythingも単数扱いだから、「are」にはならない。はい、ダメダメの反論。>>772も単数で受けてるので「are」で受ける例とは言えない。はい、ダメダメの反論。
0829😉三年英太郎🌈 ◆3CZBjOt3.Y (ワッチョイW ca86-zOhc)
垢版 |
2022/10/09(日) 22:13:50.56ID:2uGe830o0
あともう一つね

長文の人が後で何を言ったのかおれは読んでないけど、彼は最初に、whatは具体的には beauty とtruth とか単数で表せるものって言ったんだよね

彼の解釈で言えばなおさらあれやこれやのすべてじゃないじゃん🤣
援護になってない😂😂
0831ブタ耳 (ワッチョイ 4ee3-3fXN)
垢版 |
2022/10/09(日) 22:28:43.69ID:9R+4fEMC0
>>828
あいかわらずwhatを別の表現で言い換えた語の文法的性質(単複)にこだわってるんですか?
anything とか everything とか、what を近似的に他の語に言い換えてるだけで、 what が anything とか everything とかの文法的性質を受け継ぐわけではないことは、すでにテテン氏が >>615 でも >>761 でも指摘してますよ。
0832名無しさん@英語勉強中 (テテンテンテン MM86-rqSc)
垢版 |
2022/10/09(日) 22:33:58.04ID:F4sTpvfKM
>>789
作者が苦笑するのはお前の解釈だろうな
啓蒙主義思想からロマン主義に移ってるから
「全部」の解釈は間違いとか・・・
完全なアホだな
0833エキセントリックなソフィスト ◆BWpafMA3P8me (ワッチョイ 6f96-kHT+)
垢版 |
2022/10/09(日) 22:40:15.86ID:02qfBh9k0
□□□
まとめ
□□□
英英辞典では、関係代名詞は基本的に「all」の意味は持っていない。日本人の書いた参考書を絶対視してはいけない。

英英辞典
https://www.merriam-webster.com/dictionary/what
https://www.collinsdictionary.com/dictionary/english/what
ここでもあるように、関係代名詞のwhatは、基本的に「the thing or things that」と「anything or everything that」(whatever)、「~するだけの」(as much/many as)の意味なのである。
この中で「are」で受けることができるのは、「things that」と「as many as」だけだが、後者は「ありったけの」という言葉と入れ替えられる文脈で使うのである。

その本の訳者は、「anything or everything that」で解釈したのかもしれないが、それだと「are」で受けるのはおかしい。だから、anything/everythingはで解釈するのは間違い。
もし、その翻訳者が、関係形容詞と見誤ったなら、has toを「~しなければならない」と訳さなくなるのでおかしい。やっぱり、それも間違い。

映画・テレビの字幕とかの翻訳なんか間違い結構あるよ。プロの翻訳者は英文法のプロじゃないからね。たまには間違うもんだと思わなくちゃ。どんなに有能な翻訳者でも間違いはするのだ。
結論。 私の言う「things that」で解釈するのが正しい。ここまで読んでまだわからないのなら、「判断力が壊れている」としか感じられない。
0834名無しさん@英語勉強中 (テテンテンテン MM86-rqSc)
垢版 |
2022/10/09(日) 22:41:31.46ID:F4sTpvfKM
つか「ソフィスト」とか名乗ってる奴、恥ずかしくねーの? 苦笑

all が「全て」という意味で使われていても
それを受けるbe動詞は、isであったりareであったりすることは
ネイティブのアンケートを元に反論済み。
複数のものかどうかという話者の感覚で左右される。
0835ブタ耳 (ワッチョイ 4ee3-3fXN)
垢版 |
2022/10/09(日) 22:41:41.80ID:9R+4fEMC0
>>828
>この「what」を「すべての(all the)」で解釈できるのは、「what」を関係形容詞で解釈するときだけである。辞書や参考書でもそうなっているはずだ。
>関係形容詞のwhatのみが「all」で解釈できるのだ。これ重要。(>>749

>>772のCOBUILDの説明と例文だけでも上の珍説を否定できますよね。
0836エキセントリックなソフィスト ◆BWpafMA3P8me (ワッチョイ 6f96-kHT+)
垢版 |
2022/10/09(日) 22:48:54.09ID:02qfBh9k0
まず、常識でもわかるよね。
What human knowledge has to offer on 人間の心の神秘 are contained in the volumes.

A:この一群の書物の中には、人知が人間の心の神秘に寄与している「すべて」が収められているのだ。
B:この一群の書物の中には、人知が人間の心の神秘に寄与している「いろいろなもの」が収められているのだ。

どっちが、ウソっぽい? 「人知が人間の心の神秘に寄与している「すべて」が漏れなくこの本に載ってるよ」って、ペテン師の台詞じゃないか (^-^)

まず、本当は、文法を考えるまでもなく、この時点でBが適切だとわからないといけない。これでも分からない人は、もうダメかもしれない。\(^o^)/
0838ブタ耳 (ワッチョイ 4ee3-3fXN)
垢版 |
2022/10/09(日) 22:51:08.43ID:9R+4fEMC0
>>836
文脈からそこだけ切り離して判断しようとしてる段階で、あなたが文脈を読めてないことが曝露されちゃってますよ
0839名無しさん@英語勉強中 (テテンテンテン MM86-rqSc)
垢版 |
2022/10/09(日) 22:53:28.79ID:F4sTpvfKM
>>もし、その翻訳者が、関係形容詞と見誤ったなら、has toを「〜しなければならない」と訳さなくなるのでおかしい。やっぱり、それも間違い。

なんか前のそんな話してたけど、
だれもそんなアホな解釈主張してねーだろw

ただ安藤氏が
「この意味では、限定詞にも用いられる」と書いて
Show me what books(=all the books that) you have.
という例文を挙げていると言ってんの。

安藤氏が自由関係代名詞のwhatにallの意味合いがあると考えてるんじゃね
と示してるんだよ。
0840😉三年英太郎🌈 ◆3CZBjOt3.Y (ワッチョイW ca86-zOhc)
垢版 |
2022/10/09(日) 22:54:00.59ID:2uGe830o0
他の部分は同意しないが

>「人知が人間の心の神秘に寄与している「すべて」が漏れなくこの本に載ってるよ」って、ペテン師の台詞じゃないか

これだよこれ🤦🏻‍♂
文脈を見ろって言うけどさ、「すべて」を支持する文脈がどこにあるのさ?
0843名無しさん@英語勉強中 (テテンテンテン MM86-rqSc)
垢版 |
2022/10/09(日) 23:00:50.36ID:F4sTpvfKM
isじゃなくてareだからって、なんどもソフィストさん(w)が言ってるけどさぁ
>>245 とか見てないふりしてんのかな? 苦笑
そんなんで、意味は確定しねーよ。

単数か複数かなんて、感覚に左右されがちなんだよ。
volumesが複数形であることに影響された可能性もあるしね。
0844エキセントリックなソフィスト ◆BWpafMA3P8me (ワッチョイ 6f96-kHT+)
垢版 |
2022/10/09(日) 23:01:54.76ID:02qfBh9k0
>>839
だから、>>749で、関係形容詞what にはallの意味があると言ってるだろ。辞書にも載ってるあたりまえのことだ。

今回の問題は、関係形容詞ではなく関係代名詞なんだから、関係形容詞の例文を出さないでくれるかな。

ほんとなんでこんな低レベルの反論に時間を使わないといけないの?寝るね。(つ∀-)オヤスミー
0845名無しさん@英語勉強中 (テテンテンテン MM86-rqSc)
垢版 |
2022/10/09(日) 23:03:10.17ID:F4sTpvfKM
>>841
>>いつ、私が、all は areで受けないと言った。

areで受けてることが「全部」の意味でない証拠と
お前言ってんだろーが、コラ
「全部」の意味でもareで受ける可能性があると言ってんだよ。

読解力の無いのはお前だろ、アホ
0846ブタ耳 (ワッチョイ 4ee3-3fXN)
垢版 |
2022/10/09(日) 23:05:12.75ID:9R+4fEMC0
>>840
一番大事な文脈は、この老人を突き動かしている動機と、沼地干拓と医学研究で繰り返される同一の行動パターンの理解です。
それと、in the 1820's のここでのニュアンスは感知してるんですか?
とりあえずロマン主義とは無関係ですけど。
0847名無しさん@英語勉強中 (テテンテンテン MM86-rqSc)
垢版 |
2022/10/09(日) 23:05:40.68ID:F4sTpvfKM
>>844
>>関係形容詞の例文を出さないでくれるかな。

安藤氏の自由関係代名詞allについての見解を示すために
出しただけだ。
この英文のallが限定詞と主張してるなんていう
解釈こそ、関係ないので、示さないでもらえるかな。
0849エキセントリックなソフィスト ◆BWpafMA3P8me (ワッチョイ 6f96-kHT+)
垢版 |
2022/10/09(日) 23:09:32.61ID:02qfBh9k0
>>845
やっぱ、君、アホだわ。 『areで受けてることが「全部」の意味でない証拠』なんて私は一度も言ったことはない。

私は、『areで受けてることが「anything」「everything」の意味でない証拠』といっただけ。デタラメを言うのはやめてくれるかな。
ほんと、国語力つけようね。
0850エキセントリックなソフィスト ◆BWpafMA3P8me (ワッチョイ 6f96-kHT+)
垢版 |
2022/10/09(日) 23:14:06.22ID:02qfBh9k0
>>847
君に教えておいてあげよう。辞書では次のように定義されている。

関係形容詞のwhat = all (しかもウィズダムによれば「少ないながらも」の意を含有)
関係代名詞のwhatにはallの意味はない。

これがいろんな辞書が言ってることだから、あきらめなさい。これ以上反論すると、恥をかくよ。
0852😉三年英太郎🌈 ◆3CZBjOt3.Y (ワッチョイW ca86-zOhc)
垢版 |
2022/10/09(日) 23:16:52.43ID:2uGe830o0
>>846
全然分からん🤦🏻‍♂🤷🏻‍♂
あなたがそう読めるならそう読めば?というくらいの話で、人に押し付けるほどの説得力はない
もはや宗教です👆🏻

では日付も変わることだし、わたくしはこの話題はおりる
0854名無しさん@英語勉強中 (テテンテンテン MM86-rqSc)
垢版 |
2022/10/09(日) 23:20:58.30ID:6+oaRBuBM
>>849
>>『areで受けてることが「anything」「everything」の意味でない証拠』

そうやって誤魔化すのはやめようね。
つか、自分で何も言ってるかわからなくなってるのかな?

今話されているテーマは
allが「全部」という意味かどうか。

で、お前は
『areで受けてることが「anything」「everything」の意味でない証拠』
と何度も言ってるけど、それは何の為に言ってんだよ、馬鹿。

「anything」「everything」の意味でない
よって
「全部」の意味でない
って、言ってるじゃねーか、お前は。
0855名無しさん@英語勉強中 (テテンテンテン MM86-rqSc)
垢版 |
2022/10/09(日) 23:30:39.62ID:6+oaRBuBM
例えばWisdomの例文
する物[事]は何でも[すべて](anything that)
▶Show me what you’ve got in your pockets.
ポケットに入っているものをみんな見せなさい

このwhatはallという意味である。
anythingで書き換え可能、anythingに似た意味
という説明は
「全部」という意味にもなるということ。

anythingを文法的な扱いが違うからといって
「全部」という意味にならないということではない。

そう何度も言ってのに
anythingと書き換えるなんたらこうたらって
何度も言って奴なんなの?

関係ない話をしないでもらえますか?
0856名無しさん@英語勉強中 (テテンテンテン MM86-rqSc)
垢版 |
2022/10/09(日) 23:32:54.80ID:6+oaRBuBM
訂正
×anythingを文法的な扱いが違うからといって
〇anythingと文法的な扱いが違うからといって

×anythingと書き換えるなんたらこうたらって
〇anythingと書き換えるとなんたらこうたらって
0857名無しさん@英語勉強中 (テテンテンテン MM86-rqSc)
垢版 |
2022/10/09(日) 23:35:01.61ID:6+oaRBuBM
>>848
そういうなら
プロの翻訳家が間違う可能性も低いと言っておこう。
それも一人じゃない。
0858名無しさん@英語勉強中 (テテンテンテン MM86-rqSc)
垢版 |
2022/10/09(日) 23:35:43.19ID:6+oaRBuBM
(本当はプロの翻訳家ってしょっちゅう誤訳してるけどw)
0859名無しさん@英語勉強中 (テテンテンテン MM86-rqSc)
垢版 |
2022/10/09(日) 23:47:13.70ID:6+oaRBuBM
あ、失礼。
プロの翻訳家の訳は秘湯でしたね。勘違いしてました。

まず、妻のことで非常に心を痛めて、
研究に没頭して妻を救おうとしている老人の話である。
必死にありとあらゆる知識を集めようとしている様子を描写した
英文として解釈することは極めて自然なことだ。

単に、
「人間の心について書かれた本を集めている」
ということを言いたいだけなら
'what human knowledge, in the 1820s, has to offer on the mystery of the human mind'
なんて言い方をするだろうか?
中学生でもわかることだと思う。
(因みに私はofは同格ではなく「〜に関しての」という意味だと解する)
0860名無しさん@英語勉強中 (テテンテンテン MM86-rqSc)
垢版 |
2022/10/09(日) 23:50:21.55ID:6+oaRBuBM
訂正
×プロの翻訳家の訳は秘湯でしたね。
〇プロの翻訳家の訳は一つでしたね。

ミスが多くてすみません。
0861ブタ耳 (ワッチョイ 4ee3-3fXN)
垢版 |
2022/10/09(日) 23:56:08.67ID:9R+4fEMC0
He spends what he earns in a day.
He gave out what he had to his son.
Show me what you’ve got in your pockets.

ソフィスト氏、こういう文に初めて出会ったときにどう訳すのだろうか?
だいたい想像できるけど。
0862ブタ耳 (ワッチョイ 4ee3-3fXN)
垢版 |
2022/10/10(月) 00:03:22.68ID:/uVa+ohB0
で、私が、例えば、
Show me what you’ve got in your pockets.
を、「ポケットに入っているものをすべて見せなさい」
と訳したら、what に「すべて」という意味はない、とか言うんだろうね。
0863名無しさん@英語勉強中 (テテンテンテン MM86-rqSc)
垢版 |
2022/10/10(月) 00:36:38.90ID:d/PakvARM
>>848
補語を伴う繫辞は2番目のbe動詞
1番目のbe動詞は、当該英文と同様に受動態を構成するbe動詞。

20%もの人が選択しているものを
「間違い」なんて断じていいものだろうか?

「複数の事柄を意識したall」を三人称単数扱いしないと
「間違い」だとする理論的な根拠でもあるのだろうか?
こんなのは実際の運用状況によって修正され得るルールだろ。
0864名無しさん@英語勉強中 (テテンテンテン MM86-rqSc)
垢版 |
2022/10/10(月) 00:59:47.37ID:d/PakvARM
単に
「心の神秘について書かれた本を集める」なら
he will add volumes the mystery of the human mind.
くらいの英文で済むはずだ。
he will add volumes in which are contained
what human knowledge, in the 1820s, has to offer on the mystery of the human mind.
なんていう仰々しい言い回しは、
本を媒介に、その当時における、ありとあらゆる情報を集めようとしたことを表現しているのだろうと
中学生でも読み解くであろう。
0865名無しさん@英語勉強中 (テテンテンテン MM86-rqSc)
垢版 |
2022/10/10(月) 01:00:51.79ID:d/PakvARM
×he will add volumes the mystery of the human mind.
〇he will add volumes about the mystery of the human mind.
0866ブタ耳 (ワッチョイW 4ee3-7V8K)
垢版 |
2022/10/10(月) 01:04:05.81ID:/uVa+ohB0
>>864
いや、中学生には分からないと思うよw
関係代名詞のwhatは学習指導要領に入ってないし。

文脈捉えるって結構難しい課題なんだろうね。
ソフィスト氏が読めないのはまだ納得できるとして、英太郎さんまで読めない人だったのは本当に驚いたわ。
0867ブタ耳 (ワッチョイW 4ee3-7V8K)
垢版 |
2022/10/10(月) 01:15:50.34ID:/uVa+ohB0
1820年代で、ロマン主義とか言い出して、お前フリードリヒもワーグナーもノヴァーリスも知らねーだろ、とか。
追い詰められ橋下徹みたいな論点のすり替えとチープな知識自慢。
いったい何で俺がフリードリヒもワーグナーもノヴァーリスも知らないなどと考えたんだ?
0868ブタ耳 (ワッチョイW 4ee3-7V8K)
垢版 |
2022/10/10(月) 01:41:16.52ID:/uVa+ohB0
既に>>732でも触れておいたことだが、作者は巧妙に伏線を張っていて、ある箇所でさりげなく「1800年から1815年」においては、「医学はまだ初期の段階にある」と述べている。” but medical science is still in its infancy"
この伏線が in the 1820’s という語句の登場で回収され、読者は当時の医学が提供できる知識がごく限られたものであったということが即座に了解できる。
そんな限られた知識でも、藁にもすがるように、あらんかぎり集めようとした。そういう哀れな老人の努力が自ずと推察される仕組みになっているのだ。
見事な構成というべきだろう。
0869名無しさん@英語勉強中 (テテンテンテン MM86-rqSc)
垢版 |
2022/10/10(月) 01:44:22.84ID:d/PakvARM
>>849
>>やっぱ、君、アホだわ。 『areで受けてることが「全部」の意味でない証拠』なんて私は一度も言ったことはない。、

>>837
>>それisで受けているでしょ。だから、ダメダメ反論。areで受けてるのを探してきなって。

いろいろ言い訳してるけど
やっぱ、言ってるよねw
0870名無しさん@英語勉強中 (テテンテンテン MM86-rqSc)
垢版 |
2022/10/10(月) 02:08:58.80ID:d/PakvARM
>>837
>>それisで受けているでしょ。だから、ダメダメ反論。areで受けてるのを探してきなって。


つか、コイツさぁ。
それはisで受けてるけど、当該英文はareだから、ダメって言ってるけど
isで受けてる場合は「全部」の意味で使われることがあるって認めてんだよな。

じゃあ
>この「what」を「すべての(all the)」で解釈できるのは、「what」を関係形容詞で解釈するときだけである。辞書や参考書でもそうなっているはずだ。
>関係形容詞のwhatのみが「all」で解釈できるのだ。これ重要。
というコメントについて
謝罪して修正してくれないかな。ソフィスト大先生。
0871ブタ耳 (ワッチョイW 4ee3-7V8K)
垢版 |
2022/10/10(月) 02:10:00.40ID:/uVa+ohB0
いちばん救いようがないのは>>836の書き込み。
これまで私が文脈を読む大切さを何度も強調してきたのに、そういう文脈を一切無視した文を二つ並べて「どちらが自然か」などと問うている。
自分がまったく文脈を考えていないことをそうやって暴露しているのだが、そのことに気づいていない。

「もうダメかもしれないの」はお前だろうが。
アボカド、もとい、アホかと。
0872ブタ耳 (ワッチョイW 4ee3-7V8K)
垢版 |
2022/10/10(月) 02:19:25.46ID:/uVa+ohB0
いやー悪口言うのは楽しいなー
ああー、どんどん心が汚れていくー
これはいかんなー
明日の休日はウ・ヨンウ見て心を浄めてもらおー

>>870
お互い無理せず行こうねー
0873名無しさん@英語勉強中 (テテンテンテン MM86-rqSc)
垢版 |
2022/10/10(月) 02:27:44.51ID:d/PakvARM
>>863
訂正
×「複数の事柄を意識したall」を三人称単数扱いしないと
〇「複数の事柄を意識したallという意味合いのwhat」を三人称単数扱いしないと


あとイイタイコトは
ホモ太郎がロマン主義がどうたらとか言っているけど
文脈上
この時代に人類の英知が発見した知識というのは
主に科学的知識、特に医学的知識のことを指しているのでしょう。
彼は妻を助けたいと思って本を収集しているのだし、
続く文章
「こんどはさらに入り組んだ、脊髄と小脳の地形について読みふけるのである。読んでみれば脊髄と小脳もやはり、水路と導管の網状組織をもち、やはり、液体が常時各所に供給されることにより成り立っている。」
からもそう言えるでしょう。

で、その当時もコツコツと研究をしている科学者がいて
そして、彼らから教えを請おうとしている当主人公のような人もいるわけです。

彼らがそのような知識がテキスト化不可能と考えているのでしょうか?
ロマン主義とか全然関係ない話だろ。
何が時代考証だよwwww

なんか
この深夜のレス群も間違いなく自演扱いされるだろうねw
でも、ブタ耳さんとは比較級の解釈の件で見解が相違しております。
そして、ひそかに、ブタ耳さんが、ご自分の説こそが「シンプル」だとおっしゃていることにカチンときておりますw
俺の説の方がシンプルだろ!と思っておりますが、この件はもう言及しないことに決めています。
0874名無しさん@英語勉強中 (テテンテンテン MM86-rqSc)
垢版 |
2022/10/10(月) 02:32:50.52ID:d/PakvARM
>>872
私ももう寝ます。
明日、というか今日、
友達と
『ミーティング・ザ・ビートルズ・イン・インド』
という映画に誘われれいるので・・・
なんかつまんなそうなんだけど 苦笑
0875ブタ耳 (ワッチョイW 4ee3-7V8K)
垢版 |
2022/10/10(月) 02:40:03.31ID:/uVa+ohB0
>>873
「一人で会話してる」とか言われたもんね。
もっと罵り合えばよかったのかね?

ああ、そう言えば思い出した。
「他の文献では」の先をまだ聞いてない。
けど、もうこのスレ残り少ないし、やめときましょうか。
0876名無しさん@英語勉強中 (ワッチョイW b39d-ZDf4)
垢版 |
2022/10/10(月) 06:02:56.91ID:7M/ciOIv0
確かに、関係形容詞whatだけがall っていうくだりは、はやく撤回した方がいいと思う
0877名無しさん@英語勉強中 (テテンテンテン MM86-rqSc)
垢版 |
2022/10/10(月) 08:43:55.95ID:y+s7oVCOM
>>833

>>後者は「ありったけの」という言葉と入れ替えられる文脈で使うのである。

「全部」って、普通「ありったけ」という言葉と入れ替え可能だろ 苦笑

Show me what you’ve got in your pockets.
「ポケットに入っているもの全部を見せなさい」
「ポケットに入っているありったけのものを見せなさい」

当時、人類の英知が獲得していた全部の知識
当時、人類の英知が獲得していたありったけの知識
0878名無しさん@英語勉強中 (テテンテンテン MM86-rqSc)
垢版 |
2022/10/10(月) 08:46:47.53ID:y+s7oVCOM
ありったけ

《「ありたけ」の音変化》

【一】[名]あるだけ全部。「—の声を張り上げる」「資金の—をつぎ込む」

【二】[副]可能な限り多く。できるだけ。思う存分。「力を—出す」
0879名無しさん@英語勉強中 (テテンテンテン MM86-rqSc)
垢版 |
2022/10/10(月) 09:34:11.74ID:y+s7oVCOM
>>874
ビートルズつながりで言うとさ
All We Need Is Love っていうタイトルの曲があるでしょ

で、他のアーティストが
What We Need Is Love といっていう曲出してんだよね

これ、ほぼ同じ意味でしょ



これ以上、死者に鞭打つのは止めておきますかw
0880名無しさん@英語勉強中 (マグーロ MM86-rqSc)
垢版 |
2022/10/10(月) 10:05:56.13ID:y+s7oVCOM1010
×といっていう曲出してんだよね
〇っていう曲出してんだよね

今後はよく推敲してから投稿します
0881名無しさん@英語勉強中 (マグーロ 4a33-Phf8)
垢版 |
2022/10/10(月) 10:16:39.07ID:fhABQbfK01010
英語の数学の本を読んでいて質問があります。

代数においては通常「10^x」が有理数 x のみに対して定義されます。
これから「10^x」を実数に対しても定義する前のところで以下の文章があります。

In algebra, 10^x is usually defined only for rational x, while the definition for irrational x
is quietly ignored. A brief review of the definition for rational x will not only explain this
omission, but also recall an important principle behind the definition of 10^x.

「not only A but also B」が出てきますが、

「S does not only V1, but also V2」

の意味は、「SはV1するだけでなくV2もする」という意味でOKですか?
「SはV1しないだけでなくV2もする」ではないですよね?
0882ブタ耳 (マグーロW 4ee3-7V8K)
垢版 |
2022/10/10(月) 10:18:38.86ID:/uVa+ohB01010
ソフィスト氏には、>>772で引用したCOBUILDの例文を和訳して提示してもらいたい。無理にとは言わない。無理だろうから。
0883名無しさん@英語勉強中 (マグーロ 4a33-Phf8)
垢版 |
2022/10/10(月) 10:20:09.56ID:fhABQbfK01010
「not only」の「not」は単に「only」を否定しているだけということですよね?
0885名無しさん@英語勉強中 (マグーロ 4a33-Phf8)
垢版 |
2022/10/10(月) 10:23:42.32ID:fhABQbfK01010
>>884
ありがとうございました。
0886名無しさん@英語勉強中 (マグーロ 8a5d-Rol6)
垢版 |
2022/10/10(月) 11:12:42.57ID:Xxbp2Sau01010
I am from Japan.
はSVCかSVどちらでしょうか。SVの場合、from JapanはIを修飾しますか?
0887ブタ耳 (マグーロ 4ee3-3fXN)
垢版 |
2022/10/10(月) 11:25:15.15ID:/uVa+ohB01010
>>886
それ様々な捉え方ができると思うんだけど、とりあえず江川『英文法解説』 (p.133) から一つの考え方を引用しとくね。
その考え方にしたがうとSVCということになります。

*****
学習英文法が8品詞5文型を続ける限り、「副詞は補語になれるのか」という疑問が常に生じるであろう。
答えは簡単に Ye sでよい。同じことは次のような句 (phrase )にも言える。
I'm from Ohio. (=I came from Ohio) (オハイオ州出身です)
This parcel is (meant) for you. (この包みはあなたにあげるものです)
*****
0888ブタ耳 (マグーロ 4ee3-3fXN)
垢版 |
2022/10/10(月) 11:33:36.03ID:/uVa+ohB01010
SOED (Shorter Oxford English Dictionary)上下2巻が机の前の本棚に鎮座しているのを思い出した。
これ昔、Le Guin が愛用してる辞書だというので買ったんだけど、以来ほとんど活用できてなかった。
せっかくだから what を調べてみることに。

まず関係代名詞の項。
A (pron) II. rel.pron.
7. That or those which, the things which;
as much or as many as;
the kind or sort that;
gen.(=generally) something which;

こんな感じで他の辞書だったら別々に項目立てするだろうような定義がひとまとめに提示されてる。
さすが shorter。

で、この as much or as many as に対応するであろうと思われる例文を抜粋してあげておく。
ついでに言っておくと、誰が書いた文なのかが常に明示されてるのがOED, SOEDの特徴。

D.Barnes. What she steals she keeps.
O.Manning. Harriet returned to hear what he had to say.

和訳は今は試みないでおく。
後者の文は what S have to do のパターンで、件の英文と重なるのでとりわけ興味深い。

8. Anything that, whatever,
Formerly also, whoever, Now only in phrs.

S.Pepys. Their service was six biscuits a-piece, and what they pleased of burnt claret.
0889ブタ耳 (マグーロ 4ee3-3fXN)
垢版 |
2022/10/10(月) 11:35:45.81ID:/uVa+ohB01010
つづいて関係形容詞の項。
E.(adj) II.rel.adj.
4. The ... that, as much ... as, as many ... as.

B.T.Washington. What little clothing and household goods we had were placed in a cart.
W.Golding. I exerted what strength I had left.

いずれも解釈しやすい例文だと思うが、ここでは定義の仕方に注目したい。

SOEDの関係形容詞の定義、 as much ... as, as many ...as という定義は、
先の関係代名詞の定義、as much as, as many as に対応している。

>>529 及び>>764-765で引用した安藤の解説中にある「この意味では」という文言は、まさに上記の対応関係のことをさすものだと考えられる。
0890ブタ耳aka高卒3号 (マグーロ 4ee3-3fXN)
垢版 |
2022/10/10(月) 11:44:02.07ID:/uVa+ohB01010
>「>>529 及び>>764-765で引用した安藤の解説中」

このように書くと、両者を同一人物が引用したかのような印象を与えることに気づいた。
私とテテン氏が別人であることは言うまでもない。
一部の人間に言っておきたいが、私が高卒3号、テテン氏が高卒2号である。
しっかり区別するように。

それからこういうバカ(>>333)のために、akaというのは「別名」という意味であって、bakaのタイポではないことも付記しておく。
0891😉三年英太郎🌈 ◆3CZBjOt3.Y (マグーロW ca86-zOhc)
垢版 |
2022/10/10(月) 13:05:22.06ID:TIMlWRIS01010
1820年代に入手可能な医学知識のすべて、ならわかるが、ここはmystery

高橋巌 大先生の神秘学入門でも読みなさい。いまやどんな知識本で読めてしまうようだけど、神秘っつーのはそうもんじゃないって冒頭から書いてあったはず。師匠から弟子への口伝伝承、あるいは神秘体験を経てのみわかることもある(とゆーか神秘主義の中心は後者が多い)。mystery 語源は「(耳・目を)閉ざす」なのだ

なおこれは一文の範囲を越えた語意の判断。3行前と先の内容で意味が確定するなら、それは文法問題ではないので⭐スレ⭐チ⭐

ついでに言えば、論文と違って多様に解釈して良いのが文学。そこが強みなのに、ここはこう読めという押しつけは文学を逆に殺す
0892ブタ耳 (マグーロ 4ee3-3fXN)
垢版 |
2022/10/10(月) 13:14:06.71ID:/uVa+ohB01010
>>891
ここでの「the mystery of human mind とは」は、妻から失われてしまった「心」に関わるもので、神秘学のようなものとは一切関係ない。
老人は当時の乏しい医学的知識に基づいて脳と神経システムの研究に没頭したのであって、高橋巌大先生の「神秘学入門」を読んだところでこの一節の理解に資するところはおそらく何もない。
0896ブタ耳 (マグーロ 4ee3-3fXN)
垢版 |
2022/10/10(月) 13:33:58.10ID:/uVa+ohB01010
個人的には結論はもう出ている。
「権威ある文献」と「英英辞書」が大好きなソフィスト先生の趣味におつきあいしているだけだよ。
徹底的につきあってあげるつもりでいる。
0900ブタ耳 (マグーロ 4ee3-3fXN)
垢版 |
2022/10/10(月) 17:00:53.09ID:/uVa+ohB01010
SOED の what の記述をたどっていて、一つ興味深い事実を発見したので報告しておく。

SOEDは、語義の配列として歴史的に古いものを先に記述していく方針で編集されているはず。
で、関係代名詞の項で真っ先に出てくるのが次のような意味。

II re.pron
6. Referring to a preceding pron. (esp. all) or n.: which, who; that.
Formerly also w.(=with) preceding prep.
Now non-standard.

whichやwhoと同様に、what にも先行詞をとる用法が以前はあって、とりわけ代名詞の all を先行詞にとることが多かった、ということ。

all を先行詞とする例文としては次の文が出ている。
J.B.Morton. If I sat down to write a book, I'd want to shove in all what I saw.

現在の what が持つ [all that] に近いニュアンスは、もしかするとこの古い用法に源流があるのではないだろうか?
素人の思いつきですが。
0902ブタ耳 (マグーロ 4ee3-3fXN)
垢版 |
2022/10/10(月) 17:27:55.42ID:/uVa+ohB01010
>>770で引用したOED 2nd edition には、1646年から1789年の用例しか出ていなくて、ソフィスト氏には
「用例が古いのなら今回の英文には当てはまらない可能性が濃厚。はい、ダメダメの反論。」(>>828)
と言われてしまった。ごもっともである。

では、OED 3rd edition に出ている2014年の用例ならどうだろうか。
https://www.oed.com/viewdictionaryentry/Entry/228078;jsessionid=5D33EDC11C0933FFEC1D3007BBB49BFD?print

III. As a relative pronoun
11. With a specific sense implied.
b. Expressing quantity or amount: so much (or many) as, as much as. Cf. sense A. 4.

2014 New Yorker 6 Jan. 44/1 The U.S. military monitors what it can of the hundreds of tons of cocaine that enter the U.S.
0904名無しさん@英語勉強中 (マグーロW b39d-ZDf4)
垢版 |
2022/10/10(月) 17:43:58.81ID:7M/ciOIv01010
検索すると現代のでも用例そのままじゃなくて
削ってるんだね
知らなかったよ

原文はこれだ

The U.S. military monitors what it can of the hundreds of tons of cocaine that enter the U.S. by plane, boat, automobile, submarine, tunnel, backpack, and catapult.
0905名無しさん@英語勉強中 (マグーロ 4ee3-3fXN)
垢版 |
2022/10/10(月) 17:59:45.29ID:/uVa+ohB01010
>>904
おお、ありがとうございます。

余計なことかもしれませんが、この例文の意味がつかみにくい人のために >>501で私があげた例文をヒントとしてあげておきます。

Don't worry. I'll do what (=anyting that) I can for you.
0906名無しさん@英語勉強中 (マグーロ 83b9-tCk3)
垢版 |
2022/10/10(月) 18:10:50.84ID:suQqSvR+01010
>>811
博士と狂人の原書を昔読んだ。
OEDは、項目を書いてくれる人を一般募集
して、書いてもいいという人を選んで、
その人に必要な本を送って、書いてもらった。
上の本の「狂人」という人は狂人かどうか
知らないけど、逮捕されて刑務所にいながら
辞書の項目を書く作業をしていた。
OEDは一般の人が項目を書いた大衆による
大辞典。偉大かどうかは、大衆の作ったと
いうことを偉大と考えるかどうかだと思うんだ。
なんかベートーベンの曲みたいな感じだな。
(一人が作ったという意味ではなく、大衆と
いうものによりできた。(大衆がお金をベートーベンに
お金出したとか、辞書の項目を書いたとか、大衆に
よりできたという意味で))

OEDよりも偉大というと、一人で作った
ジョンソン博士の辞書ということかも
しれない。

とOEDもジョンソン博士の辞書も読んだ
ことがない自分が書いてみた
0907名無しさん@英語勉強中 (マグーロ 83b9-tCk3)
垢版 |
2022/10/10(月) 18:21:18.70ID:suQqSvR+01010
the mystery of human mind
の部分は
神秘な人間の心、
ということでしょう。つまり平たくいうと心理学
(心理学が確立されていたか知らんけど)

だから
To his library in Cable House he will add volumes in which are contained what human knowledge,
in the 1820s, has to offer on the mystery of the human mind.
は、不思議な人間心理に関わる1820年代の知識をちょっとでも含んでいる本ならなんでも
自分の文庫に彼は付け加えようとする
という意味だと思う。
0912ブタ耳 (マグーロ 4ee3-3fXN)
垢版 |
2022/10/10(月) 21:35:26.42ID:/uVa+ohB01010
Win7上のJaneStyleで、表示されない絵文字をコピペするとどうなるかちょっと実験。

ス⭐レ⭐チ

聞くたびに畏怖の念に打たれる楽曲が3つある。その内1つがこれ。
聴衆の受けが一番ひどかった曲もたぶんこれ。
https://youtu.be/hXvP0bqw2Cs?t=2140

ヴィオラが女性奏者に交代する前のEbene。
交代後に出た全集は超お勧め。個人的ベストだったスメタナ盤をはるかに超えてきた。

コピペ後に表示されている記号を文字で打って絵文字を再現できるか実験。

ス⭐マ⭐ソ
0914😉三年英太郎🌈 ◆3CZBjOt3.Y (ワッチョイW ca86-zOhc)
垢版 |
2022/10/10(月) 22:08:55.55ID:TIMlWRIS0
op.130番代はヘンテコだらけやろ🤔
第九も大衆的な人気を勝ち得ているが、相当のヘンテコ曲

人生の最後は自身が実験場にしてたピアソナタで締めくくってほしかったが弦ばっかというね🥺
0915エキセントリックなソフィスト ◆BWpafMA3P8me (ワッチョイ 6f96-kHT+)
垢版 |
2022/10/10(月) 23:10:42.87ID:BnmQocJa0
>>854
君は頭が悪いんだから、私の言うハイレベルな意見は理解できないんだよ。だから、だまっていたまえ。
>>855
なぜ英英辞典がかたくなに関係代名詞whatがall等しいと言わないのか理解できないだんだろ。なら、黙ってろ。
>>857
プロの翻訳家がミスするのは、よくあること。だから、翻訳チェッカーという仕事がある。翻訳チェッカーは、翻訳者よりも英語を知らないけどね。
翻訳者が全員英文法が完璧だと思ってるなんて、おめでたい人だね。
>>862
文章変えたらだめ。あくまでも、あの文章の解釈なのだから。
0916エキセントリックなソフィスト ◆BWpafMA3P8me (ワッチョイ 6f96-kHT+)
垢版 |
2022/10/10(月) 23:10:54.11ID:BnmQocJa0
>>863
出版物は、校正が入ってるから、基本的に間違いはない。
>>869
君は、本当に読解力がない。『areで受けてることが「全部」の意味でない証拠』とは、私は言っていない。これはこれまでの書き込みを見たらわかる。
なぜなら、「全部」と言うと、allが含まれるからだ。allは複数扱いする単語だから、私は「全部」という日本語はあえて避けてきた。そこが読み取れていない。
もっと国語力つけたほうがいいね。
>>870
「all the」の意味だと限定しているだろ。ほんと読解力がないな。
0917エキセントリックなソフィスト ◆BWpafMA3P8me (ワッチョイ 6f96-kHT+)
垢版 |
2022/10/10(月) 23:11:43.73ID:BnmQocJa0
□□□□□□□
まとめ(修正版)
□□□□□□□
英英辞典では、関係代名詞whatを「all」と等しいものだと定義しているものはない。(日本人の書いたものを絶対視してはいけない。英英辞典を使う必要がある)
https://www.merriam-webster.com/dictionary/what
https://www.collinsdictionary.com/dictionary/english/what
これらのページにあるように、関係代名詞のwhatは、基本的に「the thing or things that」と「anything or everything that」(whatever)、「~するだけの」(as much/many as)の意味なのである。
この中で「are」で受けることができるのは、「things that」と「as many as」だけだが、後者は「ありったけの」という言葉と入れ替えられる文脈で使うのだ。

そもそも、>>522の"Give me as much as there is."なんて英文、ネイティブが普通に使ってる表現ではない。 この部分は、"give as much/many as we can"は、"give what we can"と書いても同じ意味だよと説明したほうがはるかに分かりやすく現実的である。

※※※※※※※※※※※※※※※※※※
whatを「the whole of an amount」で書き換えられるケースもあるが、ジーニアス英和辞典によれば、「the whole of ~」は~の部分に複数名詞は来れず、単数扱いなので、これも「are」で受けることができないので、今回適用することはできない。
※※※※※※※※※※※※※※※※※※

以上より、問題となっているwhatは、「things that」で解釈するしかないのである。
0918エキセントリックなソフィスト ◆BWpafMA3P8me (ワッチョイ 6f96-kHT+)
垢版 |
2022/10/10(月) 23:12:07.96ID:BnmQocJa0
□□□□□□□
まとめ2(修正版)
□□□□□□□
本当は文法を論じるまでもなく、常識で分からないといけない。どっちの訳が常識的?
What human knowledge has to offer on 人間の心の神秘 are contained in the set of books (=in the volumes).

A:その一群の本には、人知が人間の心の神秘に関して寄与している「すべて」が含まれている。
B:その一群の本には、人知が人間の心の神秘に関して寄与している「いろいろなもの」が含まれている。(things thatで解釈した場合の訳)

どっちが、ウソっぽい? 「人知が人間の心の神秘に関して寄与しているすべてが漏れなくこれらの本に載ってるよ。さあ、買った買った」って、うさんくさいことこの上ない、ペテン師の台詞じゃないか。 何個までが「すべて」かなんて誰にも判定できないのだから、安易に「すべて」と言ってはいけない。言うととても浅はかに聞こえる。文豪がそんな安っぽい表現を使うわけがない。

本当は、文法を考えるまでもなく、文脈からBが適切だとわからないといけないのだ。
文脈でわからない人たちがいるので、仕方なく文法で攻めたけど、これは本来は国語力というか文章読解力というか、そういう次元の問題なのである。だから、ある一定以上の知能がないと、私の説明が理解できないのである。

ここまでの私の説明を読んでもまだ理解できない人たちは、その程度の人たちなのだから、いくら私が説明しても永遠に正解を理解することはできないだろう。よって、この問題に関するコメントは今回で終わりにしたい。以後、一切この問題に関してレスしないのでご了承を。反論しないからその反論を受け入れたのだと思わないようにね (^-^)
0922ブタ耳 (ワッチョイ 4ee3-3fXN)
垢版 |
2022/10/10(月) 23:32:39.17ID:/uVa+ohB0
>>920
ついでにこれも訳してみてください。
The U.S. military monitors what it can of the hundreds of tons of cocaine that enter the U.S.
0923ブタ耳 (ワッチョイ 4ee3-3fXN)
垢版 |
2022/10/10(月) 23:35:58.16ID:/uVa+ohB0
>>920
あと、これの和訳もお願いします。
Show me what you’ve got in your pockets.

>>915 「文章変えたらだめ。あくまでも、あの文章の解釈なのだから。」

これ意味不明なんですけど、あなたの訳を見せてもらえば分かるかも。
0925ブタ耳 (ワッチョイ 4ee3-3fXN)
垢版 |
2022/10/10(月) 23:48:44.35ID:/uVa+ohB0
>>918
それ、私の>>871の書き込みを読んで多少反省するところがあったせいか、文脈という言葉を盛り込んでいますけど、
★文脈という言葉を盛り込んだだけ★で基本何も変わってませんよね。
相変わらず文脈無視の比較を続けている。
「文脈」という言葉の意味さえ分かっていなかったんですか?

かなり悲惨な状況です。もうダメですね。
0927名無しさん@英語勉強中 (ワッチョイ 4ee3-3fXN)
垢版 |
2022/10/11(火) 00:18:57.04ID:1xSFtJiF0
あ、「文章変えたらだめ。あくまでも、あの文章の解釈なのだから(>>915)」の意味が分かったかもしれない。

「他の文について考えてはいけない。あの小説からの引用文の解釈だけを考えるべきだ」

という意味なのかな?

そりゃ~ないだろ~
https://uploader.cc/s/kh5tliwq7w8vi1fv4ibc5hfl6bku4zyx3qrpyzh2unjpcqplst4knrpsdjm9h82s.mp3
0928ブタ耳 (ワッチョイ 4ee3-3fXN)
垢版 |
2022/10/11(火) 00:53:34.07ID:1xSFtJiF0
この what の件は、最後に建設的な考察でしめくくっていきたい。

今回話題になったwhatの用法、
anything that, とか as much/many as 系の、「すべて」とか「ぜんぶ」と訳せる場合がある用法。

これを、誰か他の人(たとえばかなりよくできる高校生など)に説明しようとしたとき、いったいどうやったら他の用法との見分け方を分かりやす教えられるだろうか?
分かりやすく説明できる自信がまったくない。
実例を数多く提示する? これもあんまり有効ではなさそうだ(このスレが実証している)

そもそも他人事ではない。
今後、読んでいる英文中にこの what が出てきたとき、それを普通の用法の what とは違うものとして即座に感知できるようにするにはどうしたらいいのだろうか?

とりあえず一つ考えているのは、OEDの定義(>>902)にあった "Expressing quantity or amount".
「数量が問題になっている場合には特に注意する」ということ。

あとは can とかが後に続く場合? 
have など、この用法で頻出する特定の動詞がある?

このスレで延々と関わってきたおかげで、what感覚みたいなものがかなり身についてきたように感じてはいるのだが、先は長いというか…
0929名無しさん@英語勉強中 (テテンテンテン MM86-rqSc)
垢版 |
2022/10/11(火) 01:18:08.45ID:pCKHgaY8M
>>924
Show me what you’ve got in your pockets.

を訳してみてというのが
どうして論点ずらしになるんだよwアホ

whatが「全部」の意味として使われている例があるかないかは
最重要論点だろうが。

お前は無いと終始主張しているが
「全部」の意味としか解せない英文が存在してると
こっちは主張してんだよ。

これが「全部」という意味でないなら
どう訳すのかと聞いている。

逃げるんじゃねーよ、卑怯者。
0930名無しさん@英語勉強中 (テテンテンテン MM86-rqSc)
垢版 |
2022/10/11(火) 01:29:43.95ID:pCKHgaY8M
>>916
>>「all the」の意味だと限定しているだろ。ほんと読解力がないな。

「関係形容詞のwhatのみが「all」で解釈できるのだ。これ重要。」
の部分を訂正しろと言ってんだよ、馬鹿。
お前が読解力がねーんだろ。
0931名無しさん@英語勉強中 (テテンテンテン MM86-rqSc)
垢版 |
2022/10/11(火) 01:33:14.44ID:pCKHgaY8M
テーマはこのwhatに「全部」という意味があるのか
という話なのに
結局こいつは
allと入れ替えたら、anythinngと入れ替えたら・・・・
文法的にどうのこうの・・・・
とグダグダ言い続けている。

何度も言うが
テーマはこのwhatに「全部」という意味があるのかどうか
他の英単語だとこれに意味が近いという意味で、別の単語をだしていることくらい
アホでもわかると思うのだけどな。
0932名無しさん@英語勉強中 (テテンテンテン MM86-rqSc)
垢版 |
2022/10/11(火) 01:47:13.22ID:pCKHgaY8M
つかさぁ
「ケイブル邸の彼の蔵書に新たに加わる一群の書物には、1820年代の人類が、人間精神の神秘についてすでに得ている知識の全部が詰まっている。」
っていう小説の一文を見て

え?全部が詰まってるだって?
そんなことあり得ないだろ!!!ペテン師め!!!!
って、逆上する基地外なんて本当にいるんだろうか?wwww
一部、アスペルガー的要素のある人はそうなのかもしれんけどwwwwww

でも
ソフィスト大先生によると

>>これは本来は国語力というか文章読解力というか、
>>そういう次元の問題なのである。
>>だから、ある一定以上の知能がないと、私の説明が理解できないのである。

ということのようです。 苦笑
0933名無しさん@英語勉強中 (テテンテンテン MM86-rqSc)
垢版 |
2022/10/11(火) 01:54:23.75ID:pCKHgaY8M
>今回のキーポイント
>What (主語) are contained in 〜.が今回の英文なので、Whatは、複数扱いする言葉と交換できなくてはならない。

こいつは最後まで、他の言葉との交換の話ばっかり。
当該whatは「全部」という意味かどうかというテーマに対して
全く本質的な話になっていない。

↓これもそう。
>※※※※※※※※※※※※※※※※※※
>whatを「the whole of an amount」で書き換えられるケースもあるが、ジーニアス英和辞典によれば、「the whole of 〜」は〜の部分に複数名詞は来れず、単数扱いなので、これも「are」で受けることができないので、今回適用することはできない。
>※※※※※※※※※※※※※※※※※※

馬鹿なのかな?
0934名無しさん@英語勉強中 (テテンテンテン MM86-rqSc)
垢版 |
2022/10/11(火) 02:10:01.66ID:pCKHgaY8M
文法を離れた読解力の話をするなら

ソフィスト大先生のようなアスペ目線で解釈してはダメですね
普通の人は
「ケイブル邸の彼の蔵書に新たに加わる一群の書物には、1820年代の人類が、人間精神の神秘についてすでに得ている知識の全部が詰まっている。」
という一文に何の違和感も抱かないでしょう。

むしろ
単に「心の神秘について書かれた本を集める」という情報を伝えるだけなら
he will add volumes about the mystery of the human mind.
くらいの英文で済むはずなのに
he will add volumes in which are contained
what human knowledge, in the 1820s, has to offer on the mystery of the human mind.
なんていう仰々しい言い回しをしているのは何故なのかを読み取らなくてはなりません。

ある一定以上の知能がないと、無理かもしれませんが。
0935名無しさん@英語勉強中 (テテンテンテン MM86-rqSc)
垢版 |
2022/10/11(火) 02:24:14.44ID:pCKHgaY8M
ソフィスト大先生は
まず
1)Show me what you’ve got in your pockets.
2)What we need is love.
のwhatを「全部」と解釈して良いか悪いかについて答えなさい

まぁ逃げるんでしょうけどね。

でも、逃げるのなら、もう今後一切口出しして来るなよ。
つか、テメーでもうコメントしないと宣言してるのだから
それでいいけどね。負け犬。
0936名無しさん@英語勉強中 (US 0Hff-rqSc)
垢版 |
2022/10/11(火) 06:34:54.48ID:ggUkr2xcH
>>899
亀レスですいませんが、返信ありがとうございました

英語やっていると、ネイティブの人は本当にこんな細かなルール意識せずに運用できているのかと不思議に思われてくる
生成文法の分野とかでこのあたりの議論はなされているのかな
0938名無しさん@英語勉強中 (ワッチョイ de89-iygP)
垢版 |
2022/10/11(火) 12:21:06.60ID:i+8+UKXl0
>>915に関連して言うと、エキセントリックなソフィスト氏は以前どこかのスレでも
自分がプロの翻訳家であると言っていた。
これはまず間違いなく嘘でしょうね。

ちなみに、プロの翻訳界は2種類あって、文芸翻訳(出版翻訳とも)と産業翻訳
(ビジネス翻訳、実務翻訳とも)に分けられる。そして、「翻訳家」なる呼称は文芸

翻訳をする人々が使うもの。産業翻訳をする人は自分のことを普通「翻訳者」としか
名乗らない(まともな言語感覚を持っていれば)。

ところが、エキセントリックなソフィスト氏は>>915
「プロの翻訳家がミスするのは、よくあること。だから、翻訳チェッカーという仕事がある。翻訳チェッカーは、翻訳者よりも英語を知らないけどね。」
と言っている。

しかし、「翻訳家」すなわち「文芸翻訳をする人」は「翻訳チェッカー」など使わない
のである。「翻訳チェッカー」を使うのは「産業翻訳」の世界である。
(そもそも、今の日本でプロの「文芸翻訳家」は30人もいるかどうかわからないほど
小さい世界である)

エキセントリックなソフィスト氏はその違いがわかってない。
本当に2、3年程度でも翻訳の仕事(文芸翻訳であろうと産業翻訳であろうと)をして
いたなら、「翻訳家」と「翻訳者」の使い分けを知っているはずである。実際に少し
でも継続して仕事をしていれば間違えるはずがない。
「翻訳家」=「文芸翻訳家」が「翻訳チェッカー」を使うなど大笑いである。
0939ブタ耳 (ワッチョイ 4ee3-3fXN)
垢版 |
2022/10/11(火) 13:43:33.81ID:1xSFtJiF0
COBUILDの例文(>>772)を訳してみる

He drinks what is left in his glass as if it were water.

全部のニュアンスを盛り込まない直訳:
・彼はグラスに残されたものを、あたかもそれが水であるかのように飲む。

「全部」を盛り込む:
・彼はグラスに残されたものを全部、あたかもそれが水であるかのように飲む。

「全部」のニュアンスを「飲み干す」に移す:
・彼はグラスに残されたものを、あたかもそれが水であるかのように飲み干す。

一つの解釈。(これはやりすぎかも)
・グラスに少しでも酒が残っていると、彼はそれをまるで水でも飲むようにきっちり最後まで飲み干してしまう。
0940ブタ耳 (ワッチョイ 4ee3-3fXN)
垢版 |
2022/10/11(火) 13:56:35.61ID:1xSFtJiF0
OED 3rdの例文も訳してみる

The U.S. military monitors what it can of the hundreds of tons of cocaine that enter the U.S.
(can の後には do=monitorが省略されている)

「全部」のニュアンスを盛り込まない直訳
・米軍は、 米国に入ってくる何百トンものコカインに関して、それ(=米軍)が監視できるものを監視している。

江川の例文(>>501)に即して訳してみる
・米軍は、米国に入ってくる何百トンものコカインに関して、それ(=米軍)が監視できるかぎりものを監視している。

意訳
・米軍は、米国に流入する何百トンものコカインに対して、可能なかぎり万全の監視体制を敷いている。
0941ブタ耳 (ワッチョイ 4ee3-3fXN)
垢版 |
2022/10/11(火) 14:01:33.26ID:1xSFtJiF0
Show me what you’ve got in your pockets.
この文にについては辞書(Wisdom >>762 >>807)の訳を再録するに留める。

・ポケットに入っているものをみんな見せなさい

本日の書き込みは以上です。

では、(^^)/~~~ 
0942名無しさん@英語勉強中 (ワッチョイ de8f-Rol6)
垢版 |
2022/10/11(火) 14:03:00.76ID:uqeHRxNK0
Only by bus can you go there.

by buyがonlyを修飾しているのか、Onlyがby busを修飾しているのかどちらでしょうか。
0943名無しさん@英語勉強中 (ワッチョイ de8f-Rol6)
垢版 |
2022/10/11(火) 14:09:17.90ID:uqeHRxNK0
分裂分(強調構文)の
It is the theater that she went to.
ですが、これは文型としてはSVCなんでしょうか。それとも強調したものを後の文から引っ張ってきたので
SV(強調したいこと)という文型(SV文型)になりますか?
また、後のthatはthat節と考えるのが適当でしょうか?(強調したい事柄によってSやOがない不完全文になることもあると思いますが)
0944名無しさん@英語勉強中 (テテンテンテン MM86-rqSc)
垢版 |
2022/10/11(火) 22:57:08.43ID:sxHJigNqM
>>943
5文型偏愛者の私も流石に
強調構文の形式を5文型に分類する意味はない
と思いますけど
何か分類する必要性があるのでしょうか?

例えば準補語などを伴う場合なども
5文型分類が想定していませんし。
これも同様に、想定されていないケースに当たるのではないですか?
0945名無しさん@英語勉強中 (テテンテンテン MM86-rqSc)
垢版 |
2022/10/11(火) 23:28:13.34ID:sxHJigNqM
>>後のthatはthat節と考えるのが適当でしょうか?(強調したい事柄によってSやOがない不完全文になることもあると思いますが)

that節と呼んで問題ないと思います。
主語がないケースについても、関係代名詞が主語である場合の関係詞節と同様に考えればよいのではないでしょうか?

thatの品詞については
>>JespersenのMEG*1に見られる強調構文の分析の変遷が、大塚・中島(1982)にまとめられている。
>>Jespersenは当初、It is the wife that decides.という文で、実質的な意味がThe wife is the deciding person.
>>であることを理由に関係詞節のthat decidesをthe wifeよりもItにかかるものであると分析した(MEG, Vol.3)。
>>しかし後になってJespersenは、関係詞節中の動詞の数の一致が説明できないことと、it isの後に副詞句が生じる場合があることを踏まえて、
>>it is ... thatを特殊な外置構文、つまり形式主語構文の特殊なものという分析に改めている(MEG, Vol.7)。
>>強調構文にはこのように形式と意味のねじれが生じているために、thatの品詞について深入りしない参考書が現在の主流である。
0946名無しさん@英語勉強中 (テテンテンテン MM86-rqSc)
垢版 |
2022/10/11(火) 23:30:15.69ID:sxHJigNqM
thatの品詞について書かれた記事のリンクを貼ろうとしたのですが
NGワードとなってしまいました。
https⇒ttps とかに変更してもダメでした。
0947名無しさん@英語勉強中 (テテンテンテン MM86-rqSc)
垢版 |
2022/10/11(火) 23:36:03.39ID:sxHJigNqM
>>943
>>これは文型としてはSVCなんでしょうか。
>>それとも強調したものを後の文から引っ張ってきたので
>>SV(強調したいこと)という文型(SV文型)になりますか?

ん?
SVCと考える場合は、Cは何ですか?
SVと考える場合は、何がSで何がVなんでしょう?
0948名無しさん@英語勉強中 (テテンテンテン MM86-rqSc)
垢版 |
2022/10/11(火) 23:43:05.77ID:sxHJigNqM
多分
SVというのは
She went to the theater.
という文についての文型で(7文型ではSVA)

SVCというのは
S→It
V→is
C→the theater

ということですよね?
0950名無しさん@英語勉強中 (US 0Hff-rqSc)
垢版 |
2022/10/12(水) 10:42:40.47ID:cy1eZ2cpH
今までvintageで英文法を学んできたのですが、最近になって文章内で出てくる文法がvintageではカバーしきれていないことが多くなってきており物足りなさを感じています、これ以上の知識をつけたい場合、どのような教材又は手段がありますか? 
現在は逐一検索していますが、出来るだけ体系的に学びたいと考えています
0951名無しさん@英語勉強中 (ワッチョイW b39d-ZDf4)
垢版 |
2022/10/12(水) 10:48:11.85ID:qxj8yrVq0
そうか
カバーしきれない文を貼ってみてくれるかな
0952名無しさん@英語勉強中 (US 0Hff-rqSc)
垢版 |
2022/10/12(水) 11:12:36.06ID:cy1eZ2cpH
例えば、 Much as they sympathize with students ~, the faculty are not ~. のような文章です

vintageでは 形容詞 + much as SV~ という形で 「Sは...であるけれども」という意味を取ることができると書いてありますが、形容詞のみに限定されおり、上記の文章は解釈できませんでした

他にも、vintageでは省略できる部分と省略できない部分が明記されていない所も不満ですね
0954名無しさん@英語勉強中 (オッペケ Sr03-ZDf4)
垢版 |
2022/10/12(水) 11:16:03.15ID:f5RyqxLcr
細かい語法はまずはジーニアスかウィズダムを引くことを考えた方がいいと思うよ
その上でじゃあどうするかっていう話になる
0955名無しさん@英語勉強中 (US 0Hff-rqSc)
垢版 |
2022/10/12(水) 11:27:16.43ID:cy1eZ2cpH
ありがとうございます、なるほど能動的な学習が必要になってくるということですね
私の場合あまり英語学習に時間を割けないので、上記の方法だと間違えたときに初めて能動的な学習が可能になるのでどうしても後手に回ってしまいかねないんですよね...
0956ブタ耳 (ワッチョイ 4ee3-3fXN)
垢版 |
2022/10/12(水) 11:52:59.41ID:x5llJ+n50
>>952
江川の『英文法解説』、お勧め。

ただし、体系的に書かれている本ではあるけれども、この本に沿って体系的に勉強しようとすると挫折するかもしれない。
私は、折にふれて疑問点を調べ、その都度芋づる式に関連事項を調べていく、みたいな付き合い方をしている。
振り返ってみると、この本を最初から最後まで通読したことは一度もない。

それでも、確か翻訳家の柴田元幸さんだったと思うが、「この本の例文をすべて自分で翻訳し直す」という訓練(?)を実行した人もいる。
たしかにそれをやったら相当な勉強になるだろう。

問題の as の用法に関して引用しておく。
*****
as (=though) 譲歩の意味を強調する文語的な構文で, (程度を示せる)形容詞・副詞・それに動詞が先行する)。
~略~
Much as we resemble each other, we are none of us exactly alike.
(人間は互いによく似てはいるが, まったく同じという人はいない)
***** 江川, 『英文法解説』, pp.392-393

Vintage英文法については参照したことがないのでノーコメント。
0958名無しさん@英語勉強中 (US 0Hff-rqSc)
垢版 |
2022/10/12(水) 12:13:33.90ID:cy1eZ2cpH
紹介ありがとうございます、折を見て軽く読んでみようと思います
英語ではないのですが、一定以上の専門性を持つ本を読むときはどんな内容が書かれているのかあらかじめ確認した上で必要に応じて辞書的な使い方をする、というようなことが多いので上記の勉強法には共感できるところが多かったです
英語の上達は如何に英語に触れられる環境を意識的に作っていけるかにかかっているのかもしれませんね
0959名無しさん@英語勉強中 (US 0Hff-rqSc)
垢版 |
2022/10/12(水) 12:20:26.29ID:cy1eZ2cpH
>>958
ありがとうございます、個人的に哲学の発展・受容史における言語間の影響について関心があるんですよね
友人から『言語研究の世界』という本をお勧めされたので、こちらも参照しつつ読んでみたいですね
0960ブタ耳 (ワッチョイ 4ee3-3fXN)
垢版 |
2022/10/12(水) 14:22:44.76ID:x5llJ+n50
>>956
訂正。柴田元幸ではなく安西徹雄だった。

「江川泰一朗教授の『英文法解説』をテキストに、英文法にかかわるあらゆる項目にわたって、おびただしく挙がっている例文を片端から訳してゆき、構文上、なんらかの基本的な変換の必要なケースを逐一洗い出して、翻訳上、どう処理すべきか検討してみたのだった」
安西徹夫『英語の発想』ちくま学芸文庫版あとがき

『翻訳英文法』(文庫版では『英文翻訳術』)執筆時のエピソードとしてそう語られている。

引用部分を改めて読み返してみると、必ずしも「例文をすべて自分で翻訳し直した」わけではなさそうだ。
『英文翻訳術』(ちくま学芸文庫)には江川本からの例文が多数とりあげられているが、すべての例文が収録されているわけではない。
それでも、「執筆前段階の準備として例文をすべて翻訳し直した上で、逐一洗い出した結果を本にまとめた」、と読めないこともない。

日本語の解釈も難しいねー。

教訓:「すべて」という言葉はうかつに使うべきではない。
0961名無しさん@英語勉強中 (テテンテンテン MM86-rqSc)
垢版 |
2022/10/12(水) 17:58:00.27ID:O+SyDa8VM
『学校で教えてくれない英文法』 薬袋 善郎・著
↑とりあえずこの本をお勧めします。
文字通り、学校の英文法で取りこぼしている英文法を扱っていて、その構文についても詳しく載っています。

体系的とはいけないけど。
0962名無しさん@英語勉強中 (ワッチョイ 83b9-tCk3)
垢版 |
2022/10/12(水) 18:03:53.06ID:igL+EMdA0
>>956
副詞を倒置で前に持ってくるのか。
Poor as he was, he inherited vast amount of wealth.
みたいな使い方と同じ使い方だな。こちらは形容詞だけど。
alikeが副詞で、none of usが補語ですね。
0963名無しさん@英語勉強中 (テテンテンテン MM86-rqSc)
垢版 |
2022/10/12(水) 18:20:44.10ID:O+SyDa8VM
>>962
none of us は補語ではありません。
この文の主節は
None of us are exactly alike.
という文と同様な意味になります。

We are all are exactly alike.
という文のallと同じような位置づけにあり、
そして意味は反対になる感じかと。

参考
We shall none of us leave this island.(私たちの誰もこの島を出ることはないだろう)
0964名無しさん@英語勉強中 (テテンテンテン MM86-rqSc)
垢版 |
2022/10/12(水) 18:22:58.30ID:O+SyDa8VM
訂正
×We are all are exactly alike.
〇We are all exactly alike.
0965名無しさん@英語勉強中 (テテンテンテン MM86-rqSc)
垢版 |
2022/10/12(水) 19:20:12.84ID:Uysfb2FKM
『学校で教えてくれない英文法』には一つだけ、間違ったことが書かれています。

有名な米国のリンカーン大統領が1863年ゲチスバーグで行った演説
"government of the people,by the people,for the people"
の解釈についてです。

それ以外は問題ないと思います。
0966名無しさん@英語勉強中 (ワッチョイW b39d-ZDf4)
垢版 |
2022/10/12(水) 19:33:12.36ID:qxj8yrVq0
あああれね
東大で聞いたネタをそのまま書いたやつか
0967名無しさん@英語勉強中 (ワッチョイ de8f-Rol6)
垢版 |
2022/10/12(水) 20:24:41.47ID:mk99NLiq0
ご回答ありがとうございます。

>>944
>何か分類する必要性があるのでしょうか?

英文解釈の練習のためです。どれがSでVかなどを振る勉強をしています。

>>948
It is the theater that she went to.

S: It
V: is
強調したいこと: he theater

なのか

S: It
V: is
C: he theater

ということについて話していました。
0968名無しさん@英語勉強中 (ワッチョイ de8f-Rol6)
垢版 |
2022/10/12(水) 20:26:49.26ID:mk99NLiq0
he theaterはthe theaterの間違いでした。
準補語についてはわからないので(初めて聞いた)調べますね。
0969名無しさん@英語勉強中 (ワッチョイ 83b9-tCk3)
垢版 |
2022/10/12(水) 20:38:23.24ID:igL+EMdA0
>>960
そんな本だとは知らなかった。
買ってたけど全然読んでなかった。
ありがとう
0970ブタ耳 (ワッチョイ 4ee3-3fXN)
垢版 |
2022/10/13(木) 01:13:11.75ID:cmeNxSRc0
スレ残り少ないんだけど、>>772のCOUBUILDの例文の解釈、まだもやもやしてる。
・He drinks what is left in his glass as if it were water
・グラスに酒が残っていると、彼はそれをまるで水でも飲むように一口で飲んでしまう。

drinks → なんで現在形なの?
「普段の習慣」なんだろうと解釈して、語義の説明(the whole of an amount)を踏まえた上で、上記のように訳してはみたが、今一納得できない。
どういう状況これ? こんなことが習慣的に反復されるのは不自然だっ。

「普段の習慣」じゃなくて、シナリオのト書きみたいなものなのかな?
だとすれば、例えば、

彼は、グラスに残った酒を、まるで水でも飲むようにひょいっと一口で飲み干す。
「マスター、もう一杯頼む」
バーテンダーは客に背を向けたままつぶやく。
「もうそろそろおやめになった方が…」

.みたいな感じ?
0971名無しさん@英語勉強中 (ワッチョイ 4af0-SUfT)
垢版 |
2022/10/13(木) 07:33:43.29ID:fgZvb0I70
「what 〜 の what をどう訳すかの議論」、面白かったし参考になった。・・・、

このwhatにはいろいろな意味の可能性はあるが、結局は文脈が決める。そして
その文脈の解釈も読み手によって微妙な違いが出てくる場合があるし、さらに、
書き手の意図するところも当然関わってくる。そしてその場合、書き手によっては
表現力の不足で、必ずしもその意図、言いたいことがが十分に伝わらないままの
文章や表現で終わる場合もあるだろう。  

今回の課題の文章は素人の文章や一般の例文よりはこういう曖昧さは遥かに少ない
だろうが、それでも書き手の意図するところは、どうしても推察の域を出ない。

・・・ということで、この議論は”今のところ”「勝負なし」でいいのかな。
0972ブタ耳 (ワッチョイW 4ee3-7V8K)
垢版 |
2022/10/13(木) 09:11:18.58ID:cmeNxSRc0
>>971
その議論に関してこちらが主張したいことは既に十分語り尽くしたので、あとはご自分で判断してください。
今後の課題については>>928で触れました。
最後にモヤってるのが、COBUILDの例文訳(>>970)なので、できたらあなたの解釈をお聞かせください。
「一口で」という翻訳は、自分としては「かいしんのいちげき」なんですけど、辞書の例文をト書き的な現在形の文だと解釈するとか、ありえなくないですか?
0973名無しさん@英語勉強中 (ワッチョイW b39d-ZDf4)
垢版 |
2022/10/13(木) 09:27:40.54ID:LIC2lh280
▶ He drinks what is left in his glass as if it were water.
彼はグラスに残されたものをまるでそれが水であるかのように飲んだ.

この訳はよくないけど
酒と同定するのもないな

現在形の意味だが、内容からしてモダンな現在形での
描写と読むのが自然だと思う
辞書の例文としては高度だけどまあそうなんだろう
0974😉三年英太郎🌈 ◆3CZBjOt3.Y (ワッチョイW ca86-zOhc)
垢版 |
2022/10/13(木) 10:41:52.09ID:zGrS5IOE0
>>971
受容美学という考え方がある

https://artscape.jp/artword/index.php/%E5%8F%97%E5%AE%B9%E7%BE%8E%E5%AD%A6#:~:text=%E7%A4%BE%E4%BC%9A%E6%A7%8B%E9%80%A0%E3%81%AE%E5%A4%89%E5%8C%96%E3%82%84,%E5%AD%A6%E6%B4%BE%E3%81%AB%E3%82%88%E3%81%A3%E3%81%A6%E6%8F%90%E5%94%B1%E3%81%95%E3%82%8C%E3%81%9F%E3%80%82

芸術は作者だけではなく、最終的には鑑賞者によって完成するのだ🧐
0975😉三年英太郎🌈 ◆3CZBjOt3.Y (ワッチョイW ca86-zOhc)
垢版 |
2022/10/13(木) 10:56:06.34ID:zGrS5IOE0
ロゼッティのBeata Beatrixという有名な絵があるが、シュルレアリスムの画家のダリは、ベアトリクスの右側描かれた日時計をフロイト風にチンチンであると解釈して、彼女はチンチンを咥えて性的エクスタシーにあるとみなす🧐
https://en.m.wikipedia.org/wiki/Beata_Beatrix#/media/File%3ADante_Gabriel_Rossetti_-_Beata_Beatrix%2C_1864-1870.jpg

レオナルドの聖アンナ聖母子、あるフランスの学者が聖アンナの足元にある石ころが実は胎盤ではないかと言ったのだが、林達夫の『精神史』はそこから始まる論考である🧐
https://en.m.wikipedia.org/wiki/The_Virgin_and_Child_with_Saint_Anne_(Leonardo)#/media/File%3AL%C3%A9onard_de_Vinci%2C_sainte_Anne%2C_Louvre.jpg
0976名無しさん@英語勉強中 (テテンテンテン MM86-yynh)
垢版 |
2022/10/13(木) 11:06:17.26ID:ycUeUWBwM
>>973 と同様に、過去の出来事を表しているのだと思います。
historical presentは、日常的な過去の出来事を表すのにも使われます。
生き生きとその情景を描写する目的で。

>「モダンな」現在形での描写って
どういう意味なのかわからないですが・・・

↓’everyday conversation'でも使われるとあります。
https://en.wikipedia.org/wiki/Historical_present
0977名無しさん@英語勉強中 (ワッチョイW b39d-ZDf4)
垢版 |
2022/10/13(木) 11:21:20.11ID:LIC2lh280
ああそれもあるね
その時こいつがこうしてさあでこうしてこうしてこうなの〜
みたいな回想ではごく自然な日常的な用法だけど

件の文は現在形を地の文に使う現代文学の感じに見える
as if とか酒と言わない表現法とかあるから
日常的な会話じゃないかんじ
0978名無しさん@英語勉強中 (ワッチョイW b39d-ZDf4)
垢版 |
2022/10/13(木) 11:26:14.75ID:LIC2lh280
でも件の和訳はかなりセンスあると思うよ
俺よりもうまいと思う
0979ブタ耳 (ワッチョイ 4ee3-3fXN)
垢版 |
2022/10/13(木) 11:51:48.22ID:cmeNxSRc0
やはり historical present の線ですか。
辞書の例文としてどうなのよ、と文句を言いたくもなりますが、いったんそれで納得しておきます。

ところで愚痴聞いて!

問題の小説の一節、私と真野さんで参照している版が異なるため、訳してる原文の表現が異なっている可能性が微レ存じゃないですか。
なので1992年の改訂版以前の版を確認しようと思って、尼で中古のペーパーバック版取り寄せたんですよ。
事前に業者(名前出しちゃうけど「もったいない本舗」)とやり取りして、
「お問合せ頂いた件を確認したところ、1992年以前に出版された商品でございました。」
の言質を得ました。
で、届いたのがこれだよ。
https://imgur.com/3cw4lpO
返品はしないでおくけどね。

実はハードカバー版も海外の業者に発注してある。
問い合わせ結果:
Thanks for your email. We do understand your concern this copy was published by Simon & Schuster; 1st edition (March 30, 1984). I hope it helps.
これ信じて初版本の到着を待つわ。
相当時間かかると思うけどね。
ちなみに送料込で\3,000円ちょいでした。
0980ブタ耳 (ワッチョイ 4ee3-3fXN)
垢版 |
2022/10/13(木) 11:59:33.03ID:cmeNxSRc0
あ、業者からお詫びのメール来た。
返品しなくても全額返金してくれるらしい。
よし、許す。
0982名無しさん@英語勉強中 (オッペケ Sr03-ZDf4)
垢版 |
2022/10/13(木) 12:41:07.42ID:ZyvgVlYKr
コウビルドの和訳はわりと微妙
小学館オックスフォードコロケのはかなりいいように思う
0984名無しさん@英語勉強中 (US 0Hff-rqSc)
垢版 |
2022/10/13(木) 14:25:27.44ID:vzHpAZFIH
>>961
ご紹介ありがとうございます、実際に読めるのは少し先になりそうですが是非読んでみようと思います

皆様とは比べ物にならない浅学者ですが今後も合間を見て覗きに来るかと思います、その時はお手柔らかにお願いしますね
0985名無しさん@英語勉強中 (ワッチョイW ca80-uXp2)
垢版 |
2022/10/13(木) 18:47:07.99ID:ku6ylitM0
失礼します。
参考書に関係代名詞の例文で

The street I use to live on was very n
oisy at night, which made it difficult to sleep.
とあるのですが、
which made it difficult to sleep.
の部分のitって何故必要なんですか?whichに変えて消える訳ではないのですか?
ではなく別の所にかかるitだとしたらどこにかかるのでしょうか?
それともitは単に誤植なんでしょうか?
0986名無しさん@英語勉強中 (ワッチョイW b39d-ZDf4)
垢版 |
2022/10/13(木) 18:49:59.84ID:LIC2lh280
make o c でitはto sleepを受けてるんですよ
高校レベルの構文じゃなかったけ
0987名無しさん@英語勉強中 (ワッチョイW ca80-uXp2)
垢版 |
2022/10/13(木) 18:56:58.72ID:ku6ylitM0
そうでしたか!
無知、失礼致しました。
即答ありがとうございます。
0988名無しさん@英語勉強中 (ワッチョイW ca80-uXp2)
垢版 |
2022/10/13(木) 19:01:52.87ID:ku6ylitM0
直後のto sleepって意外でした。
0989名無しさん@英語勉強中 (ワッチョイ 4af0-SUfT)
垢版 |
2022/10/13(木) 21:02:47.32ID:fgZvb0I70
>>972
ブタ耳さんの訳を超えるものが思いつかない。「 一口・・・」の流れをまねて
しかも「全て」という言葉を使わずに近い意味となると「・・・一気に飲み干す」
くらいかな。・・・しかし、これだと、別の英語が使われそう。
0990名無しさん@英語勉強中 (ワッチョイ 4af0-SUfT)
垢版 |
2022/10/13(木) 21:08:26.31ID:fgZvb0I70
>>974
さすが博学の英ちゃん。
その考え方は知っていたけど、「受容美学」という言葉は知らなかった。
少し賢くなったかな、 ありがとう
0991名無しさん@英語勉強中 (ワッチョイ 9f5d-Vt39)
垢版 |
2022/10/15(土) 02:27:36.90ID:+5yfYDsi0
ロラン・バルト、現代思想の入門書を読めば大体出てきますね。
0993名無しさん@英語勉強中 (ワッチョイ 0f8f-Vt39)
垢版 |
2022/10/15(土) 11:46:51.15ID:KhI87yGM0
同格のofと形容詞句のofって機械的にはどうやって判別すればいいんでしょうか?
0994ブタ耳 (ワッチョイ 4fe3-Jfgl)
垢版 |
2022/10/15(土) 11:54:12.47ID:hGjqklXq0
>>975
ス⭐レ⭐チ

聖アンナと聖母子
https://collections.louvre.fr/en/ark:/53355/cl010066107
アンドレ・シャステルの言う「胎盤」とは赤い楕円のところで、「切断された胎児」とは青い円の箇所だと推測する。
https://imgur.com/hwMleg5
「胎盤」は納得できるとして、「胎児」の方はどうしてもそうは見えない。確かにホルモン焼きに出てきそうなブツではある。


岩窟の聖母
https://collections.louvre.fr/en/ark:/53355/cl010062373
若桑みどりの説によると、この岩窟は「神聖なるマリアの胎内」を表象していて、画面右上の出口が岩で塞がれているのは、「マリアの純潔な受胎」を示しているとのこと。
https://imgur.com/XsrgA3N
本人の講義を聴講したことがあるのだが、その際この岩に関して、「しっかり処女膜が残ってるわけですね」みたいな、今となってはかなり際どい冗句をかまして笑いをとっていた。
更に、この岩の上にはオリーブが描かれており、これは「聖なるキリストの勝利」を示している、とも。
(若桑みどり, 『薔薇のイコノロジー』, pp.55-56)

一方、ロンドンのナショナルギャラリーに所蔵されている同名の絵画では、この岩の上には何も描かれていない。したがってこれは贋作だと考えた方が良い、とも講義では述べておられた。
https://www.nationalgallery.org.uk/paintings/leonardo-da-vinci-the-virgin-of-the-rocks

ス⭐マ⭐ソ
0995名無しさん@英語勉強中 (ワッチョイ 4fe3-Jfgl)
垢版 |
2022/10/15(土) 12:01:19.15ID:hGjqklXq0
>>993
機械的には判断できないでしょ。
その都度、意味を考えないと。

例の、"the mystery of the human mind" に関しては、私は「人間の心という神秘」だととりました(>>752)。

これに対する、ソフィスト氏の反論。
「人間の心という神秘」という訳し方のみが正解で、「人間の心の神秘」は間違いだとする感覚にも問題がありますね。(>>757)

これはまったくその通りですね。どちらとも考えられると思います。
10011001
垢版 |
Over 1000Thread
このスレッドは1000を超えました。
新しいスレッドを立ててください。
life time: 45日 18時間 25分 10秒
10021002
垢版 |
Over 1000Thread
5ちゃんねるの運営はプレミアム会員の皆さまに支えられています。
運営にご協力お願いいたします。


───────────────────
《プレミアム会員の主な特典》
★ 5ちゃんねる専用ブラウザからの広告除去
★ 5ちゃんねるの過去ログを取得
★ 書き込み規制の緩和
───────────────────

会員登録には個人情報は一切必要ありません。
月300円から匿名でご購入いただけます。

▼ プレミアム会員登録はこちら ▼
https://premium.5ch.net/

▼ 浪人ログインはこちら ▼
https://login.5ch.net/login.php
レス数が1000を超えています。これ以上書き込みはできません。

ニューススポーツなんでも実況